Daily MCQ. Source: NIOS

Topics-

1- History.

2- Polity. Question 2

3- Economics. Consider the following statement regarding Election Commission of 4- Environment and Ecology. 1. The chief election commissioner holds his 5- Science and tech. office until the pleasure of the president 6- Geography. 2. Any other election commissioner or a 7- 4 MCQ of Current Affairs. regional commissioner cannot be removed from office except on the recommendation of the FEB 1 chief election commissioner.

Question 1 Which of the following statement is correct?

The object of the Lyall commission of 1897 a) 1 only was to b) 2 only (a) Define the jurisdiction of the Central and Provincial Governments. c) Both 1 and 2

(b) Define the powers of the Governor General d) Neither 1 nor 2 for India. Answer: b (c) State interference in food trade in the event of famine Explanation:

(d) Improve the relationship between the Government of India and the princely states. Article 324 of The Constitution of India mentions the provisions to safeguard and Answer: c ensure the independent and impartial functioning of the Election Commission which Explanation: is as follows.  It was set up in 1878 under the Chairmanship of Sir Richard Strachey.  The chief election commissioner is  The commission recommended state provided with security of tenure. interference in food trade in the event of  He cannot be removed from his office famine. except in the same manner and on the  India witnessed another major famine in same grounds as a judge of the 1896-97. Supreme Court.

 In other words, he can be removed by Answer: d the President on the basis of a resolution passed to that effect by both the Houses Explanation: of Parliament with a special majority, either on the ground of proved  An Open Market Operation (OMO) is misbehavior or incapacity. the buying and selling of government  Thus, he does not hold his office until securities in the open market, hence the the pleasure of the president, though he nomenclature. is appointed by him. Hence statement  It is done by the central bank in a 1 is correct country (the RBI in India).  The service conditions of the chief  When the central bank wants to infuse election commissioner cannot be varied liquidity into the monetary system, it to his disadvantage after his appointment. will buy government securities in the open market. Hence statement 1 is  Any other election commissioner or a regional commissioner cannot be correct. removed from office except on the  The use of open market operations as a recommendation of the chief election monetary policy tool ultimately helps commissioner. Hence statement 2 is the Fed pursue its dual correct. mandate—maximizing employment, promoting stable prices—by Source: Laxmikanth influencing the supply of reserves in Question 3 the banking system, which leads to If the RBI has decided to infuse Rs.10, 000 interest rate changes. Hence statement crore liquidity in the banking system by buying 2 and 3 are correct. government securities through Open Market  Open market purchases raise bond Operations (OMO), what will happen prices, and open market sales lower bond prices. When the Federal Reserve 1. Liquidity will increase buys bonds, bond prices go up, which in turn reduces interest rates. 2. Leads to interest rate changes.  Open market purchases increase the money supply, which makes money 3, Raise bond prices, less valuable and reduces the interest rate in the money market. Select the correct answer using the code given below: Source: NIOS

Question 4 a) 1 and 2 only Consider the following statement regarding b) 2 only dead zones c) 1 and 3 only 1. Hypoxic zones in oceans where oxygen d) 1, 2 and 3 only level is high

2. Dead zones are irreversible Answer: a

Which of the following statement is correct? Explanation: a) 1 only  It is a fuel cell that combines hydrogen and oxygen to produce electricity with b) 2 only water and steam as the only byproducts. Hence statement 1 is correct. c) Both 1 and 2  The excess energy can be stored on board in ion lithium batteries. d) Neither 1 nor 2  It is a climate friendly fuel as it does not emit carbon dioxide or particulate Answer: c matter as the case with conventional fuels like diesel, coal etc. Hence Explanation: statement 2 is incorrect.  A fuel cell is composed of an anode, a  Dead zones (Hypoxic zones) are areas cathode, and an electrolyte membrane. of the ocean (occasionally in lakes and  A fuel cell works by passing hydrogen even rivers) where oxygen has fallen to through the anode of a fuel cell and such low levels that most marine life oxygen through the cathode. cannot survive. Hence statement 1 is  Fuel cells do not need to be periodically correct recharged like batteries, but instead  Dead zones are reversible if their continue to produce electricity as long causes are reduced or eliminated. as a fuel source is provided Hence statement 2 is correct Source: NCERT Source: General Science Question 5

Consider the following statement Hydrogen fuel cell Question 6

1. It is a fuel cell that combines hydrogen and Consider the following statement: oxygen to produce electricity 1. Jet streams generally forms upper layers of 2. It emits carbon dioxide or particulate matter troposphere like fossil fuels 2. Jet Streams extend from equator to the poles in both hemispheres Which of the following statement is correct? a) 1 only Which of the following statement is correct? b) 2 only a) 1 only c) Both 1 and 2 b) 2 only d) Neither 1 nor 2 c) Both 1 and 2

d) Neither 1 nor 2 b) Microlithic site

Answer: a c) An old city core of Bhubaneswar

Explanation: d) Temple complex at Konark  The Jet Stream is a geostrophic Answer: c wind blowing horizontally through the upper layers of the troposphere, Explanation: generally from west to east, at an altitude of 20,000 - 50,000 feet.  Ekamra Kshetra’ comprises the area Hence statement 1 is correct. of the old city of Bhubaneswar that  Jet Streams develop where air masses forms the center of this temple of differing temperatures meet. So, architecture. usually surface temperatures  Started being celebrated as a Shaiva determine where the Jet Stream will centre - at least from 7th century A.D. form.  Lingaraj Temple was the centre of  Greater the difference in temperature, Ekamra Kshetra. • Divided into faster is the wind velocity inside the asta-ayatana - ritualistically and jet stream. symbolically connected to Lingaraja  Jet Streams extend from 20 degrees Temple. • Kshetra - area / religious latitude to the poles in both center; Ekamra - mango tree. hemispheres. Hence statement 2 is incorrect Source: The Hindu

 There are two permanent jet streams – subtropical jets at lower latitudes and polar front jets at Question 8 mid-latitudes.  Both the Northern and Southern Consider the following statement hemispheres have jet streams, although the jet streams in the north are more 1. The state election commissioner enjoys the forceful due greater temperature same status as the Election Commissioner of gradients. India.

Source: NCERT 2. Only after the polls are over can the SECs’ decisions or conduct be questioned through an election petition.

Current affairs Question 7 Which of the following statement is correct? The term ‘Ekamra Kshetra’, recently seen in news, is associated with which of the a) 1 only following? b) 2 only a) A Buddhist site

c) Both 1 and 2 Consider the following statements with regard to impacts of low sovereign credit d) Neither 1 nor 2 ratings:

Answer: c 1. Lead to Indian government and companies’ borrowing at a very high interest rate. Explanation:  The Constitution of India vests in the 2. It will encourage investments. State Election Commission, consisting Which of the statements given above is/are of a State Election Commissioner, the correct? superintendence, direction and control of the preparation of electoral rolls for, a) 1 only and the conduct of all elections to the Panchayats and the Municipalities b) 2 only (Articles 243K, 243ZA) c) Both 1 and 2  The provisions of Article 243K of the Constitution, which provides for setting d) Neither 1 nor 2 up of SECs, are almost identical to Answer: a those of Article 324 related to the EC. In other words, the SECs enjoy the Explanation: same status as the EC. Hence statement 1 is correct  A sovereign credit rating is an  Courts cannot interfere in the conduct independent assessment of the of polls to local bodies and creditworthiness of a country or self-government institutions once the sovereign entity electoral process has been set in  Articlemotion. 243 -O of the Constitution bars Impact of low credit ratings: interference in poll matters set in motion by the SECs; Article 329 bars  Lead to Indian government and interference in such matters set in companies’ borrowing at a very high motion by the EC. interest rate. Hence statement 1 is  Only after the polls are over can the correct SECs’ decisions or conduct be  Foreign portfolio investments will go questioned through an election petition. down. Hence statement 2 is incorrect Hence statement 2 is correct  That is becomes costlier for the Indian  These powers enjoyed by the SECs are government as well as all Indian the same as those by the EC. companies.

Source: The Hindu

Question 10 Question 9

Which of the following statement is correct with respect to Pradhan Mantri Jan Arogya Source: The Hindu Yojana?

1. PMJAY is based on portable approach FEB 2 2. Beneficiaries are identified using Socio-Economic Caste Census (SECC) data. Question 1

Which of the statements given above is/are Consider the following statement correct? 1. Mahayana believes in salvation by work a) 1 only 2. Hinayana believes in salvation by faith b) 2 only Which of the following statement is/are c) Both 1 and 2 correct? d) Neither 1 nor 2 a) 1 only

Answer: c b) 2 only

Explanation: c) Both 1 and 2

 PMJAY offers a sum insured of 5 d) Neither 1 nor 2 lakh per family for secondary care (which doesn’t involve a super Answer: d specialist) as well as tertiary care (which does). For the beneficiaries, Explanation:  Itthis is isan a entitlement free scheme.-based scheme that  After the death of Buddha, Buddhism targets the beneficiaries as identified by latest Socio-Economic Caste was divided into two sects namely Census (SECC) data. Hence Mahayana and Hinayana. The terms statement 2 is correct. Hinayana (Lesser Vehicle or Modest  Individuals can walk into Vehicle) and Mahayana (Greater any empanelled hospital that can Vehicle or Vast Vehicle) originated in process cashless payments. The Prajnaparamita Sutras (The Sutras  Once identified by the database, the on Far-Reaching Discriminating beneficiary is considered insured. Awareness, The Perfection of Wisdom  The insurance cost is shared by the center and the state mostly in the Sutras). ratio of 60:40.  Mahayana worships the bodhisattvas  PMJAY is portable, which means the and read the Mahayana sutras while beneficiary can avail treatment in any Hinayanists do not perform these. of the states that have implemented the scheme. Hence statement 1 is  Mahayana Buddhism followers think correct that The Buddha is a God because they think that the Buddha came down to

earth to help people cross the sea of life.  Abolition of provincial dyarchy and So the Buddha can be worshipped as a introduction of dyarchy at centre. Hence God because he is eternal and comes both statements are correct down to earth.  Abolition of Indian Council and introduction of an advisory body in its  On the other hand Hinayana Buddhists place. think that The Buddha was a Human  Provision for an All India Federation with instead of a God because they think The British India territories and princely states. Buddha was simply a man who found a  Elaborate safeguards and protective way to Nirvana. instruments for minorities.  Supremacy of British Parliament.  Mahayana believes in salvation by faith  Increase in size of legislatures, extension of Hinayana believes in salvation by franchise, division of subjects into three work. Hence both the statements are lists and retention of communal electorate. incorrect.  Separation of Burma from India

Source: NCERT Source: Laxmikanth

Question 2 Question 3 Consider the following statement regarding With reference to Goods and Service Tax 1935 Act: regime, consider the following statements. 1. Diarchy was abolished at the provincial levels.  Only parliament has the power to make 2. Diarchy introduced at the center. laws with respect to GST where the supply of good and services takes place Which of the following statement is/are correct? in the course of interstate trade/commerce. a) 1 only  Central Government will levy and b) 2 only collect CGST and the State will levy c) Both 1 and 2 and collect SGST on intra-state supply d) Neither 1 nor 2 of goods or services.

Answer: c  The Goods and Services Tax Council is a statutory body whose chairman is Explanation the Union Finance Minister. Salient Features of the Government of India  Vice Chairperson of GST council is Act 1935 were as follows: finance secretary

Which of the above statements is/are correct?  Article 246A(1): Parliament and the state legislatures have power to make (a) 1 and 3 only laws with respect to GST imposed by (b) 1 and2 only the Union or by such state.  Article 246A(2), Parliament has (c) 1, 2 and 4 only exclusive power to make laws with (d) All the above respect to GST where the supply of good and services takes place in the Answer: b course of inter-State trade/commerce. Explanation:  Central Government will levy and collect CGST and the State will levy  Goods and Services Tax Council and collect SGST on intra-state supply (Article 279A) is a constitutional body, of goods or services. hence statement 3 is incorrect  The Centre will also levy and collect  Members of the council IGST on inter-state supply of goods or 1. Union Finance Minister as services. Chairperson 2. Union Minister of State in  Constitution provides for compensating charge of Revenue or Finance; the States for loss of revenue arising 3. Minister in charge of out of implementation of the GST.

Finance/Taxation or any other Source: current affair Minister nominated by each Question 4 State Government; 4. Vice Chairperson to be chosen Which of the following statement is correct among the members. Hence regardingPrevention of Cruelty to Animals Act, 1960? statement 4 is incorrect.

 It is tasked with the duties to make 1. India’s first piece of legislation on animal important decisions and changes in welfare GST. 2. It aims to protect both domestic and captive  Any tax that was levied by the animals from unnecessary pain and Central/State Government on the supply suffering. of goods or services was converted into Which of the above statements is/are correct? GST.

a) 1 only the Indian region for all three armed forces b) 2 only Which of the above statements is/are correct? c) Both 1 and 2 (a) 1 only d) Neither 1 nor 2 (b) 2 only Answer: c (c) Both 1 and 2 Explanation: (d) Neither 1 nor 2  India’s first piece of legislation on animal welfare law. The law lists down Answer: c what constitutes an offence of animal Explanation: cruelty. Hence statement 1 is correct  It aims to protect both domestic and GSAT-7A captive animals from unnecessary pain • One of India’s most advanced military and suffering. Hence statement 2 is satellites. correct. • Launched by ISRO.  It establishes the Animal Welfare • Built to provide communication Board of India. capability to the users in Ku-band over  Animal Welfare Board overlooks the the Indian region. enforcement of the Act and further • To be used by the Indian Air Force and contributes towards animal welfare. the Indian Army.

Source: current affair • Modeled on the GSAT-7 satellite built for the Indian Navy. Question 5 • Uses advanced Gregorian antenna to Consider the following statement improve its performance

 GSAT-7A is one of India’s most Source: ISRO advanced military satellites. Question 6  It has built to provide communication capability to the users in Ku-band over Consider the following statement regarding temperature inversion:

1. Intense thunderstorms and tornadoes are rainfall and no showers. So, it causes a also associated with inversion of temperature problem for agricultural productivity.

2. It lowers the visibility  Lower visibility: Fog is formed due to the situation of warm air above and 3. Diurnal variations in temperature tend to be cold air below, and hence visibility is very high. reduced which causes disturbance in transportation. 4. Air pollutants such as dust particles and smoke do not disperse on the surface.  Thunderstorms and tornadoes: Intense thunderstorms and Which of the following statement is/are tornadoes are also associated with correct? inversion of temperature because of the intense energy that is released after a) 1 and 2 only an inversion blocks an area’s normal convention patterns. b) 1 and 3 only  Diurnal variations in temperature tend c) 1, 2 and 3 only to be very small. Statement 3 is incorrect d) 1, 2 and 4 only

Answer: d Source: NCERT Explanation:

Temperature inversion: It is a reversal of the normal behavior of temperature in the Current affairs troposphere. Under this meteorological Question 7 phenomenon a layer of warm air lies over the cold air layer. World Food price index is published by

Temperature inversion determines the (a) Food and Agricultural Organization precipitation, forms of clouds, and also causes frost due to condensation of warm air due to its (b) International food policy Research cooling. Institute  Dust particles hanging in the (c) World Food Programme air: Due to inversion of temperature, air pollutants such as dust particles and (d) International Fund For Agricultural smoke do not disperse on the surface. Development.  Stops the movement of air: It causes the stability of the atmosphere that Answer: a stops the downward and upward movement of air. Explanation:  Less rainfall: Convection clouds can not move high upwards so there is less • The FAO Food Price Index (FFPI) is a measure of the monthly change in

international prices of a basket of food Explanation: commodities. National Nutrition Mission: • It measures changes for a basket of  Flagship programme of the Ministry of cereals, oilseeds, dairy products, meat Women and Child Development and sugar. (MWCD Base Period: 2014-16. •  Aimed at improving the nutritional Source: The Hindu status of Children from 0-6 years, Adolescent Girls, Pregnant Women and

Lactating Mothers Question 8  Convergence with various welfare Consider the following statement with programmes regard to the National Nutrition Mission.  Reduction in stunting from 38.4% as per the National Family Health 1. It aims reduction in stunting from 38.4% as Survey-4 to 25% by 2022. To reduce per the National Family Health Survey-4 to stunting, under-nutrition and reduce 25% by 2022. low birth weight by 2%, 2%, and 2% 2. Executive Committee (EC) of the National per annum respectively. Hence Nutrition Mission is headed by Union Health statement 1 is correct Minister.  Bring down anemia among young

3. Bring down anemia among young children, children, women and adolescent girls women and adolescent girls by 3% per year by 3% per year, hence statement 3 is correct. Which of the following statement(s) is/are  Reduce under-nutrition and low incorrect? birth-weight by 2% each year. a) 1 only Committees constituted to achieve b) 2 only objectives of National Nutrition Mission: c) 1 and 2 only  National Nutrition Council (NNC)-headed by Vice-Chairman of d) 1 and 3 only NITI Aayog Answer: b

 Executive Committee (EC) of the  The Palayakkarar system evolved in National Nutrition Mission-headed by TamilNadu with the extension of Secretary of the Ministry of Women Vijayanagar rule into . and Child Development, hence Hence statement 1 is correct statement 2 is incorrect  Each Palayakkarar was the holder of a  National Technical Board on Nutrition territory or Palayam granted in return (NTBN)-headed by a Member of NITI for military service and tribute. Aayog, V K Paul.  In most cases, the Palayakkarars gave little attention to perform their duties Source: The Mint and they were interested in increasing Question 9 their own powers.  Came to constitute a powerful force in With reference to Palayakkarar system, the political system of . consider the following statements  Regarded themselves as independent, 1. The Palayakkarar system evolved in Tamil sovereign authorities within their Nadu with the extension of Vijayanagar rule respective Palayams. Hence statement into Tamil Nadu 2 is correct  Argued that their lands had been 2. Poligars were feudatories who controlled handed down to them across a span of group of villages known as ‘palayam’ sixty generations. Which of the above given statements is/are  Their claims were brushed aside by the correct? East India Company.  One of the blocs – led by Veera (a) 1 only Pandiya Kattabomman. (b) 2 only Source: The Hindu (c) Both 1 and 2 Question 10 (d) Neither 1 nor 2 Consider the following statements

Answer: C 1. Neutron stars comprise one of the possible evolutionary end-points of high mass stars Explanation 2. Once energy production stops the squeezing electrons and protons transferred to neutron star

Which of the following is/are the correct? pulsar if its magnetic field is favorably aligned with its spin axis. (a) 1 only

(b) 2 only Source: The Hindu

(c) Both 1 and 2

(d) Neither 1 nor 2 FEB 3 Question 1 Answer: C With reference to the period of ancient Explanation India, the places , Nagarjunakonda and Somapura were well Black Holes- The term ‘black hole’ was coined known as in the mid-1960s by American Physicist John Archibald Wheeler. It refers to a point in space a) Ancient capital cities where the matter is so compressed as to create a gravity field from which even light cannot b) Ancient Buddhist sites escape. Black-holes were theorized by Albert Einstein in 1915. c) Ancient Buddhist learning center d) Places of exquisite stone art and architecture Supernova- A supernova is the explosion of a star. It is the largest explosion that takes place Answer: C in space. A supernova happens where there is a change in the core, or centre, of a star. A Explanation: change can occur in two different ways, with both resulting in a supernova.  Pushpagiri in Jajpur district, Odisha (on Langudi hills). Buddhist seat of Neutron stars- learning. Mentioned in the travelogues of Chinese traveler Huien Tsang.  Neutron stars comprise one of the possible evolutionary end-points of  Nagarjunakonda in Guntur district of high mass stars. Hence statement 1 is A.P. Neolithic and Megalithic culture. correct Buddhist learning center.  Somapura: In Naogaon District,  Once the core of the star has completely burned to iron, energy Bangladesh. A Buddhist Mahavihara. production stops and the core rapidly Built by Pala king Dharmapala. collapses, squeezing electrons and protons together to form neutrons and Source: NIOS neutrinos. Hence statement 2 is correct Question 2  A star supported by neutron Consider the following statement degeneracy pressure is known as a ‘neutron star’, which may be seen as a

1. Once a member of the assembly incurs of six months after the Proclamation has disqualification under the Tenth schedule, he ceased to operate. Hence statement 2 is cannot permit to contest again during the term correct for which he was elected.  As per Supreme Court judgment disqualification will not bar a 2. Membership of a house co terminus with the disqualified member from contesting term of five years of the house except in any future election during the circumstances mentioned in constitution remaining period of the house. Hence Which of the following statement(s) is/are is statement 1 is incorrect. correct? Source; Laxmikanth a) 1 only Question 3 b) 2 only Consider the following statement regardingPurchasing Manager’s Index c) Both 1 and 2 (PMI)

1. PMI is an indicator of business activity- in d) Neither 1 nor 2 the manufacturing and services sectors. Answer: b 2. The PMI is usually released at the start of Explanation: every year

 Article 172 says every Legislative 3. PMI of the previous month is higher than the Assembly of every State, unless sooner PMI of the current month, it represents that dissolved, shall continue for five years the economy is contracting from the date appointed for its first Which of the following statement is/are meeting and no longer and the correct? expiration of the said period of five years shall operate as a dissolution of (a) 1 and 2 only the Assembly: Provided that the said (b) 2 and 3 only period may, while a Proclamation of Emergency is in operation, be extended (c) 1 and 3 only by Parliament by law for a period not (d) 1, 2 and 3 exceeding one year at a time and not extending in any case beyond a period Answer: c

Explanation: With reference to Olive Ridley Turtles, consider the following statements:  PMI is an indicator of business 1. Gahiramatha marine sanctuary is known activity- in the manufacturing and services place for Olive Ridley Turtles sectors. Hence statement 1 is correct. 2. The Olive Ridley Turtles are one of the  It is a survey-based measure that asks the smallest and most abundant of all sea turtles respondents about changes in their found in the world,

perception about key business variables as Which of the following statement is/are compared with the previous month correct?

 The PMI is usually released at the start a) 1 only of every month. It is, therefore, b) 2 only

considered a good leading indicator of c) Both 1 and 2 economic activity. Hence statement 2 is d) Neither 1 nor 2 incorrect. Answer: c The PMI is a number from 0 to 100. Explanation:

 PMI above 50 represents an  Gahiramatha, located in Odisha, is known as the world’s largest Olive expansion when compared to the previous Ridley rookery. Hence statement 1 is month; correct  The Olive Ridley Turtles are one of the  PMI under 50 represents a smallest and most abundant of all sea contraction, and turtles found in the world, inhabiting warm waters of the Pacific, Atlantic  A reading at 50 indicates no change. and Indian oceans. Hence statement 2 is correct. If PMI of the previous month is higher than the PMI of the current month (as is the case  These turtles, along with their cousin, mentioned above), it represents that Kemps Ridley turtle, are best known the economy is contracting. Hence statement for their unique mass nesting called 3 is correct Arribada, where thousands of females Source: current affairs come together on the same beach to lay eggs. • IUCN Red list: Vulnerable; CITES: Appendix 1, CMS: Appendix 1. Question 4 Source: Current affairs

Question 5

Consider the following statements regarding  Metabolism of otherwise complex micro biome: indigestible carbohydrates and fats  Production of essential vitamins 1. The collective genome of all  Maintaining immune systems and it micro-organisms contained within the human does not trigger auto immune disease. body. Hence statement 4 is incorrect 2. It acts as a first line of defense against  Acting as a first line of defense against pathogens. pathogens.  Determines how one responds to a 3. It plays vital role in production of essential particular drug treatment vitamins Source: Science Reporter 4. It triggers autoimmune disease

Which of the following statement is/are correct? Question 6 Red sea is a border of which of the following (a) 1 and 4 only countries?

(b) 2 and 3 only 1) Yemen

(c) 1 2, and 3 only 2) Somalia 3) Eritrea (d) 1, 2 and 4 only 4) Egypt. Answer: c Select the correct answer using the code given Explanation: below: (a) 1 and 3 only  The collective genome of all micro-organisms contained within the (b) 2 and 3 only human body, residing inside tissues & bio-fluids is called Human (c) 1 3, and 4 only Microbiome. It includes bacteria, archaea, fungi, protists and viruses. (d) 1, 2 and 4 only Hence statement 1 is correct  Human microbiome makes up around Answer: c 2% of the body mass of the adult. There are 10 times as many outside organisms Explanation: as there are human cells in human body. Microbial communities play a key role in many aspects of host physiology:

Explanation:

 Hong Kong International Convention’ intends to address all the issues around ship recycling, including the probable presence of environmentally hazardous substances such as asbestos, heavy metals, hydrocarbons, ozone depleting substances and others.  It aims to address concerns about working and environmental conditions in many of the world's ship recycling facilities.

Source: The Hindu

Question 8 3. Eastern shore: Saudi Arabia. Yemen. 4. Western Consider the following statements with shore: Egypt. Sudan. Eritrea. Djibouti regard to the Food Corporation of India 5. Somalia borders around Gulf of Aden. (FCI):

1. FCI is an executive body Current affairs 2. It has function in both procurement and distribution

3. It comes under Ministry of Agriculture and Question 7 Farmers Welfare

‘The Hong Kong International Convention’, Which of the following statement is/ are recently seen in news, is related to: correct? (a) To control terror financing. (a) 1 and 3 only

(b) To strengthen governance. (b) 2 only

(c) Global coordination against cyber warfare (c) 1 and 2only

(d) Safe and Environmentally Sound Recycling (d) 1, 2 and 3 only of Ships. Answer: b Answer: d

Explanation: d) Neither 1 nor 2

 Food Corporation of India (FCI) is Answer: b a Public Sector Undertaking, under Explanation: the Department of Food & Public  Budget of Government of India gives a complete picture of the estimated Distribution, Ministry of Consumer receipts and expenditure of the Affairs, Food and Public Distribution. Government for the upcoming financial year - based on the budget figure of the Hence statement 3 is incorrect. past two years.  FCI is a statutory body set up in 1965  The term ‘budget’ is nowhere mentioned in the Indian Constitution. under the Food Corporations Act Hence statement 1 is incorrect. 1964. It was established against the  Article 112 of Indian Constitution - annual financial statement – a statement backdrop of major shortage of grains, of the estimated receipts and especially wheat. Hence statement 1 is expenditure of the Government of India for the upcoming financial year. incorrect  The Revenue and the Capital sections together, make the Union Budget. Major Activities Undertaken by FCI Hence statement 2 is correct.

 Distribution Source: The Mint  Procurement Question 10  Public distribution system Consider the following statement regarding Disinvestment Source: The Hindu

1. In the short run, it is helpful in financing the

increasing fiscal deficit. Question 9 2. It can help in generating a better Consider the following statements. environment for investment.

1. The term ‘Budget’ is mentioned in the Indian Constitution. Which of the above statement(s) is/are correct? 2. The Revenue and the Capital sections together, make the Union Budget a) 1 only b) 2 only Which of the above statement(s) is/are correct? c) Both 1 and 2 d) Neither 1 nor 2 a) 1 only b) 2 only Answer: c c) Both 1 and 2 Explanation:

Benefits of disinvestment a) 1 only

 Helps raising valuable resources for b) 1, 2 and 4 only the government.  Government can focus more on core c) 3 only activities.  Result in a leaner government - d) 2, 3, and 4 only reduction in the number of ministries and bureaucrats. Answer: b  Benefits the Markets and Economy by bringing about greater efficiencies. Explanation:  Letting go of certain assets is in the  On August 16, 1932, the British long term interest of the tax payers. Prime Minister, Ramsay  In the short run, it is helpful in MacDonald, announced financing the increasing fiscal deficit. the Communal Award which Hence statement 1 is correct provided for separate  It can help in generating a better electorates for the ‘Depressed environment for investment. Hence Classes’, the Muslims, the Europeans, statement 2 is correct the Sikhs, the Anglo-Indians and the Indian-based Christians. Source: The Hindu  Initially, Ambedkar was in favors of the award, as according to him political solutions like separate FEB 4 electorate would work for upliftment of depressed classes. Question 1  However, after the series of The Poona –pact included which of the negotiations both Gandhiji and following? Ambedkar agreed to a solution called Poona Pact and thereby withdrawing 1.Depressed class candidates to be elected by separate electorate for depressed a joint electorate classes.

2. Representation of the depressed classes in  Representation of Depressed the public services while earmarking a portion Classes: In a settlement negotiated with of the educational grant for their uplift. Mahatma Gandhi, Ambedkar agreed for depressed class candidates to be 3. Ambedkar was against the communal award elected by a joint electorate. announced by British Prime Minister, Ramsay  The Poona Pact assured a fair MacDonald. representation of the depressed classes in the public services while 4. Madan Mohan Malaviya signed it on behalf of Gandhi. earmarking a portion of the educational grant for their uplift.

Source: spectrum Select the correct answer using the code given below

Question 2 also ordinances, orders, regulations, notifications, etc. Which part of the Constitution of India Source: Laxmikanth declares the ideal of political democracy? a) Directive principles of state policy Question 3 b) Fundamental rights Which of the following statements is/are correct with reference to PM Street Vendor c) Preamble Atma Nirbhar Nidhi (PM SVANidhi) ? 1. It is a Central Sector Scheme launched by d) Fundamental duties Ministry of Housing and Urban Affairs with collaboration with RBI and NABARD. Answer: b 2. It provides a special micro-credit facility for Explanation: street vendors up to Rs. 10,000 at subsidized rate of interest. • Articles 12-35 of Indian Constitution deal with Fundamental Rights. These 3. It is available to all street vendors engaged human rights are conferred upon the in vending in urban areas as on or before citizens of India for the Constitution March 24, 2020. tells that these rights are inviolable. Right to Life, Right to Dignity, Right to Select the correct answer using the code given Education etc. all come under one of the below. six main fundamental rights. a) (a) 1 only They are applied without discrimination • b) (b) 2 only on the basis of race, religion, gender, etc. Significantly, fundamental rights c) (c) 2 and 3 only are enforceable by the courts, subject to certain conditions. It establishes d) (d) 1 and 3 only political democracy. Answer: c • Fundamental rights are very important because they are like the backbone of Explanation: the country. They are essential for  Ministry of Housing and Urban safeguarding the people’s interests. • According to Article 13, all laws that are Affairs signed MoU with Small Industries violative of fundamental rights shall be Development Bank of India (SIDBI) in void. Here, there is an express provision order to engage SIDBI as the for judicial review. The SC and the Implementation Agency for the scheme. High Courts can declare any law Hence statement 1 is incorrect. unconstitutional on the grounds that it is violative of the fundamental rights.  SIDBI will manage the credit guarantee to Article 13 talks about not just laws, but the lending institutions through Credit

Guarantee Fund Trust for Micro and Small Which of the following statement is/are Enterprises (CGTMSE). correct?  It is a special micro-credit facility plan to a) 1 only provide affordable loan of up to ₹10,000 to more than 50 lakh street vendors, who had b) 2 only their businesses operational on or before 24 March 2020. c) Both 1 and 2  It will manage the credit guarantee to the d) Neither 1 nor 2 lending institutions through Credit Guarantee Fund Trust for Micro and Small Answer: c Enterprises.  Under the scheme, vendors can avail Explanation: working capital loan of up to ₹10,000,  Wetlands are areas where water is the which is repayable in monthly installments primary factor controlling the within one year. environment and the associated plant and  On timely/early repayment of the loan, an animal life. They occur where the water interest subsidy of 7% per annum will be table is at or near the surface of the land, or where the land is covered by water. credited to the bank accounts of beneficiaries through Direct Benefit  Wetlands are defined as: "lands Transfer (DBT) on six-month basis. transitional between terrestrial and aquatic  The scheme is applicable to vendors, eco-systems where the water table is usually at or near the surface or the land is hawkers, thelewalas, rehriwalas, covered by shallow water". theliphadwalas in different areas/contexts who supply goods and services. Street Montreux Record vendors belonging to the surrounding peri-urban/rural areas are also included  It is maintained as part of the Ramsar List.  Montreux Record is a register of wetland Source: current affairs sites on the List of Wetlands of International Importance where changes in ecological character have occurred, are occurring, or are likely to occur as a Question 4 result of technological developments, Consider the following statement pollution or other human interference.  Two wetlands of India are in Montreux 1. Montreux Record is a register of wetland Record: Keoladeo National Park sites on the List of Wetlands of International (Rajasthan) and Loktak Lake (Manipur). Importance Chilka lake (Odisha) was placed in the record but was later removed from it. 2. Keoladeo National Park (Rajasthan) and Loktak Lake (Manipur) are only two wetlands Source: Shankar IAS in India in Montreux Record Question 5

Consider the following statement regarding well-tolerated by healthy individuals Messenger RNA (mRNA) vaccine: with few side effects. Production: Vaccines can be produced 1 Messenger RNA (mRNA) vaccine has high more rapidly in the laboratory in a efficiency compare to other vaccines process that can be standardized, which 2. Messenger RNA (mRNA) vaccines are improves responsiveness to emerging infectious outbreaks

Which of the following statement is/are Source: current affairs correct? Question 6 a) 1 only The concerns of Madhesi population are b) 2 only discussed in relation to the affairs of c) Both 1 and 2 a) Tibet d) Neither 1 nor 2 b) Pakistan

Answer: a c) Afganistan

Explanation: d) Nepal

• mRNA vaccines are a new type of Answer: d vaccine to protect against infectious Explanation: diseases. Messenger RNAs (mRNA) are one of the types of RNA that is  The Madhesi people are hill dwellers in transcribed from DNA and travels into the Nepal. The Buddha is said to have a cell's cytoplasm where it's translated been born here, in the Buddhist by ribosomes into proteins. pilgrimage site of Lumbini.

Benefits of RNA Vaccines  They retain cultural (and culinary) ties with various cultural groups living in Safe and non-infectious: RNA north India, Bihar, and West Bengal. vaccines are not made with pathogen particles or inactivated pathogen, so are  Roughly half of Nepal's 30-million non-infectious. population lives in the Terai. The RNA does not integrate itself into the Madhesi make up about 20 percent of host genome and interact with our the total population, with the notable DNA. The RNA strand in the vaccine is exclusion of the Tharu people, who are degraded once the protein is made. native to Terai. Efficacy: Clinical trials found that  Madhesh, as a political term, was first mRNA vaccines can generate a stronger bandied about in the 1940s, with a type of immunity as compared to political movement calling for regional traditional vaccines, and are

autonomy emerging a few years later in south and southwest, Bangladesh to the 1951. west, and India to the northwest.  It is major crisis between India and Nepal  Nepal adopted a new constitution in 2015, India unhappy as it did not provide adequate safeguards for Madheshis - unrest in southern Nepal has the potential to destabilize the Indian side of the border.

Source: Human Geography. Current affairs Source: The Hindu

Question 7 Question 8

Consider the following countries. Consider the following statements about Lok Adalat : 1. India 1. Lok adalat uphold Gandhian principle 2. Bangladesh 2. Both compoundable and no compoundable 3. Vietnam cases come under Lok Adalat jurisidiction 4. Thailand 3. Every award made by a Lok Adalat shall 5. Laos be final and binding on all the parties to the dispute. Which of the above countries share border with Myanmar? 4. There is no court fee payable when a matter is filed in a Lok Adalat a) 1, 2, 4, 5 only Which of the following statement is/are b) 1, 2, 3, 4 only correct? c) 1, 3, 4, 5 only d) 1, 2, 3, 4 and 5 a) 1 only

Answer: a b) 1, 2 and 4 only

Explanation: c) 3 only

 The country is bordered by China to the d) 1, 3, and 4 only north and northeast, Laos to the east, Thailand to the southeast, the Answer: d Andaman Sea and Bay of Bengal to the Explanation:

 The term ‘Lok Adalat’ (b) 2 only means ‘People’s Court’ and is based on Gandhian principles. (c) Both 1 and 2  As per the Supreme Court, it is an (d) Neither 1 nor 2 old form of adjudicating system prevailed in ancient Answer: c India and its validity has not been taken away even in the modern days Explanation: too.  Prime Minister Narendra Modi will  The offences which are lay the foundation stone of Light House Projects (LHPs) under Global non-compoundable under any law fall Housing Technology Challenge-India outside the purview of the Lok Adalat. (GHTC-India) at six sites across six  The Lok Adalat shall have the same states. powers as are vested in a Civil  GHTC aims to fast-track the Court under the Code of Civil construction of affordable housing and Procedure (1908). meet the target of constructing 2 crore  There is no court fee and if court fee is houses by 2022. already paid the amount will be  GHTC focuses on identifying and refunded if the dispute is settled at mainstreaming proven demonstrable Lok Adalat technologies for lighthouse projects and spotting potential future technologies Source: The Hindu for incubation and acceleration support through ASHA (Affordable Sustainable Question 9 Housing Accelerators) — India. Consider the following statement with  They LHPs comprise about 1000 reference to Global Housing houses at each location along with Technology Challenge India (GHTC allied infrastructure facilities. India) initiative 1. The Light House Projects are being Source: The Hindu constructed at Indore (Madhya Pradesh),

Rajkot (), Chennai (Tamil Nadu), Ranchi (Jharkhand), Agartala (Tripura) and Question 10 Lucknow (Uttar Pradesh). Granting bail is the rule of 2. They comprise about 1000 houses at each a) Constitutional discretionary power of location along with allied infrastructure judiciary facilities. b) Legislative discretionary power of judiciary Which of the following statement is/are c) Legislative discretionary power of higher correct? judiciary only d) Constitutional discretionary power higher (a) 1 only of judiciary only

a) Buddhism Answer: b b) Jainism Explanation: c) Vaishnavism  The power to grant bail is a discretionary power vested in judges d) Shaivism and it is meant to be exercised liberally. Answer: b  The Code for Criminal Procedure act 1973, does not define bail, although the Explanation: terms bailable offence and non-bailable offence have been defined in section  Jainism is an ancient religion that is 2(a) of the Code. rooted in the philosophy that teaches the way to liberation and a path to  A Bailable offence is defined as an spiritual purity and enlightenment offence which is shown as bailable in through disciplined nonviolence to all the First Schedule of the Code or which living creatures. is made bailable by any other law, and  Anekantavada: Emphasizes that the non-bailable offence means any other ultimate truth and reality is complex, offence. and has multiple-aspects i.e. theory of  A person who is arrested for a 'bailable' plurality. It refers to the simultaneous offence may secure bail at the police acceptance of multiple, diverse, even station, while those who fail to secure contradictory viewpoints. police bail and those arrested for non-bailable offences have to secure Source: NIOS bail in court. Question 2  Sections 436 to 450 set out the provisions for the grant of bail and Which of the following is/are the bonds in criminal cases. consequences of the proclamation of the national emergency At present, the power to grant bail is exercised sparingly. Subordinate courts even routinely 1. Centre becomes entitled to give executive reject bail for specific offences like minor directions to a state on ‘any’ matter excise offences. 2. The parliament becomes empowered to Source: The Indian express make laws on any subject mentioned in the state list

FEB 5 3. Dissolution of the local bodies

Question 1 Select the correct answer using the code given With reference to the religious practices in below: India, the Anekantavada philosophy sect belongs to (a) 1 and 2 only

Considering the following statements (b) 1 and 3 only regarding foreign currency borrowing:

(c) 2 and 3 only 1. It will reduce crowding out effect.

(d) 1, 2and 3 2. It will free resources for domestic savings and production Answer: a Which of the statements given above is / are Explanation: correct?

A proclamation of Emergency has drastic and (a) 1 only wide-ranging effects on the political system. These consequences can be grouped into 3 (b) 2 only categories: (c) Both 1 and 2  Effects on the center-state relations: While a proclamation of Emergency is in force, (d) Neither 1 nor 2 the normal fabric of the Centre-State relations undergoes a basic change. this can Answer: c be studied under three heads:  Executive: Centre becomes entitled to Explanation: give executive directions to a state on ‘any’ matter  It is debt issued by the national  Legislative: The parliament becomes government in a foreign currency in empowered to make laws on any order Sovereign debt is usually created subject mentioned in the state list; the by borrowing government bonds president can issue ordinances on State It will help in following way: subjects also, if the parliament is not in session. The laws made on state subjects  Address the shallowness of bond by the parliament become inoperative market which exists in India as well as six months after the emergency has reduce crowding out effect. ceased to be in operation.  Frees up resources for domestic  Financial: the president can modify the savings and production constitutional distribution of revenues  Bring less expensive sourcing of between the center and thestates. resources and force financial discipline  It does not affect local bodies on governments

Source: Laxmikanth Source: Mrunal economy Question 3

Question 4

Consider the following statement regarding reduce photosynthesis and slow the Ozone pollution: plant growth.  Damages some types of materials, 1. Ozone in troposphere is called bad ozone especially objects made of rubber 2. It can increase photosynthesis and increase Source: Shankar IAS the plant growth

Which of the statements given above is / are correct? Question 5

(a) 1 only Consider the following statement regarding optoelectronic: (b) 2 only 1. Data transmission through photons

(c) Both 1 and 2 2. Converts electricity into photon signals

Which of the statements given above is / are (d) Neither 1 nor 2 correct?

Answer: a (a) 1 only

Explanation: (b) 2 only Generally, ozone is classified into two types: (c) Both 1 and 2  Good ozone: Found in stratosphere, it protects the Earth's surface from (d) Neither 1 nor 2 dangerous ultraviolet light.  Bad Ozone: Found in the Answer: c troposphere (also known as ground level ozone), it is man-made. The Explanation: released nitrogen oxide (NOx), carbon monoxide (CO) and volatile organic  Optoelectronics is the study and compounds (VOC), (NOx, CO, and application of electronic devices and VOCs are known as ozone precursors) systems that source, detect and control  Surface level ozone is also the third light. most potent greenhouse gas after  It is based on the quantum mechanical carbon dioxide and methane. effects of light on electronic materials,  Irritation to skin and the respiratory especially semiconductors. system, higher rates of pulmonary  It encompasses the design, manufacture disease. and study of electronic hardware  When sufficient ozone enters the devices that, as a result, converts leaves of a sensitive plant, it can electricity into photon signals for various purposes such as medical

equipment, telecommunications and are, between an oceanic and continental general science. plate, between two oceanic plates; and  Optical Fiber- where data is between two continental plates. transmitted in the form of light  Fold Mountains are often associated particles or photons that pulse through with continental crust. They are a fiber optic cable created at convergent plate boundaries, sometimes called Source: current affairs continental collision zones or compression zones.  Divergent Boundaries: Where a new Question 6 crust is generated as the plates pull away from each other. The sites Consider the following statement regarding where the plates move away from each plate boundaries: other are called spreading sites. The 1. New crust formation takes place in best-known example of divergent Transform boundaries boundaries is the Mid-Atlantic Ridge. At this, the American Plate(s) is/are 2. Fold Mountains are often associated separated from the Eurasian and convergent boundaries African Plates. 3. Crust destruction takes place in divergent  Transform Boundaries: Where the boundaries crust is neither produced nor destroyed as the plates slide Select the correct answer using the code given horizontally past each other. below.  Transform faults are the planes of separation generally perpendicular to (a) 1 and 2 only the mid oceanic ridges. As the (b) 2 only eruptions do not take all along the entire crest at the same time, there is a (c) 1 and 3 only differential movement of a portion of the plate away from the axis of the (d) 1, 2 and 3 earth. Also, the rotation of the earth has Answer: b its effect on the separated blocks of the plate portions. Explanation Source: NCERT There are three types of plate boundaries:

 Convergent Boundaries: Where the crust is destroyed as one plate dived Current affairs under another. The location where the Question 7 sinking of a plate occurs is called a subduction zone. There are three ways Global Risks Report published by in which convergence can occur. These

a) World Bank 5. Mizoram b) International Monetary Fund Select the correct answer using the code given below. c) World Economic Forum d) World Health Organization a) 1- 3-4-2-5

Answer: c b) 2-1-3-4-5

Explanation: c) 2-3-4-5-1

 According to the Global Risks Report, d) 1-3-4-5-2 2021 released virtually by World Economic Forum (WEF) recently, Answer: C infectious diseases topped the global risks chart, displacing climate change. Explanation  It analyses the risks from societal fractures - manifested through persistent and  Indian states that share borders with emerging risks to human health, rising Bangladesh are: Assam, West Bengal, unemployment, widening digital divides, Mizoram, Meghalaya and Tripura.The youth disillusionment and geopolitical State-wise length of the borders is as fragmentation. under:  Other reports of WEF: Global Competitiveness Report, Global Gender State Total Length (in Km) Gap Report, Global Social Mobility Index, West Bengal 2216.70 Global Competitiveness Report and Travel and Tourism Competitiveness Report etc. Assam 263.00 Meghalaya 443.00 Source; The Hindu Tripura 856.00 Question 8 Mizoram 318.00 India shares land border with Bangladesh, arrange the following states length wise in Source: The Hindu descending order:

1. Assam Question 9 2. West Bengal Consider the following statement regarding 3. Tripura Jal jeevan Mission:

4. Meghalaya 1. Greywater management is one of the components of the JJM.

2. The funding pattern between states and center is 40:60 and 100 percent from center for Question 10 Himalayan and north eastern states. Consider the following statements with Which of the statements given above is / are reference to Carnatic Music: correct? 1. Carnatic Music gives more importance to (a) 1 only Vocal than Instruments.

(b) 2 only 2. Tabla, Sarangi, Sitar and Santoor are musical instruments of Carnatic Music.

(c) Both 1 and 2 Which of the statements given above is / are correct? (d) Neither 1 nor 2 (a) 1 only Answer: a (b) 2 only Explanation: (c) Both 1 and 2  Jal Jeevan Mission (JJM) envisages

supply of 55 litres of water per person per day to every rural household (d) Neither 1 nor 2 through Functional Household Tap Connections (FHTC) by 2024. Answer: d  JJM focuses on integrated demand and supply-side management of water Explanation: at the local level.  Carnatic music owes its name to the  Funding Pattern: The fund sharing Sanskrit term Karnâtaka pattern between the Centre and Sangîtam which denotes “traditional” states is 90:10 for Himalayan and or “codified” music. North-Eastern States, 50:50 for other states, and 100% for Union  Composed of a system of Territories. Ragam (Raga) and Thalam (Tala), it  Every village will prepare a Village has a rich history and tradition. Action Plan (VAP) which will have  Carnatic Sangeet has developed in the three components: south Indian states of Tamil Nadu, , Andhra Pradesh and  Water source & its maintenance . These states are known for  Water supply and their strong presentation of Dravidian culture.  Greywater (domestic wastewater) management.  Carnatic Music was evolved mainly by Shyama Shastri, Tyagraja, Source: The Hindu

Muthuswamy Dikshitar, and Saint  It was established on 20 November Purandardas. 1916 in Victoria Memorial Hall  Carnatic Music gives more in Madras by Dr C. Natesa importance both instruments and Mudaliar and co-founded by T. M. vocal equally. Nair and P. Theagaraya Chetty as a  Veena, Mridangam and Mandolin are musical instruments of Carnatic result of a series of non-Brahmin music conferences and meetings in the  Tabla, Sarangi, Sitar and Santoor are presidency. musical instruments of Hindustani  The Justice Party was isolated in Music contemporary Indian politics by its many controversial activities. It Source: The Hindu opposed Brahmins in civil service and politics, and this anti-Brahmin attitude shaped many of its ideas and policies. FEB 6  It opposed Annie Besant and Question 1 her Home rule movement, because it believed home rule would benefit the Consider the following statement regarding Brahmins. Justice Party:  The party also campaigned against the non-cooperation movement in the 1. It opposed Annie Besant and her Home rule presidency. It was at odds with M. K. movement, because it believed home rule Gandhi, primarily due to his praise for would benefit the Brahmins Brahminism. Its mistrust of the 2. It supported the non-cooperation movement "Brahmin–dominated" Congress led it to adopt a hostile stance toward Which of the statements given above is / are the Indian independence movement. incorrect? Source: Spectrum (a) 1 only

(b) 2 only Question 2 (c) Both 1 and 2 Consider the following statement regarding (d) Neither 1 nor 2 Governor financial power:

Answer: b 1. Governor recommends for the demand for grants which otherwise cannot be given Explanation:

 The Justice Party, officially the South 2. Contingency Fund of State is under him and Indian Liberal Federation, was a he makes advances out that to meet unforeseen expenditure political party in the Madras Presidency of British India.

Which of the statements given above is / are Which of the following is/are correct? correct? a) 1 and 2 only (a) 1 only b) 1 and 3 only (b) 2 only c) 2 and 3 only (c) Both 1 and 2 d) 1, 2, and 3 only (d) Neither 1 nor 2 Answer: c Answer: c

Explanation: Explanation:

 A liquidity trap is a contradictory Financial Powers of the Governor economic situation in which interest The following are the financial powers and rates are very low and savings rates functions of the Governor: are high, rendering monetary policy ineffective.  He looks over the state budget being  It leads to a scenario where any laid in the state legislature additional money supply that is  His recommendation is a prerequisite generated in the economy get for the introduction of money bill in the channeled towards savings rather state legislature than investment thus rendering the  He recommends for the demand for economy to remain at same liquidity grants which otherwise cannot be level. given  In a liquidity trap, the demand for  Contingency Fund of State is under money is perfectly elastic. Increasing him and he makes advances out that the money supply doesn’t reduce to meet unforeseen expenditure interest rates and the impact of  State Finance Commission is increasing the money supply is constituted every five years by him. ineffective in boosting demand. (Read about the Finance Commission of India in the linked article.) Source: Ramesh Singh economy

Source: Laxmikanth Question 4 Question 3 Consider the following statements regarding Consider the following statement regarding Great Indian Bustard liquidity trap 1. It is endemic to Indian Sub-continent only 1. Interest rates will be increase 2. It has high population inDesert National 2. It will channel money supply towards Park Sanctuary (Rajasthan) savings rather than investment

3. The demand for money is perfectly elastic

3. The habitat where it is most often found is 1. Red Blood Cells count will increases arid and semi-arid grassland 2. Metabolic Rate will decrease 4. It is placed Critically Endangered under IUCN Red list. 3. Depth of respiration increases

Which of the following statement is correct? Which of the following code is/are correct? a) 1 and 3 only a) 1 and 2 only b) 1, 2 and 3 only b) 1 and 3 only c) 1 and 4 only c) 2 and 3 only d) All the above d) 1, 2, and 3 only

Answer: d Answer: b

Explanation: Explanation:

 Great Indian Bustard is endemic to • Within the first few hours of altitude Indian Sub-continent, found in central exposure, water loss also increases, India, western India and eastern which can result in dehydration. Pakistan. • Altitude can also increase your  Important Sites for the species are: metabolism while suppressing your Desert National Park Sanctuary appetite, meaning you’ll have to eat (Rajasthan), Naliya (Gujarat), Warora more than you feel like to maintain a () and Bellary (Karnataka) neutral energy balance.  The habitat where it is most often found is arid and semi-arid A number of changes take place in the body to grasslands, open country with thorn allow it to operate with decreased oxygen. scrub, tall grass interspersed with cultivation. It avoids irrigated areas. • The depth of respiration increases.  IUCN: Critically Endangered • Pressure in pulmonary arteries is (Threats: poaching, collisions with increased, "forcing" blood into portions power lines of the lung which are normally not used  It is considered as the flagship during sea level breathing. grassland species, representing the • The body produces more red blood health of the grassland ecology. cells to carry oxygen,  High population present in • The body produces more of a particular Rajasthan. enzyme that facilitates • The release of oxygen from Source: NIOS hemoglobin to the body tissues.

Question 5 Source: General Science

Which of the following is true in the process of acclimatization high latitude?

Question 6 whereas neutral soils (pH 5.5–7.5) provide conditions in which bacteria Which of the following factors responsible have a competitive advantage. for decomposition of the organic matter in the soil? • Temperature: Cool temperatures inhibit the decomposition and piling up 1. High moisture availability in the soil of the organic content in the soil takes place. Warm temperature makes the 2. Absence of oxygen in the soil microorganisms active and faster will be the decomposition 3. Warm temperature • Oxygen: It accelerates the process of decomposition. Since major organisms 4. pH of the soil depend on the oxygen for respiration and reproduction, they cannot multiply Select the correct answer using the code in hypoxic conditions i.e. lack of given below. enough oxygen levels a) 1 and 3 only Source: NCERT b) 1 and 2 only c) 3 and 4 only Current affairs d) 1, 2 and 4 only

Answer: c Question 7

Explanation: Consider the following statement regarding Pradhan Mantri Matru Vandana Yojana: • Moisture: Maximum decomposition occurs at the intermediate level of 1. Money provided directly in the account of moisture in soil. Soil gas exchange will Pregnant Women be limited in high moisture soil leading to low oxygen concentrations and 2. It implemented in all districts of India potentially anaerobic conditions. Also, low moisture content makes the lack of Which of the statements given above is / are water thus limiting the microbial correct? metabolism. • Both high and low moisture (dry) (a) 1 only conditions inhibit the decomposition process. (b) 2 only

• Soil pH can have a major effect on (c) Both 1 and 2 the rate of decomposition in soils and (d) Neither 1 nor 2 the composition of the decomposer communities in soils (10–12). Answer: c • Typically, acidic soils (pH 3.0–5.5) are dominated by fungal communities, Explanation:

Pradhan Mantri Matru Vandana Yojana Article 72 says that the President shall have the power to grant pardons, reprieves, respites or  Maternity Benefit Programme - a remissions of punishment or to suspend, remit scheme for worshipping the or commute the sentence of any person motherhood by providing maternity convicted of any offence. benefits.  Implemented in all districts of India  Pardon –A pardon completely absolves in accordance with the provision of the offender from all sentences and National Food Security Act (NFSA), punishment and disqualifications and 2013. places him in the same position as if he  Implemented using the platform of Anganwadi Services scheme of had never committed the offence. Umbrella ICDS.  Commutation– Commutation means  Rs.5000/- provided directly in the exchange of one thing for another. In account of Pregnant Women and simple words to replace the punishment Lactating Mothers - for the first living with less severe punishment. For child of the family. example for Rigorous  Provided in three instalments. imprisonment-simple imprisonment. Source: The Hindu  Reprieve– Reprieve means temporary suspension of death sentence. For example- pending a proceeding for pardon or commutation. Question 8  Respite – Respite means awarding a lesser punishment on some special Consider the following statement regarding grounds. For example- the Pregnancy pardoning power of President of India: of women offender. 1. The President exercises his power of  Remissions– Remission means the pardon independent of the Judiciary. reduction of the amount of sentence 2. Judicial review of the pardon powers of without changing its character, for President is prohibited example, a sentence of 1 year may be remitted to 6 months. Which of the statements given above is / are correct? This power of pardon shall be exercised by the President on the advice of Council of (a) 1 only Ministers. (b) 2 only Further, the constitution does not provide for (c) Both 1 and 2 any mechanism to question the legality of decisions of President or governors exercising (d) Neither 1 nor 2 mercy jurisdiction.

Answer: a But the SC in Epuru Sudhakar case has given a small window for judicial review of Explanation: the pardon powers of President and

governors for the purpose of ruling out any is called a hybrid between a company arbitrariness. and a partnership.  All limited liability partnership is The court has earlier held that court has governed under the limited liability retained the power of judicial review even on a matter which has been vested by the partnership act of 2008. The Constitution solely in the Executive. Corporate Affairs Ministry implements the Act. The President exercises his power of pardon independent of the Judiciary.  LLP will have more flexibility as compared to a company.

 LLP will have lesser compliance Source: The Hindu requirements as compared to a company.

Question 9  The management-ownership divide inherent in a company is not there in Consider the following statement regarding a limited liability partnership. Limited Liability Partnerships: Source: The Mint

1. The Ministry of Finance implements the Act.

2. The management-ownership divides Question 10 inherent in a company Consider the following statement regarding Which of the statements given above is / are Akash missile correct? 1. This is surface to air anti-aircraft missile

(a) 1 only 2. It is long range missile (b) 2 only Which of the statements given above is / are (c) Both 1 and 2 correct?

(d) Neither 1 nor 2 (a) 1 only

Answer: d (b) 2 only

Explanation: (c) Both 1 and 2

 LLP is an alternative corporate business (d) Neither 1 nor 2 form that gives the benefits of limited Answer: a liability of a company and the flexibility of a partnership. Hence LLP Explanation:

 Akash is a medium-range mobile Empire "Rajya", Rashtra", "Desha", surface-to-air missile defence system. "Mandala", "Prithvi" and "Avani". It was developed by the Defence  The Empire was divided into provinces Research and Development called as Bhukti, Bhoga and pradesha. Organisation (DRDO) and produced by Provinces further divided into "Vishayas" Bharat Electronics Limited. and came under the control of persons called "Vishaya Patis"."Vishaya" further  The missile is guided by phased array divided into "Nagaras" and "Nagares" fire control radar called ‘Rajendra’ were divided into village which is termed as Battery Level Radar (BLR) with a tracking range of about  Bhuktis was the name of Provinces in the 60 km. Gupta Empire. Uparikas was the name  The Akash-MK-1S is capable of of provincial governors. The king striking down enemy fighter jets and maintained a deep contact with the drones very effectively and accurately. provincial administration. By a group of  The Akash surface-to-air missile was officials designed to intercept enemy aircraft called Kumaramatyas and Ayuktas. and missiles from a distance of 18 to 30 km, medium-range missile. Source: NCERT

Source: current affairs Question 2

FEB 7 Other than the Fundamental Rights, which of the following parts of the Constitution of Daily MCQ India reflect/reflects the principle of gender Question 1 equality?

With reference to the administration of  Preamble Ancient India, which one of the following is  Directive Principles of State Policy the correct description of the term ‘Uparika  Eleventh Schedule a) Head of the village during satavahana period Which of the following is/ are correct? b) Provincial Governor during gupta empire a) 1 and 2 only c) Head of the village during Mauryan Empire b) 1 and 3 only d) Provincial governor under Sungas c) 2 and 3 only

Answer: b d) 1, 2, and 3 only

Explanation: Answer: d

 In Gupta Empire, the king was directed in Explanation: his administration by a community and group consisting of a chief minister and a  The principle of gender equality is Senapati. There were various names of enshrined in the Indian Constitution in

its Preamble, Fundamental Rights, Answer: b Fundamental Duties and Directive Principles. The Constitution not only Explanation: grants equality to women, but also  They are rupee-denominated bonds empowers the State to adopt measures i.e. the funds would be raised from of positive discrimination in favour of overseas market in Indian rupees. women.  Any corporate, body corporate and  Within the framework of a democratic Indian bank is eligible to issue Rupee polity, our laws, development policies, denominated bonds overseas. Plans and programmes have aimed at  The first Masala bond was issued in women’s advancement in different 2014 by International Finance spheres. Corporation (IFC) for the infrastructure  Not less than one-third (including the projects in India. number of seats reserved for women  Money raised through such bonds belonging to the Scheduled Castes and cannot be used for real estate activities the Scheduled Tribes) of the total other than for development of number of seats to be filled by direct integrated township or affordable election in every Panchayat to be housing projects reserved for women and such seats to  It also can’t be used for investing in be allotted by rotation to different capital markets. constituencies in a Panchayat (Article 243 D(3) ( Eleventh schedule) Source: Mrunal Economy

Source: Laxmikanth

Question 3 Question 4

Consider the following statement regarding Masala Bond Consider the following statement regarding 1. They are rupee-denominated bonds raised recent Forest Survey Report in domestic market 1. Mangroves cover In India has been 2. It can’t be used for investing in capital increased markets 2. Forest and Tree Cover at national level has Which of the statements given above is / are been decreased correct? Which of the statements given above is / are (a) 1 only correct?

(b) 2 only (a) 1 only

(c) Both 1 and 2 (b) 2 only

(d) Neither 1 nor 2 (c) Both 1 and 2

(d) Neither 1 nor 2 c) It is the condition where disease prevalent for small period Answer: a d) It is the condition where disease prevalence Explanation: in infant stage

 The Forest Survey of India released the Answer: a India State of Forest Report for the year 2019 Explanation  FSI undertakes biennial assessment of Orphan Diseases resembles: country’s forest resources, the results of which are presented as the India State  It is a health condition of low of Forest Report (ISFR prevalence, affecting a small number  The total forest cover of the country is of people, as compared to other 21.67% of the total geographic area of prevalent diseases in the general the country. The tree cover of the population. country is estimated as 2.89% of the  WHO defines rare disease as often geographical area. debilitating lifelong disease or disorder  There is an increase of 0.56% of condition with a prevalence of 1 or less, forest cover, 1.29% of tree cover and per 1000 persons. However, different 0.65% of forest and tree cover put countries have their own definitions together, at the national level as  These diseases are known as Orphan compared to the previous assessment Diseases as the market for its drug i.e. ISFR 2017 development is not economically  There has been a net increase of 54 viable. sq. km in the mangrove cover of the  They include genetic diseases, rare country as compared to 2017 cancers, infectious tropical diseases and assessment. degenerative diseases. 80% of rare  The mangrove cover in the country is diseases are genetic in origin and hence 4,975 sq km, which is 0.15% of the disproportionately impact children. country’s total geographical area.  Despite being less prevalent and individually rare, collectively they Source: Forest Survey Report affect between 6% and 8% of total population in any country

Question 5 Source: Science Report

Orphan Diseases resembles: a) It is a health condition of low prevalence, Question 6 affecting a small number of people They occupy about seven per cent of the earth's b) It is a health condition of high prevalence in land surface and habours more than half of the few communities world’s plants and animals. They are found

mostly near the equator. Leaves are broad and 1. He joined the Fort William College as give out excess water through teacher evapo-transpiration. Which of the following climatic types is described in the passage given 2. Opened the doors of the colleges and other above? educational institutions to the oppressed and the downtrodden (a) Mediterranean Climate 3. He wrote Barno-Porichoy book in Bengali (b) Tropical ever green fore literature

(c) Steppe Climate Which of the following statement is/ are correct? (d) Natal climate a) 1 and 2 only Answer: b b) 1 and 3 only Explanation: c) 3 only • The tropical evergreen forests usually occur in areas receiving more than 200 d) 1, 2 and 3 only cm of rainfall and having a temperature of 15 to 30 degrees Celsius. Answer: d • They occupy about seven per cent of Explanation: the earth's land surface and habours more than half of the world’s plants and  Ishwar Chandra Vidyasagar is one of animals. the pillars of Bengal • They are found mostly near the equator. renaissancecontinued the reforms • These forests are dense and movement started by Raja Rammohan multi-layered. They harbour many Roy. types of plants and animals.  In his book, ‘Barno-Porichoy’ • The trees are evergreen as there is no (Introduction to the letter), period of drought. They are mostly tall Vidyasagar refined the Bengali and hardwood type. language - made it accessible to the • Leaves are broad and give out excess common people. water through evapo-transpiration.  1841 - joined the Fort William College as a head of the Sanskrit Source: GC Leong department.  Initiated the concept of widow remarriage and raised concern for the Current affairs abolition of child-marriage and polygamy. Question 7  Opened the doors of the colleges and Consider the following statement regarding other educational institutions to the Ishwar Chandra Vidyasagar: oppressed and the downtrodden.

Source: The Hindu  It is done by detecting the presence of specific antibodies that are produced against the virus. ICMR is nodal body.

Question 8 Source:The Hindu

Consider the following statement regarding Indian Council Medical Research Question 9 1. It funded by the Government of India through Ministry of Health and Family Consider the following statement regarding Welfare 5G technology

2. It is nodal body to conduct serological 1. The latency period is high compare to 4G survey 2. 5G uses microwave frequency to increase Which of the statements given above is / are speed and capacity correct? Which of the following statement is correct? (a) 1 only Which of the statements given above is / are (b) 2 only correct?

(c) Both 1 and 2 (a) 1 only

(d) Neither 1 nor 2 (b) 2 only

Answer: c (c) Both 1 and 2

Explanation: (d) Neither 1 nor 2

 The Indian Council of Medical Answer: d Research (ICMR), the apex body in India for the formulation, Explanation: coordination and promotion  5G has lower latency i.e. the delay before of biomedical research, is one of the a transfer of data begins following an oldest and largest medical research instruction. Latency for 5G is predicted to bodies in the world. be below 10 milliseconds and in best cases  The ICMR is funded by around 1 millisecond. Where 4G has higher the Government of India through the latency as compared to 5G. Latency for 4G Department of Health is around 20-30 milliseconds. Research, Ministry of Health and  5G uses utilize much higher radio Family Welfare frequencies of 28 ghz.  A serological survey is conducted to  5G uses millimeter wave spectrum which assess the prevalence of a disease in a enables more devices to be used within the population.

same geographic area supporting around radio, television, satellite, telephone one million per square kilometer. and the Internet.  India has been an active member of Source:The Hindu the ITU since 1869 and has been a regular member of the ITU Council since 1952.

Question 10  In November 2018, India was elected as a Member of the ITU Council for Consider the following statement regarding another 4-year term (2019-2022). International Telecommunications Union: Source:The Hindu 1. It is an agency of the United Nations (UN)

2. India is a permanent member FEB 8 Which of the statements given above is / are Question 1 correct? With reference to the history of India, (a) 1 only consider the following statement regarding (b) 2 only Akbar

(c) Both 1 and 2 1. Synthesis of Hindu- Muslim art tradition

(d) Neither 1 nor 2 2. City of Fatehpur Sikri was built

Answer: c 3. Extensive use of red stone in architecture

Explanation: Which of the following is/are correct?

International Telecommunications Union: a) 1 and 2 only

 It is an agency of the United Nations b) 1 and 3 only (UN) whose purpose is to coordinate telecommunication operations and c) 2 and 3 only services throughout the world. d) 1, 2, and 3 only  Originally founded in 1865, as the International Telegraph Union, the ITU Answer: d is the oldest existing international organization. Explanation:  Headquarters are in Geneva, • The history of Mughal architecture Switzerland. really starts with Akbar. Just as Akbar  The ITU sets and publishes regulations built up an extensive empire on the and standards relevant to electronic goodwill of the , in the same communication and broadcasting way he utilized local talent and took technologies of all kinds including inspiration from Indian architecture.

Important buildings built during Akbar’s Explanation: time include the following:  Sixth schedule established Autonomous • Red Fort at Agra. District Councils (ADC) in four northeastern states, namely Assam, • City of Fatehpur Sikri Meghalaya, Tripura and Mizoram. These ADCs envisage protecting and preserving tribal culture. • Lahore fort,  The rationale behind the creation of ADCs is the belief that relationship to • Tomb at Sikandra. the land is the basis of tribal or indigenous identity. The culture and Main features of Akbar’s buildings are: identity of indigenous people can be preserved by ensuring their control over  Synthesis of Hindu- Muslim art land and natural resources, as these tradition factors to a large extent determine the lifestyle and culture of the indigenous  Extensive use of red stone people. Source: Laxmikanth  Construction of buildings for civilian purposes. Question 3 Consider the following pairs Source: NCERT Index

Institution Question 2 1. Global Livability Index-----Economist Consider the following statement regarding Intelligence Unit Sixth Schedule 2. Ease of Living Index ------Ministry of 1. The rationale behindto protect tribal or Housing and Urban Affairs indigenous identity. 3. Municipal Performance Index ------Ministry 2. Autonomous Development Council in tribal of Housing and Urban Affairs region across India Which of the following pair correctly Which of the statements given above is / are matched? correct? a) 1 and 2 only (a) 1 only b) 1 and 3 only (b) 2 only

(c) Both 1 and 2 c) 2 and 3 only

(d) Neither 1 nor 2 d) 1, 2, and 3 only

Answer: a Answer: d

Explanation: Consider the following statement regarding Bonn convention: Global Livability Index 1. India has officially taken over its Presidency  Released by- Economist Intelligence for the next three years, till 2023 Unit  It assesses 140 cities and ranks them 2. Gandhinagar Declaration adopted to protect according to their performance in over migratory species 30 qualitative and quantitative factors Which of the statements given above is / are across five broad categories - Stability, correct? Healthcare, Culture and Environment, Education, and Infrastructure (a) 1 only

Ease of Living Index (b) 2 only

 Released by- Ministry of Housing and (c) Both 1 and 2 Urban Affairs (d) Neither 1 nor 2  It is aimed at providing a holistic view of Indian cities -services provided by Answer: c local bodies, effectiveness of Explanation: administration, outcomes generated through these services in terms of  India has officially taken over its livability within cities and citizen Presidency for the next three years, perception of these outcomes till 2023.  CMS COP13 was the largest ever in the

history of the Convention. The CoP is Municipal Performance Index the decision making organ of the CMS.  Adoption of Gandhinagar  Released by- Ministry of Housing and Declaration- which calls for migratory Urban Affairs species and the concept of ‘ecological  It will assess performance of connectivity’ to be integrated and municipalities on five enablers service, prioritized in the Post-2020 Global Finance, Planning, Technology and Biodiversity Framework, which is Governance which include 20 expected to be adopted at the UN indicators such as Education, Health, Biodiversity Conference in October this Water & Wastewater, SWM & year Sanitation, Registration & Permits, Infrastructure, Revenue Management Source: current affairs etc. Question 5

Source: Current Affairs Consider the following statement regarding Question 4 Astrosat

1. It is first Indian multi-wavelength space 3. Uvalas observatory 4. Stalagmite 2. It functions like NASA's Hubble Space Telescope. Which of the landforms given above are Depositional landforms formed by the Which of the statements given above is / are action of groundwater? correct? a) 1 and 4 only (a) 1 only b) 2 and 3 only (b) 2 only c) 1, 2 and 4 only (c) Both 1 and 2 d) 1, 2, 3 and 4 (d) Neither 1 nor 2 Answer: c Answer: c • Physical or mechanical removal of Explanation: materials by moving groundwater is  Astrosat, the first Indian insignificant in developing landforms multi-wavelength space observatory • But in rocks like limestone’s or which completed four years in orbit dolomites rich in calcium carbonate, in Sept 2019. the surface water as well as  It enables the simultaneous groundwater through the chemical multi-wavelength observations of various astronomical objects with a process of solution and precipitation single satellite deposition develop varieties of  It is seen as a smaller version of landforms NASA's Hubble Space Telescope.  It has 5 payloads which includes Depositional landforms:  Ultraviolet Imaging Telescope (UVIT) Stalactites: They hang as icicles from  Large Area X-ray Proportional Counter (LAXPC) the roofs and are of different diameters.  Soft X-ray Telescope (SXT) Normally they are broad at their bases  Cadmium Zinc Telluride Imager and taper towards the free ends. (CZTI) Stalagmite: They rise up from the floor  Scanning Sky Monitor (SSM) of the caves. In fact, stalagmites form due to dripping water from the surface Source: Current affairs or through the thin pipe, of the Question 6 stalactite, immediately below it. Pillars: The stalagmite and stalactites Consider the following landforms: eventually fuse to give rise to columns 1. Stalactites and pillars of different diameter Erosional landform 2. Pillars

 Swallow holes: also known as  A bad bank is technically an asset Sinkholes are funnel-shaped shallow reconstruction company that buys bad depressions formed on the surface of loans(NPAs) from the commercial banks at a discount and tries to recover limestones through solution the money from the defaulter by  Doline: refers to the collapse sinks providing a systematic solution over a  Uvalas: When sinkholes and dolines period of time. join together because of slumping of materials along their margins or due to  The idea of a bad bank seeks to reduce roof collapse of caves, long, narrow to the NPAs in the banking sector and then wide trenches called valley sinks or revive lending and credit growth. Uvalas form.  Moreover, the creation of a bad bank  Lapies: are the ridges formed due to allows the segregation of a bank’s differential solution activity along good assets from its bad assets. This parallel to sub-parallel joints. The lapie allows investors to assess its financial field may eventually turn into health with greater clarity and for banks somewhat smooth limestone pavements to grow financially.

Source: GC Leong  Bad bank by way of absorbing NPAs, will ease the provisioning requirement Current affairs by the banks and help them to get on with business as usual. Question 7

Which of the following statement is correct regarding Bad Bank? Question 8

1. The creation of a bad bank allows the Consider the following statement regarding segregation of a bank’s good assets from its Unsaturated Fats bad assets. 1. Unsaturated fats are important part of a healthy diet- reduce the risk of heart disease 2. It is set up to buy non-performing or bad loans from banks. 2. omega-3 fats, found in fish

Which of the statements given above is / are 3. Unsaturated fats that have been processed correct? will increases the levels of ‘bad’ cholesterol

(a) 1 only Which of the following pair correctly matched? (b) 2 only a) 1 and 2 only (c) Both 1 and 2 b) 1 and 3 only (d) Neither 1 nor 2 c) 2 and 3 only Answer: c d) 1, 2, and 3 only Explanation:

Answer: d  Published annually since 2005, the CCPI is an independent monitoring tool Explanation: for tracking countries’ climate  WHO called for global elimination of protection performance. o In 2017, the Trans fat by 2023. methodology of the climate action  Saturated fats: Increased risk of heart network. disease and high blood cholesterol  India ranked 10th in the latest edition of levels. Found in animal-based products the Climate Change Performance Index (dairy foods, meat); (CCPI).  Unsaturated fats: Important part of a healthy diet- reduces the risk of heart disease and lower cholesterol levels when they replace saturated Question 10 fats.  Polyunsaturated fats: omega-3 fats, Consider the following statement regarding found in fish and omega-6 fats, found ammonium Pollution in safflower and soybean oil.  Unsaturated fats that have been 1. The largest source of NH3 emissions is processed: Increases the levels of agriculture, ‘bad’ cholesterol and decreases the levels of ‘good’ cholesterol. Found in 2. Ammonia (NH3) is a color less highly many packaged foods and also in butter reactive and soluble alkaline gas. and some margarines Which of the statements given above is / are correct?

Question 9 (a) 1 only

Climate Change Performance Index (b) 2 only released by (c) Both 1 and 2 a) Germanwatch (d) Neither 1 nor 2 b) World economic Forum Answer: c c) United Nation Environment Programme Explanation: d) World Bank  Ammonia (NH3) is a colorless highly Answer: a reactive and soluble alkaline gas.  It is prominent constituent of the Explanation: nitrogen cycle that adversely affects ecosystems at higher concentrations. Climate Change Performance Index (CCPI)  The largest source of NH3 emissions is agriculture, including animal  The Index is published by husbandry and NH3-based fertilizer Germanwatch, New Climate Institute applications. and the Climate Action Network.

 Other sources of NH3 include industrial the teachings of the Buddha could be processes, vehicular emissions, spread further. volatilization from soils and oceans,  It was held at Rajagriha. decomposition of organic waste, forest fires, animal and human waste, nitrogen fixation processes. Second Buddhist Council

 Conducted under the patronage of King FEB 9 Kalasoka of Sisunaga dynasty.  It was held in 383 BC, i.e., a hundred years after the Buddha’s death. Question 1  It was held at Vaishali

Consider the following pairs: Third Buddhist Council Buddhist councils Dynasty • Conducted under the patronage of 1. First council ------Sisunaga dynasty. Emperor Ashoka of Maurya dynasty. 2. Second council----- Haryanka dynasty. • It was held in 250 BC at Pataliputra. • The council was presided over by 3. Third council----- Maurya dynasty. Mogaliputta Tissa. 4. Fourth council-----Kushan dynasty.

Which of the following is/are correctly Fourth Buddhist Council matched?  Conducted under the patronage of King a) 1 and 2 only Kanishka of Kushan dynasty.  It was held in the 1st century AD at b) 3 and 4 only Kundalvana in Kashmir. c) 2 and 3only  Vasumitra and Ashvaghosha presided over this council. d) 1, 2, and 4 only Source: NCERT Answer: b Question 2 Explanation: In the context of India, which one of the following is the characteristic appropriate First Buddhist Council for secularism?’

 Conducted under the patronage of King a) The state cannot give any financial Ajatasatru of Haryanka dynasty. support for religious activities  The council was established in order to arrive at a consensus on how b) “State” and the “religion” have their own separate spheres

c) Religion is relegated entirely to the private Answer: d sphere and has no place in public life whatsoever. Explanation: d) Neither in law nor in practices any wall of It refers to countries that have experienced separation between religion and the State rapid growth and thus quickly reached middle-income status (with Gross National Answer: d Product per capita between $1,000 & $12,000), but then failed to overcome that income range Explanation: to further catch up to the developed countries and achieve high-income status  In India, neither in law nor in practice any 'wall of separation' Source: Ramesh Singh between religion and the State exists.  In India, both state and religion can, and often do, interact and intervene in each other's affairs within the legally Question 4 prescribed and judicially settled parameters. Consider the following statement regarding Atal Bhujal Yojana (Atal Jal) scheme  In Indian secularism, state shall interfere in religion so as to remove 1. It is a central sponsored scheme evils in it.  As per the western model of 2. The aim of the scheme is to improve ground secularism, the “State” and the water management through community “religion” have their own separate participation spheres and neither the state nor the religion shall intervene in each other’s Which of the statements given above is / are affairs. correct?

Source: Laxmikanth (a) 1 only

Question 3 (b) 2 only

In economics Middle Income Trap means (c) Both 1 and 2 a) Countries rapidly reached middle-income (d) Neither 1 nor 2 status with poor social indicators Answer: b b) Countries rapidly reached to developed status then fall back to middle-income status Explanation: c) Countries rapidly reached middle-income  It is a Central Sector Scheme, to status with high dependency on global factors improve ground water management through community participation in d) Countries rapidly reached middle-income identified priority areas. status but fail to achieve high-income status.

 It covers seven States- Gujarat, Haryana, d) 1, 2, 3 and 4 only Karnataka, Madhya Pradesh, Maharashtra, Rajasthan and Uttar Pradesh. Answer: d  Scheme components: ATAL JAL has two major components Explanation:  Institutional Strengthening and Capacity Building component including improving  Edge computing enables data to be monitoring networks, capacity building, analyzed, processed, and transferred at strengthening of Water User Associations, the edge of a network. Meaning, the allocating more funds for Panchayats and data is analyzed locally, closer to where making Panchayat-level plans etc it is stored, in real-time without latency.  Incentive Component for incentivizing the  It offers high speed, reduced latency & States for achievements in improved groundwater management practices like better reliability which allows for data dissemination, preparation of water quicker data processing and content security plans, water budgeting, etc. delivery.  It offers better security by distributing Source: Current Affairs processing, storage, and applications across a wide range of devices and data

centers, which makes it difficult for any Question 5 single disruption to take down the network  It offers a far less expensive route to With the present state of development, Edge scalability and versatility, allowing computing can effectively do which of the companies to expand their computing following? capacity through a combination of internet of things devices and edge data 1. It enables data to be analyzed, processed, centers. and transferred locally Source: World economic forum website 2. It has high speed, reduced latency & better reliability

3. It offers better security against cyber threats Question 6

4. It offers a far less expensive route to The correct sequence of the eastward scalability and versatility flowing rivers of the peninsular India from north to south is Which of the statements given above is / are correct? a) Mahanadi—Godavari—Krishna--Valgai-- a) 1 and 2 only b) Godavari—Mahanadi---Krishna—Kaveri—Va b) 1 and 4 only lgai c) 1, 2 and 3only

c) Mahanadi—Godavari---Krishna--- Source: NCERT Kaveri—Valgai d) Mahanadi—Godvari—Krishna--- Kaveri—Valgai Current affairs

Answer: c Question 7 Explanation: Consider the following cloud brust East Flowing Peninsular Rivers

1. Mahanadi River 1. It isa localized weather phenomena

2. 2. Generally cloudbursts are associated with thunderstorms. 3. Which of the statements given above is / are 4. Kaveri (Cauvery) River correct?

5. Pennar River (a) 1 only

6. Subarnarekha River (b) 2 only 7. Brahamani River (c) Both 1 and 2 8. Sarada River (d) Neither 1 nor 2 9. Ponnaiyar River Answer: c 10. Vaigai River Explanation:  The cloudburst is a localised weather phenomena representing highly concentrated rainfall over a small area lasting for few hours. This leads to flash floods/ landslides, house collapse, dislocation of traffic and human casualties on large scale.  Meteorologists say the rain from a cloudburst is usually of the shower type with a fall rate equal to or greater than 100 mm (4.94 inches) per hour.  Generally cloudbursts are associated

with thunderstorms. The air currents

rushing upwards in a rainstorm hold up the Ganges, in the Himalayas in the Indian state of Uttarakhand. a large amount of water. If these  The Dhauliganga arises on the Raikana currents suddenly cease, the entire glacier at an altitude of 4700 m. amount of water descends on to a small  Joined by Rishiganga river at Raini in area with catastrophic force all of a Uttarakhand. sudden and causes mass destruction  Merges with the Alaknanda at  They occur most often in desert and Vishnuprayag near Joshimath in Uttarakhand. mountainous regions, and in interior  Flows through the Nanda Devi National regions of continental landmasses. Park.

Source: The Hindu Source: The Indian express

Question 9 Question 8 Consider the following statement regarding Consider the following statement KIRAN helpline

1. The Dhauliganga arises on the Raikana 1. It is toll free for mental rehabilitation glacier helpline

2. The Dhauliganga finally flows into the 2. It comes under Ministry of Health welfare Alaknanda at Vishnuprayag Which of the statements given above is / are 3. It flows through Nanda Devi National Park. correct?

Which of the statements given above is / are (a) 1 only correct? (b) 2 only (a) 1 only (c) Both 1 and 2 (b) 1 and 2 only (d) Neither 1 nor 2 (c) 1 and 3 only Answer: a (d) All the above Explanation: Answer: d Explanation KIRAN helpline (1800-599-0019)

 The Dhauliganga is is a left tributary of  24.X.7 toll free mental rehabilitation the Alaknanda, the left headwater of helpline.

 Launched by Dept of Empowerment of Persons with Disabilities, Ministry of Social Justice.  13 languages - Assamese, Tamil, Marathi, Odia, Telugu, Malayalam, Gujarati, Punjabi, Kannada, Bengali, Urdu, Hindi and English.  Aim - provide relief and support to persons with mental illness.

Source: The Hindu

Question 10 Source: The Hindu

The following country not opens mouth with Black sea? FEB 10

 Ukraine  Russia  Romania Question 1  Moldova With reference to the history of India, the Answer: d terms Kurinji, Mullai , Marudam Neydal and Palai denote Explanation: a) Classification of urban land during Pallavas  Six countries border with the Black Sea, including Ukraine to the b) Division of land during Sangam age north, Russia and Georgia to the east, Turkey to the south, c) Measurement Units of land during Cholas and Bulgaria and Romania to the west.  Moldova is land locked country d) Different types of rituals practiced during medieval period

Answer: b Explanation:

 Tolkappiyam refers to the Five-fold division of lands – Kurinji (hilly tracks), Mullai (pastoral), Marudam (agricultural), Neydal (coastal) and Palai (desert).  Tolkappiyam also refers to four castes namely arasar(Ruling Class), anthanar, vanigar(carried on trade and

commerce) and  In the case of transfer of a Chief Justice, vellalar(Agriculturists). only the views of one or more  Ancient primitive tribes like Thodas, knowledgeable Supreme Court Judges Irulas, Nagas and Vedars lived in this need to be taken into account. period.  The views on the proposed transfer of a Judge or a Chief Justice of the High Court should be expressed in Source: NCERT (TN) writing and should be considered by Question 2 the CJI and the four senior most Judges of the Supreme Court. Consider the following statements:  The proposal once referred to the Government, the Union Minister of 1. The views on the proposed transfer of a Law, Justice and Company Affairs Judge or a Chief Justice of the High would submit a recommendation to the Court should be expressed in writing and Prime Minister who will then advise the should be considered by the CJI and the four President as to the transfer of the Judge senior most Judges of the Supreme Court. concerned. 2. The Collegium of judges is the Supreme  After the President approves the Court’s invention after three judge case and is transfer, the notification will be gazetted mentioned in the Constitution. and the judge remains transferred.  The Collegium of judges is the Which of the following statement is/ are Supreme Court’s invention after incorrect? three judge case and is not mentioned (a) 1 only in the Constitution.  In case of selection and transfer of HC (b) 2 only judges, the collegium consists of CJI, 2 senior most judges of the SC and CJ of (c) Both 1 and 2 the concerned High Courts. All (d) Neither 1 nor 2 such transfers must be made in the public interest, that is, “for the Answer: b betterment of the administration of justice”. Explanation: Source: Laxmikanth  Article 222 of the Constitution makes provision for the transfer of a Judge Question 3 (including Chief Justice) from one High Court to any other High Court. Consider the following statement  The initiation of the proposal for the transfer of a Judge should be made by 1. Foreign Direct Investment inflow in India is the Chief Justice of India (CJI). high in manufacturing sector  The opinion of the CJI “is 2. Foreign portfolio Investment are more determinative”. sustainable investment compare to FDI

Which of the statements given above is / are 1. It captures the external costs of greenhouse correct? gas (GHG) emissions

(a) 1 only 2. Emissions Trading Systems and carbon tax are major carbon pricing mechanisms (b) 2 only Which of the statements given above is / are (c) Both 1 and 2 incorrect?

(d) Neither 1 nor 2 (a) 1 only Answer: d (b) 2 only Explanation: (c) Both 1 and 2

• A foreign direct investment (FDI) is an (d) Neither 1 nor 2 investment made by a firm or individual in one country into business Answer: d interests located in another country. • Foreign portfolio investment (FPI) Explanation: instead refers to investments made in securities and other financial assets  Carbon pricing is an instrument that issued in another country. captures the external costs of greenhouse gas (GHG) emissions - the  FDI is important source to boost costs of emissions that the public pays economy for driving the investment in for, such as damage to crops, health enhancement of productivity, skills and care costs from heat waves and technology development in the country. droughts, and loss of property from FPI are volatile compare to FDI. flooding and sea level rise - and ties  During FY20, total FDI equity inflows them to their sources through a price, were US$49.98 billion as compared to usually in the form of a price on the US$44.37 billion during FY19. carbon dioxide (CO2) emitted.  The bulk of FDI equity flow is in the  Emissions Trading Systems (ETS): non-manufacturing sector leading to ETS - also referred to as a a reduction in the share of cap-and-trade system - caps the total manufacturing in the FDI flows. level of GHG emissions and allows those industries with low emissions to Source: Economic survey 2020-2021 sell their extra allowances to larger emitters  A carbon tax directly sets a price on Question 4 carbon by defining a tax rate on GHG emissions or - more commonly - on the Consider the following statement regarding carbon content of fossil fuels. It is Carbon Pricing different from an ETS in that the

emission reduction outcome of a carbon Question 6 tax is not pre-defined. Consider the following statements regarding Source: Current affairs the Earthquake:

Question 5 1. Earthquakes waves due to sudden movement in the upper mantle. Consider the following statement 2. The point where the energy is released is 1. Quantum Technology functions at the scale called the focus or the hypocentre of an of elementary particle earthquake. 2. Quantum computing functions based on bits Which of the statements given above is/are Which of the statements given above is / are correct? incorrect? a) 1 only (a) 1 only b) 2 only

(b) 2 only c) Both 1 and 2

(c) Both 1 and 2 d) Neither 1 nor 2 (d) Neither 1 nor 2 Answer: c Answer: b Explanation: Explanation  An earthquake is shaking or trembling • Quantum technology seeks to harness of the earth’s surface, caused by laws of quantum physics, which the seismic waves or earthquake describe the behaviour of matter and waves that are generated due to a sudden movement (sudden release of energy at the atomic and subatomic energy) in the level. earth’s crust (shallow-focus • This is unlike classical physics, in earthquakes) or upper mantle (some which an object can exist in one place shallow-focus and all intermediate and at one time. E.g. classical computers deep-focus earthquakes). operate using binary physical state,  The point where the energy is meaning its operations are based on one released is called the focus or the hypocentre of an earthquake. of two positions (1 or 0).  The point on the surface directly above • Quantum Computers encode the focus is called epicentre (first information as quantum bits, or surface point to experience the qubits, which can exist in earthquake waves). superposition  A line connecting all points on the surface where the intensity is the same Source: Science report is called an isoseismic line.

Source: NCERT Development Goals (SDG3: Good health and well-being).

Source: The Hindu Current affairs Question 7 Question 8 Consider the following statement Consider the following statement regarding regarding Chipko movement: Ayushman Bharat scheme: 1. It is violent ecological movement 1. It provide coverage of up to Rs 5 lakh per 2. Aimed at protecting trees and forests slated family per year for government-backed logging. 2. It is based on Universal Health Coverage Which of the statements given above is/are correct? Which of the statements given above is/are correct? a) 1 only a) 1 only b) 2 only b) 2 only c) Both 1 and 2 c) Both 1 and 2 d) Neither 1 nor 2 d) Neither 1 nor 2 Answer: b

Answer: c Explanation:

Explanation:  A nonviolent social and ecological movement organised by rural villagers,  The government-sponsored health particularly women, in the Himalayan insurance scheme will provide free coverage of up to Rs 5 lakh per region of Uttarakhand in the 1970s. family per year at any government or  Aimed at protecting trees and forests even empanelled private hospitals all slated for government-backed over India for secondary and tertiary logging. medical care facilities.  Quickly spread throughout the Indian  The scheme, if implemented properly Himalayas. could enhance access to health care  Embracing the trees to impede the including early detection and loggers. treatment services by a large section of society who otherwise could not Source: The Hindu afford them.

 NHPS could help country move towards universal health coverage Question 9 and equitable access to healthcare which is one of the UN Sustainable

Consider the following statement regarding  Advocates government for appointment of Union Public Service Commission (UPSC) meritorious and potential young men and women in administrative services after 1. Number of such members and the terms and selecting them through All India conditions of their service is to be determined competitive examinations. by president of India  Assists government in framing and 2. The Chairman is not authorized to take any operating schemes of joint recruitment for office of profit post retirement any service for which candidates possessing special qualification. 3. The UPSC recommends government  Advises the President on all matters regarding reservation quota relating to methods of recruitment to civil Which of the following statement is/are services and for civil posts. correct?  List out principles to be followed in making appointments to civil services and a) 1 only posts and in making promotions and transfers and on the suitability of b) 1nd 2 only candidates. c) 1 and 3 only  Look at all disciplinary matters affecting a person serving under the Government of d) All the above India or the Government of a State in a civil capacity, including memorials or Answer: b petitions relating to such matter. Explanation: Source: The Hindu Union Public Service Commission (UPSC) Question 10  A constitutional authority that conducts With reference to SURYA KIRAN exercise, examinations pertaining to Civil Services, consider the following statements: Engineering Services, Defence Services, and Medical Services w.r.to the Central  The exercise is an annual event which Govt. is conducted alternatively in Sri  Article 315 to 323 of Indian Constitution Lanka and India.  Article 315: There shall be a Public Service Commission for the Union and a Public  The 13th (previous) edition of the Service Commission for each State. exercise was conducted in Uttarakhand, India.  Number of such members and the terms and conditions of their service is to be which of the following statement is/ are determined by the President of India. correct?  The Chairman not authorized to take

any office of profit under the central or any of the state governments after (a) 1 only his/her retirement from service.

(b) 2 only Explanation:

(c) Both 1 and 2  Rowlatt Satyagraha officially known as the Anarchical and Revolutionary (d) Neither 1 nor 2 Crimes Act, 1919. Answer: b  The act was passed as per recommendations of the Rowlatt Explanation: Committee chaired by a judge, Sir  The joint military exercise ‘SURYA Sidney Rowlatt, after whom the act is KIRAN is between India and named. Nepal. The exercise is an annual  This act authorized the British event which is conducted alternatively government to arrest anybody suspected in Nepal and India. of terrorist activities.  The 13th (previous) edition of the  The act was passed despite unanimous exercise was conducted opposition from the Indian members of in Uttarakhand, India. the council, all of whom resigned in Source: Hindistantimes protest. These included Mohammed Ali Jinnah, Madan Mohan Malviya and

Mazhar Ul Haq. FEB 11  Two popular Congress leaders Satya Pal and Saifuddin Kitchlew were Question 1 arrested. Consider the following statement regarding  Lord Chelmsford was India’s Rowlatt Satyagraha. Viceroy during the protest.

1. Two popular Congress leaders Satya Pal and Source: Spectrum Saifuddin Kitchlew were arrested during Rowlatt Satyagraha.

2. Lord Chelmsford was India’s Viceroy Question 2 during Rowlatt Satyagraha. Consider the following statements regarding rd Which of the following statement is/are 73 amendment of the constitution correct? 1. It is Magna Carta of local democracy in (a) 1 only India.

(b) 2 only 2. One-third of the total number of seats to be reserved for women. (c) Both 1 and 2 3. Scheduled areas listed under the 5th (d) Neither 1 nor 2 Schedule are exempted from 73rd amendment of the constitution. Answer: c

Which of the following statement is/are  It is Magna Carta of local correct? democracy in India a) 1 only Source: Laxmikanth b) 1nd 2 only c) 1 and 3 only Question 3 d) 1, 2 and 3 If the RBI decides to adopt Accommodative Answer: d Monetary Policy which of the following would it not do? Explanation:

 These amendments added two new 1. Increase the money supply via quantitative parts to the Constitution, namely, easing added Part IX titled “The 2. Increase the bank and repo rate Panchayats” (added by 73rd 3. Cut the cash reserve ratio (CRR) Amendment) and Part IXA titled “The Municipalities” (added by Select the correct answer using the code given 74th Amendment). below:

 Basic units of democratic a) 1 and 2 only system-Gram Sabhas (villages) and Ward Committees b) 2 only (Municipalities) comprising all the adult members registered as voters. c) 1 and 3 only

 Three-tier system of panchayats at d) 1,2 and 3 village, intermediate block/taluk/mandal and district levels except in States with population is Answer: b below 20 lakhs (Article 243B). Explanation:  One-third of the total number of seats to be reserved for women.  Accommodative monetary policy, also known as loose credit or easy monetary  One third of the seats reserved for policy, occurs when a central bank SCs and STs also reserved for attempts to expand the overall money women. supply to boost the economy when  Scheduled areas listed under the V growth is slowing (as measured by Schedule in the states of Andhra GDP). The policy is implemented to Pradesh, Bihar, Gujarat, Himachal allow the money supply to rise in line Pradesh, Madhya Pradesh, with national income and the demand Maharashtra, Orissa and for money. Rajasthan.

 The policy can also allow the money  Among various source of IAP, the use supply to increase or increase the of traditional biomass and cookstoves money supply via quantitative is one of the key causes. Incomplete easing. combustion of biomass produces a  Accommodative monetary policy is range of toxic gases including PM, triggered to encourage more spending methane, carbon monoxide, volatile from consumers and businesses by organic compounds etc. making money less expensive to  To save people from the ill effects of borrow through the lowering of IAP, the central government has short-term interest rates. launched two schemes  The SLR and CRR will cut to a. Unnat Chulha Abhiyan was increase money supply launched to promote improved biomass cookstove Source: Mrunal economy b. Pradhan Mantri Ujjwala Yojana was launched for LPG connections to Below Poverty Question 4 Line families. c. Retrofit of Air-conditioning to Consider the following statement improve Indoor Air Quality for 1. Pradhan Mantri Ujjwala Yojana will Safety and Efficiency (RAISE) decrease indoor pollution which is a joint initiative of Energy Efficiency Services 2. Common indoor pollutants are carbon Limited (EESL) and USAID. dioxide, ozone, formaldehyde

Which of the following statement is/are Source: current affairs correct?

(a) 1 only Question 5 (b) 2 only Consider the following statement regarding (c) Both 1 and 2 Nuclear Magnetic Resonance Spectroscopy (NMR) (d) Neither 1 nor 2 1. Radio frequency waves used in NMR

2. NMR used in Food adulteration test Answer: c Which of the following statement is/are Explanation: correct?  Indoor Air Pollution (IAP) is the (a) 1 only degradation of indoor air quality by harmful chemicals and other materials. (b) 2 only

(c) Both 1 and 2 a) 1 and 4 only

(d) Neither 1 nor 2 b) 2 and 3 only

c) 1, 2 and 4 only

Answer: c d) 1, 2, 3 and 4

Explanation: Answer: c

 Nuclear magnetic resonance (NMR) Explanation: spectroscopy is the study of molecules Currents in the Indian Ocean include the by recording the interaction of following: radiofrequency (Rf) electromagnetic radiations with the nuclei of molecules placed in a strong magnetic field.  The North East Monsoon Drift  Like all spectroscopies, NMR uses an  The South West Monsoon Drift electromagnetic radiation component  North Equatorial Current (Warm) (radio frequency waves) to facilitate transitions between rates of nuclear  South Equatorial Current (Warm) energy (resonance). Some chemists use  Somali Current (Cold) NMR to determine small molecules by structure.  Mozambique Current (Warm)  NMR spectroscopy is used to  Madagascar Current (Warm) determine structure of proteins, amino acid profile, carotenoids, organic acids,  Agulhas Current (Warm) lipid fractions, and the mobility of the  West Australian Current (Cold) water in foods. It used to test food adulteration test. The Canary Current is a wind-driven surface current that is part of the North Source: current affairs Atlantic Gyre Question 6

Which of the following is Indian Ocean Source: GC Leong current?

1. Agulhas Current Current affairs 2. North Equatorial Current Question 7 3. Canary Current Consider the following statement regarding 4. West Australian Current Lokpal

Select the correct answer using code given 1. Lokpal is executive body below.

2. The members are appointed by the officers and officials of Central president on the recommendation of a Government. Selection Committee.  Jurisdiction of the Lokpal included the 3. The Lokpal does not have jurisdiction over Prime Minister except on allegations Ministers and MPs in the matter of anything of corruption relating to international said in Parliament or a vote given there. relations, security, the public order, atomic energy and space. Which of the following statement is/are correct?  The Lokpal does not have a) 1 and 2 only jurisdiction over Ministers and MPs in the matter of anything said in b) 1nd 3 only Parliament or a vote given there. c) 2 and 3 only Source: The Hindu d) All the above Question 8 Answer: c Consider the following pairs: Explanation: Passes state  Established under the Lokpal and 1. Lipu lekh Uttarakhand Lokayuktas Act, 2013. It is statutory 2. Nathu la Himachal Pradesh body.  The main function: to inquire and 3. Shipiki la Sikkim investigate into allegations of the 4. Chanag la Ladakh corruption against public functionaries falling within the ambit Which of the pairs given above is/are of the Act correctly matched? a) 1 and 2 only  The members are appointed by the president on the recommendation b) 1 and 4 only of a Selection Committee. c) 2 and 3 only  The selection committee is composed d) 1, 3 and 4 only of the Prime Minister who is the Chairperson; Speaker of Lok Sabha, Leader of Opposition in Lok Answer: b Sabha, Chief Justice of India or a Judge nominated by him/her and One Explanation: eminent jurist.  Chaang la : It is connects Ladakh  Jurisdiction of Lokpal includes Prime with Tibet Minister, Ministers, members of Parliament, Groups A, B, C and D

 Shipiki la is located in Kinnaur poor human rights records to be district in the state of Himachal members Pradesh  The UNHRC works closely with the  The pass is one of India's border Office of the High Commissioner for posts for trade with Tibet along with Human Rights (OHCHR). Nathu La in east Sikkim, and  The Council’s Membership is based Lipulekh in Uttarakhand. on equitable geographical Source: The Hindu distribution  Members of the Council serve for three years and are not eligible for immediate Question 9 re-election after serving two Consider the following statement consecutive terms. regarding UN Human Rights Council:  India has been elected to the UNHRC 1. It is an inter-governmental body within the for three years beginning January 1, United Nations system 2019.  India had previously been elected to 2. The Council’s Membership is based on the UNHRC for the 2011-2014 and equitable geographical distribution. 2014-2017 terms.

3. India has been elected first time in 2019 Source: The Hindu

Which of the following statement is/are correct? Question 10 a) 1 and 2 only Nanomicelles often seen in the news, is b) 1 and 3 only a) It is disinfectant used in water purification c) 2 and 3 only b) It is used in recombinant DNA technology d) 1,2 and 3 for efficient gene insertion c) It can be used for effective drug delivery Answer: a to treat various cancers Explanation: d) It is used in purification of air pollutant  The Human Rights Council is an inter-governmental body within the Answer: c United Nations system.  It replaced the former United Nations Explanation: Commission on Human Rights • A nanomicelle that can be used (UNCHR) that had been strongly for effective drug delivery to treat criticized for allowing countries with

various cancers including breast, List of the Ancient Indian Scholars and their colon and lung cancer. Patrons • Nanomicelles are globe-like Scholars Patrons structures with a hydrophilic outer shell and a hydrophobic interior. This Hemchandra Kumarapala dual property makes them a perfect Chalukya Of carrier for delivering drug molecules Anhilwad Nagarjuna Kanishka Source: The Mint Amarsimha Chandragupta Vikramaditya

Ravikirti Pulakesin FEB 12 Question 1 Vakapatiraja Bhavabhuti Yasovarman Of With reference to the scholars of ancient Kannauj India consider the following statements: Harisena Samudragupta 1. Allasani Peddanna was scholar during Krishnadeva Raya of Vijayanagar Empire Rajasekhara Mahipala and Mahendrapala Of 2. Banabhatta was associated to pulakeshin ii Pratiharas of Chaulkya Somadeva Prithviraj III 3. Dandin was associated with Narasimhavarma of Pallava Chandbardai Prithviraj Chauhan

Banabhatta Harsha Which of the statements given above is/are correct? Dandin Narasimhavarma Pallava (a) 1 and 2 only Bharavi Simha Vishnu Pallava (b) 1 and 3 only

Gunadhyaya Hala Of (c) 3 only Satavahana

(d) 1, 2 and 3 Jinasena Amoghavarsha Of Rashtrakuta Answer: b

Explanation:

Jaideva Lakshmanasena Of opportunities in the government and Bengal insures against discrimination by the State in matters of employment on the basis of caste, religion, etc. This right Bilhana Vikramaditya VI also includes the abolition of titles as Chalukya Of well as untouchability. Kalyani

Lakshmidhara Govindchandra of Gahadavalas Of Source: Laxmikanth Kannauj

Kalhana Harsha of Kashmir Question 3

In the context of the Indian economy, a Allasani Peddanna Krishnadeva Raya non-marketable security in public debt includes which of the following?

1. Intermediate treasury bills issued to state Source: NIOS governments

2. Special securities issued to national Small Savings Fund Question 2 3. Government –securities Which one of the following categories of constitution ensures equal employment Select the correct answer using the code given opportunities? below: a) A Directive Principle of state policy [a) 1 only b) Fundamental rights b) 1 and 2 only c) The Preamble of the Constitution c) 3 only d) Seventh schedule d) 1, 2 and 3

Answer: b Answer: b

Explanation: Explanation:

Fundamental Rights under Right to Equality  Public debt is the total liabilities of the (Articles 14 – 18) central government contracted against  Right to equality guarantees equal rights the Consolidated Fund of India. for everyone, irrespective of religion,  It is further classified into internal & gender, caste, race or place of birth. It external debt. ensures equal employment

 Internal debt is categorized into and which have not yet been altered in marketable and non-marketable any way by living organisms. securities.  Secondary succession: succession starts  Marketable government securities in an environment which has already include G-securities and T-Bills issued been altered by a period of occupancy through auction. by living organisms.  Non-marketable securities include intermediate treasury bills issued to Source: NIOS state governments, special securities issued to national Small Savings Fund among others. Question 5

Source: NIOS Multipotency is best described as the ability

of

Question 4 a) Virus to replicate inside the body of its host.

Which of the following is appropriate meaning of sere in ecological succession? b) Stem cells divide and produce any type of body cells a) A community in a climax stage of ecological succession c) Single cell to divide and produce all of the differentiated cells in an organism. b) The process of replacement of abiotic community with biotic communities d) Cells that can differentiate the limited c) It is the sequence of communities that number of cells successively change in a given environment starting from pioneer community. Answer: d d) A Pioneer community in ecological succession Explanation:

Answer: c Explanation:  It is the sequence of communities that successively change in a given environment starting from pioneer community.

It can be distinguished:

 Primary succession: it begins in • Multipotent stem cells are cells that environments that lack organic matter have the capacity to self-renew by

dividing and to develop into multiple  Cocos plate: Between Central America and specialised cell types present in a Pacific plate specific tissue or organ.  Nazca plate: Between South America and • Most adult Pacific plate stem cells are multipotent stem cells e.g. Bone Marrow  Arabian plate: Mostly the Saudi Arabian • Unlike totopotency cell , multipotent landmass cells differentiate in limited scale.  Philippine plate: Between the Asiatic and Pacific plate Source: NCERT  Caroline plate: Between the Philippine and Indian plate (North of New Guinea) Question 6  Fuji plate: North-east of Australia Which of the following are minor tectonic India-Australia-New Zealand plate is major plates? plate. 1. Cocos plate Source: NCERT 2. Arabian plate Current affairs 3. India-Australia-New Zealand plate Question 7 4. Nazca plate Consider the following statements regarding Select the correct answer using the code given the Unlawful Activities (Prevention) Act: below. 1. Under Unlawful Activities (Prevention) Act (a) 1 and 2 only deals with charges against Indian citizens only

(b) 2, 3 and 4 only 2. Empowers the government to designate individuals as terrorists (c) 2 and 4 only 3. The National investigation Agency has the (d) 1, 2 and 4 only power attachment of property when the case is being investigated by the agency

Answer: d Which of the statements given above is/are correct? Explanation: a) 1 only  A tectonic plate (also called lithospheric plate) is a massive, irregularly-shaped slab b) 1 and 2 only of solid rock, generally composed of both c) 2 and 3 only continental and oceanic lithosphere. d) All the above Some important minor plates are listed below:

Answer: c  “China Plus One” business strategy - Aimed at diversification of Explanation: investments to the Association of 6. Empowers the government to designate Southeast Asian Nations (ASEAN), individuals as terrorists. India and Bangladesh 7. Empowers the Director-General of  Following the COVID-19 pandemic, National Investigation Agency (NIA) to numerous Indian companies have grant approval of seizure or attachment of adopted strategy to find alternative property when the case is being supply chains like china’s plus one investigated by the agency. strategy. 8. Under UAPA, both Indian and foreign Source: The Hindu nationals can be charged - even if the crime is committed outside India.

Source: The Hindu Question 9

Consider the statement regarding forest diversion land in India Question 8 1. Aurnchal Pradesh is the maximum diversion China plus one” strategy relates “ of forest land in last three decades in Indian a) “China plus One” business strategy - Aimed states. at diversification of investments to the 2. The diversion of forest land is major Association of Southeast Asian Nations contributor for deforestation. (ASEAN), India and Bangladesh. Which of the following statement is/are b) China political policy towards Taiwan correct? c) “China plus One” business strategy - Aimed (a) 1 only at diversification of investments to the African nation (b) 2 only d) Chinas political policy towards Hong Kong (c) Both 1 and 2

(d) Neither 1 nor 2

Answer: a

Explanation: Answer: c

 China plus One, also known simply Explanation: as Plus One, is the business strategy to avoid investing only in China and Forest diversion land diversify business into other countries.  Aurnachal Pradesh (first)  Madhya Pradesh (second)

 Telangana (Third) Facial recognition software (FRS) work in  Uttarakhand (fourth) following way  Major contributor of deforestation is 5. A picture of your face is diversion of forest land captured from a photo or video. Source: The Hindu 6. FRS reads the geometry of your face like the distance between your eyes and the distance from forehead to chin - identifies Question 10 facial landmarks - facial With reference to Facial Recognition signature. Technology, consider the following 7. Facial signature is compared statements? to a database of known faces. 8. Determination is made – face 1. A facial recognition software match faces print may match that of an with 100% accuracy without any chances of image in a facial recognition ‘false positive’ or ‘false negative’ results. system database.

2. As of now, neither Central Government, nor There are currently 16 different facial any State Governments in India use this recognition tracking (FRT) systems in active technology. utilization by various Central and State Which of the above statements is/are correct? governments across India for surveillance, security or authentication of identity (a) 1 only Source: The Hindustan times (b) 2 only

(c) Both 1 and 2 FEB 13 (d) Neither 1 nor 2 Question 1

Consider the following events in the history Answer: d of India:

Explanation: 1. C. Rajagopalachari Formula

Facial Recognition: 2. Viceroy Wavell organised the Simla Conference to discuss Wavell Plan  A way of recognizing a human face through technology. 3. The Royal Indian Navy (RIN) Mutiny  Uses biometrics to map facial features.  Compares the information with a 4. Mountbatten Plan for the partition of India database of known faces to find a Arrange the correct chronological order using match. code given below:

a) 2-1-4-3 Which of the following statement is/are correct? b) 1-2-3-4 a) 1 only c) 2-4-1-3 b) 2 only d) 3-4-1-2 c) Both 1 and 2

d) Neither 1 nor 2 Answer: b

Explanation: Answer: a The correct chronological order starting from the earliest time: Explanation:

 1944 - Gandhi met Jinnah to discuss  Republic and Ideals of liberty, equality the C. Rajagopalachari Formula and fraternity in the Preamble-----France  1945 - Viceroy Wavell organized the  Ideals of justice (social, economic and Shimla Conference to discuss Wavell political) in the Preamble---Soviet Plan Union (USSR) (now, Russia)  1946 -The Royal Indian Navy Mutiny Source: Laxmikanth by ratings( non-commissioned officers and sailors) against the Britishin Bombay (Talwar) Question 3

 l947- Mountbatten Plan for the Decline in gross domestic product (GDP), partition of India relatively high unemployment and accompanied by rising prices is

a) Deflation Source: Spectrum

b) Inflation Question 2 c) Stagflation Consider the following statement regarding Preamble d) Hyperinflation

1. Ideals of liberty, equality and fraternity in the Preamble are inspired from French revolution Answer: c 2. Ideals of justice (social, economic and political) in the Preamble are inspired from Explanation: Ireland

• Stagflation refers to an economy that is Cultural Organization experiencing a simultaneous increase in (UNESCO) for representative parts inflation and stagnation of economic of natural and cultural landscapes output. extending over large areas of • Stagflation is characterized by terrestrial or coastal/marine slow economic growth and relatively ecosystems or a combination of high unemploymentor both. economic stagnationwhich is at the  Biosphere Reserves tries to balance same time accompanied by rising economic and social development and prices (i.e. inflation). maintenance of associated cultural • Stagflation can also be alternatively values along with the preservation of defined as a period of inflation nature. combined with a decline in gross  Biosphere Reserves are thus special domestic product (GDP). environments for both people and nature and are living examples of how human beings and nature can co-exist Source: NCERT while respecting each other’s’ needs.

Source: Down to earth

Question 4 Question 5 Consider the following statement: Consider the following statement regarding Saguna technique: 1. Biosphere Reserve includes only parts of natural landscapes 1. It is a unique new method of cultivation of rice. 2. Biosphere Reserves designated by United 2. No ploughing, puddling and hoeing is to be Nations Educational, Scientific and Cultural done to control weeds. Organization (UNESCO) Which of the following statement is/are Which of the following statement is/are correct? correct? a) 1 only a) 1 only b) 2 only b) 2 only c) Both 1 and 2 c) Both 1 and 2 d) Neither 1 nor 2 d) Neither 1 nor 2

Answer: b Answer: c Explanation:  Biosphere Reserve (BR) is an Explanation: international designation by United Nations Educational, Scientific and

 It is a unique new method of cultivation Current Affairs of rice and related rotation crops

without ploughing, puddling and transplanting (rice) on permanent raised Question 7 beds. Consider the following statement  It is azerotill, Conservation Agriculture regarding Spinal muscular atrophy (CA) type of cultivation method evolved at SagunaBaug, District 1. A genetic disorder characterized by Raigad, Maharashtra. weakness and wasting in muscles used for  No ploughing, puddling and hoeing is movement. to be done to control weeds. Weeds are 2. It Caused by a loss of specialized nerve to be controlled with weedicides and cells. manual labor.  This system will get the crop ready for Which of the following statement is/are harvesting 8 to 10 days earlier. correct?

Source: Current affairs a) 1 only b) 2 only Question 6 c) Both 1 and 2 Which of the following UNESCO Biosphere Reserve passes through Rishi d) Neither 1 nor 2 Ganga? a) Cold Desert b) Nanda Devi National Park Answer: c c) Khangchendzonga, Sikkim Explanation: d) Similipal  A genetic disorder characterized by

weakness and wasting (atrophy) in muscles Answer: b used for movement (skeletal muscles).  Caused by a loss of specialized nerve cells Explanation: - motor neurons which controls muscle  The Nanda Devi National Park was movement. inscribed a World Heritage  Many types of spinal muscular atrophy - Site by UNESCO in 1988. differ in age of onset and severity of  It was later expanded and renamed muscle weakness. as Nanda Devi and Valley of Flowers National Parks in 2005. Source: The Hindu  It drained by the Rishi Ganga through the Rishi Ganga Gorge. Source: NIOS Question 8 Consider the following stat 1. Crypto currency is legal tender in India.

2. The crypto currency regulated through b) United Nation Environment Programme legal frame work in India. c) International Centre for Integrated Mountain Which of the following statement is/are Development correct? d) Intergovernmental Panel on Climate Change a) 1 only b) 2 only Answer: c c) Both 1 and 2 Explanation: d) Neither 1 nor 2 Hindu Kush Himalaya Assessment Report (2019):

 Released by the International Answer: d Centre for Integrated Mountain Explanation: Development (ICIMOD)  One-third of the Hindu Kush Crypto currency: Himalaya’s glaciers would melt by 2100 and potentially destabilise the  Digital money - no physical coin or river regime in Asia. bill.  Will happen even if, Paris Agreement  Stored in digital wallet → online or commitments are fulfilled by all the computer or hardware.  Transferred online without an countries in the region intermediary. Source: The Hindu  Used for quick payments and to avoid transaction fees.  Investments done in the hope of rise in values. Question 10  Crypto currencies are not legal tender in India. Consider the following statements regarding  Lack of a legal framework by Accessible India campaign: regulatory bodies leading complication. 1. A nationwide flagship campaign of the Ministry of Social Justice and Empowerment. Source: The Hindu 2. The aim of the campaign tomake a barrier free and conducive environment for PWDs Question 9 (Persons with Disabilities)

Hindu Kush Himalaya Assessment Report Which of the following statement is/are published by: correct? a) World Meteorological Organization a) 1 only

b) 2 only 2. Early Vedic society was pastoralist and semi-nomadic in nature whereas in Later Vedic c) Both 1 and 2 period Society became centered on agriculture in general d) Neither 1 nor 2 Which of the following statement is/are

incorrect?

Answer: c a) 1 only Explanation b) 2 only Accessible India Campaign: c) Both 1 and 2  A nationwide flagship campaign of the Ministry of Social Justice and d) Neither 1 nor 2 Empowerment. Answer: d  To make a barrier free and conducive environment for Persons with Disabilities all over the country. Explanation: It has three components Early Vedic period  Built Environment Accessibility 9. Women were allowed a greater degree of  Transport accessibility component freedom in this period.  Accessibility of Information and Communication System 10. They were allowed to participate in the political process of the time to a certain Source: The Hindu extent i.e. female philosophers like Gargi, Maitreyi, Lopamudra

11. Early Vedic society was pastoralist and FEB 14 semi-nomadic in nature. 12. The caste system was flexible and based on profession rather than birth.

Question 1 Later Vedic period

With reference to the difference between the 13. Women were restricted from their culture of early Vedic Aryans and Later participation in society by being relegated Vedic Aryan, consider the following to subordinate and docile roles. statements: 14. Society became more settled in nature. It became centred around agriculture in 1. Women participation in Sabhas was seen in general. early Vedic period whereas in Later Vedic 15. The caste system became more rigid in this period women were restricted from their period with birth being the main criteria. participation in Sabhas Source: NCERT

 Any dispute related to his election is taken up by Supreme Court. SC’s Question 2 decision is final.  After the election of President is With reference to the election of the declared null and void, the acts done by President of India, consider the following the President in his office remain valid statements even after his removal.

1. Elections are based on secret ballot system Source: Laxmikanth of voting.

2. Any dispute related to his election is taken Question 3 up by Supreme Court, its decision is final. Consider the following statement: Which of the following statement is/are correct? 1. The Phillips curve measures the degree of income equality in a population. a) 1 only 2. Lorenz Curve measures the relation b) 2 only between unemployment and inflation. c) Both 1 and 2 3. Kuznets Curve measures the relation between inequality and economic growth. d) Neither 1 nor 2

Which of the following is/are correct?

Answer: c a) 1 and 2 only

Explanation: b) 1 and 3 only

There is no direct election for the Indian c) 3 only President. An electoral college elects him. The Electoral College responsible for President’s d) 1, 2 and 3 elections comprises elected members of:

 Lok Sabha and Rajya Sabha Answer: c  Legislative Assemblies of the states (Legislative Councils have no role) Explanation:  Legislative Assemblies of the Union Territories of Delhi and Puducherry  The Gini Coefficient, which is derived from the Lorenz Curve, can Method: be used as an indicator of economic development in a country. The Gini  Proportional Representation with Coefficient measures the degree of means of a single transferable vote. income equality in a population. The  It is a secret ballot system of voting. Gini Coefficient can vary from 0

(perfect equality) to 1 (perfect that combines Hydroponics with Aqu inequality). aculture.  The Phillips curve is an economic • With Aquaponics both fish and concept, stating that inflation and plants can grow in one unemployment have a stable and integrated ecosystem. inverse relationship. • The fish waste provides an organic  Kuznets curve used to demonstrate the hypothesis that economic growth food source for the plants, which in turn naturally filter the water for the initially leads to greater inequality, followed later by the reduction of fish, creating a balanced ecosystem. inequality. • The third participant i.e. microbes or nitrifying bacteria converts the Source: NCERT ammonia from the fish waste into nitrates which plants need to grow

Question 4 Source: Down to earth Consider the following statement:

1. With Aquaponics both fish and plants can Question 5 grow in one integrated ecosystem. Consider the following statement regarding 2. Aquaponics is an ecologically sustainable Tuberculosis modelthat combines Hydroponics with Aquac ulture 1. It is non-communicable bacterial disease.

Which of the following statement is/are 2. India’s TB burden is the highest in the correct? world. a) 1 only Which of the following statement is/are b) 2 only correct? c) Both 1 and 2 a) 1 only d) Neither 1 nor 2 b) 2 only

c) Both 1 and 2 Answer: c d) Neither 1 nor 2 Explanation:

• Hydroponics is the soilless growing of plants, where soil is replaced with Answer: b water. Aquaculture is the raising of Explanation: fish. • Aquaponics is an ecologically sustainable model

 It is communicable disease (through chain of islands similar in origin) in Bay air) caused by bacteria of Bengal and Lakshadweep islands in (Mycobacterium tuberculosis) that Arabian Sea.  Andaman and Nicobar Islands were most often affect the lungs (pulmonary formed due to collision between Indian TB) sometimes also affects other Plate and Burma Minor Plate [part of organs (extra pulmonary TB). Eurasian Plate][Similar to formation of  India’s TB burden is the highest in Himalayas]. the world, followed by Indonesia and  Andaman and Nicobar Islands are China. 26.9 per cent of the global TB southward extension of Arakan Yoma burden in 2018 was from India. In range [Myanmar]  Arakan Yoma in itself is an extension of 2017, the figure was 27 per cent. Purvanchal Hills. Source:current affairs

Current Affairs Question 6 Question 7 Inthe context of the Andaman and Nicobar Islands, consider the following statement: Consider the following statement

1. Andaman and Nicobar Islands were formed 1. Cruise missiles follow a trajectory to deliver due to collision between Indian Plate and one or more warheads on a predetermined Burma Minor Plate. target.

2. Andaman and Nicobar Islands are southward 2. Ballistic missiles is a guided missile (target extension of Arakan Yoma range. has to be pre-set) used against terrestrial targets. Which of the following statement is/are correct? Which of the following statement is/are correct? a) 1 only a) 1 only b) 2 only b) 2 only c) Both 1 and 2 c) Both 1 and 2 d) Neither 1 nor 2 d) Neither 1 nor 2

Answer: c Answer: d Explanation: Explanation:  The major islands groups of India are Andaman and Nicobar Archipelago (A Ballistic missile:

 A ballistic missile follows a ballistic  Internet & Mobile Association of trajectory to deliver one or India (IAMAI) is a not-for-profit more warheads on a predetermined target. industry body registered under the  A ballistic trajectory is the path of an Societies Act, 1896. object that is launched but has no active propulsion during its actual flight (these  IAMAI is a specialized industry body weapons are guided only during relatively in India representing the interests of brief periods of flight). online and mobile value added services industry.

Source: The Hindu Cruise missile:

 A cruise missile is a guided missile (target has to be pre-set) used against terrestrial Question 9 targets. In Ancient art and culture Tholpavakoothu  It remains in the atmosphere throughout its flight. is  It flies the major portion of its flight path a) Ancient Drama at approximately constant speed.  Cruise missiles are designed to deliver a b) Shadow puppetry large warhead over long distances with high precision. c) Ancient dance  Modern cruise missiles are capable of travelling at supersonic or high subsonic d) Folk paintings speeds, are self-navigating, and are able to fly on a non-ballistic, extremely low-altitude trajectory. Answer: b Source: The Hindu Explanation:

Tholpavakoothu:

Question 8  Shadow puppetry - a ritual art - The Internet & Mobile Association of India popular in the northern districts of (IAMAI) is Kerala - Palakkad and Malappuram.  In Malayalam, ‘Thol’ means leather, a) Nonprofit industry ‘pava’ means doll or puppet, and ‘koothu’ means play. b) Private organization c) Government organization Question 10 d) Non-government Organization Consider the following pairs. Answer: a 1. Periyar Tiger Reserve– Kerala Explanation:

2. Indravati Tiger Reserve –Telangana Explanation:

3. Mulla Periyar Dam– Tamil Nadu  The term “Satvahana” originated from the Prakrit which means ” driven by Which of the above pair(s) is/are correctly seven” which is an implication of the matched? Sun God’s chariot that is driven by seven horses as per the Hindu a) 1 only mythology.  The first king of the Satavahana dynasty b) 1 and 2 only was Simuka. c) 3 only  The Satvahanas had a deep rooted d) 2 and 3 only control over the Indian Sea coast and consequently, a firm control over the

connected trade. Answer: a  The trade was boosted during the Explanation: period. It gave strong grounds to both the internal as well as external trade.  Mulla Periyar Dam: Kerala, Idukki District.  The trade with the other countries, as  Periyar Tiger Reserve– Kerala such the external trade was carried out  Indravati Tiger Reserve—Maharashtra through some famous sea ports as the dynasty was majorly spread across the Deccan and costal region of India, that Source: The Hindu being present day Karnatka, Telangana, FEB 15 Andhra Pradesh, and Gujarat.

Question 1  These famous ports were Broach, Sopara and Kalyan

With reference to the period of the Source: Tamil Nadu NCERT Satavahana Dynasty in ancient India the towns Broach, Sopara and Kalyan

(a) Inscription sites Question 2

(b) Headquarters of provinces Consider the following statement: 1. Adjournment is an end to a session (c) Ports handling foreign trade 2. A prorogation is the termination of sitting of (d) Buddhist sites the House

Answer: c Which of the following statement is/are incorrect?

a) 1 only The unemployment is that does not affect aggregate economic output. It occurs when b) 2 only productivity is low and too many workers are filling too few jobs. It can refer to any part of c) Both 1 and 2 the population that is not employed at full capacity. Which type of unemployment is being d) Neither 1 nor 2 discussed here?

a) Cyclical Unemployment Answer: c b) Frictional Unemployment Explanation: Adjournment: c) Structural Unemployment

d) Disguised Unemployment  Adjournment terminates the sitting of the House which meets again at Answer: d the time appointed for the next sitting. Explanation:  The postponement may be for a  Disguised unemployment exists specified time such as hours, days or frequently in developing countries weeks. whose large populations create a  If the meeting is terminated surplus in the labor force. without any definite time/ date fixed  It can be characterized by low for the next meeting, it is productivity and frequently called Adjournment sine die. accompanies informal labor markets and agricultural labor markets, Prorogation: which can absorb substantial quantities

of labor.  Prorogation is the end of a session.  Disguised, or hidden, unemployment A prorogation puts an end to a can refer to any segment of the session. population not employed at full  The time between the Prorogation capacity, but it is often not counted in and reassembly is called Recess. official unemployment statistics within  Prorogation is the end of session the national economy. and not the dissolution of the  This can include those working well house (in case of Lok Sabha, as below their capabilities, those whose Rajya Sabha does not dissolve). positions provide little overall value in terms of productivity, or any group that Source: Laxmikanth is not currently looking for work but is able to perform work of value.

Question 3 Source: NCERT Question 4

Consider the following statements: grasshoppers, and rabbits are all herbivores. 1. Some species of mammals lay eggs

2. All reptiles lay eggs Source: NCERT

3. All wild animals are carnivores

Which of the statement given above is/are Question 5 correct? Consider the following statement: a) 1 only 1. A meteor is a meteoroid that enters Earth's b) 1 and 3 only atmosphere c) 1 and 2 only 2. Meteoroids are objects in space that range in size from dust grains to small asteroids. d) 1,2 and 3

Which of the following statement is/are Answer: a correct?

Explanation: a) 1 only  The laying egg mammals are monotremes, marsupials, and the b) 2 only largest group, placental c) Both 1 and 2 mammals. Monotremes are mammals that lay eggs. d) Neither 1 nor 2  Theonly monotremes that are alive today are the spiny anteater, or echidna, and Answer: c the platypus. Explanation:  Reptiles can be divided into three major  A meteor is a meteoroid that enters groups: crocodiles, lepidosaurs (snakes Earth's atmosphere. and lizards), and turtles. Of the three, live birth is only seen in lepidosaurs. Even  Meteoroids are objects in space that among lepidosaurs, most lay eggs that range in size from dust grains to small hatch into young, but there are a asteroids few lizards and snakes that give  Like meteorites, meteors are objects birth to live young. that enter Earth's atmosphere from  Animals that eat only plants are called space. But meteors—which are herbivores. Wild animals like Deer, typically pieces of comet dust no larger

than a grain of rice—burn up before  Most of the plateau comprises the basin of reaching the ground. the Chambal river which flows in a rift valley.  The term “meteorite” refers only to those bodies that survive the trip  It is a rolling plateau with rounded hills through the atmosphere and reach composed of sandstone. Thick forests grow Earth's surface. here.

Source: NCERT  To the north are the ravines or badlands of the Chambal river and Satpura ranges, to the

south is the Deccan plateau, and to the west Question 6 are Aravalli ranges. Consider the following statements in the Source: NCERT context of Central Highlands:

1. It covers the major portion of the Malwa Current affairs plateau.

2. It is a rolling plateau with rounded hills Question 7 composed of sandstone Consider the following statement regarding Kyasanur Forest Disease: 3. Chambal river basin forms the majority part of the region. 1. A viral haemorrhagic fever endemic in Karnataka Which of the statement given above is/are correct? 2. Rodents, shrews, and monkeys are common a) 1 only hosts b) 1 and 3 only Which of the following statement is/are c) 1 and 2 only correct? d) 1,2 and 3 a) 1 only

b) 2 only Answer: d

Explanation: c) Both 1 and 2

 Central Highlands: also called the Madhya d) Neither 1 nor 2 Bharat Plateau.  It covers the major portion of the Malwa plateau. Answer: c

Explanation: b) 2 only  It is caused by Kyasanur Forest disease Virus (KFDV), a member of c) Both 1 and 2 the virus family Flaviviridae. d) Neither 1 nor 2  It was first identified in 1957 in a sick monkey from the Kyasanur Forest in Karnataka. Since then, between Answer: c 400-500 humans cases per year have been reported. Explanation:  KFD is endemic to the Indian state  The term ‘Lok Adalat’ of Karnataka. means ‘People’s Court’ and is based on Gandhian principles.  Rodents, shrews, and monkeys are  In view of its growing popularity common hosts for KFDV after being over time, it was given statutory bitten by infected Hard ticks status under the Legal Services (Haemaphysalis Spinigera). KFDV Authorities Act, 1987. The Act can cause epizootics (outbreak of makes the provisions relating to the disease in animals) with high fatality organisation and functioning of the in primates. Lok Adalats.  Transmission: To humans, it may  The State/District Legal Services occur after a tick bite or contact with Authority or the Supreme Court/High an infected animal (a sick or recently Court/Taluk Legal Services dead monkey). Committee may organise Lok Adalats at such intervals and places and for  Rodents, shrews, and monkeys are exercising such jurisdiction and for common hosts. such areas as it thinks fit. Source: The Hindu  It is one of the components of the Alternative Dispute Resolution (ADR) system and Question 8 delivers informal, cheap and expeditious justice to the common Consider the following statement Lok people. Adalat:  The first Lok Adalat camp was organised in Gujarat in 1982 as 1. It is an informal alternative dispute redressal a voluntary and conciliatory mechanism. agency without any statutory backing for its decisions. 2. It was given statutory status under the Legal Services Authorities Act, 1987. Source: The Hindu

Which of the following statement is/are Question 9 correct? Consider the following statements regarding to elections for Panchayati Raj Institution a) 1 only (PRI)

1. The power of state election commission to and on the like grounds as a Judge of conduct elections in PRI is Subject to the provisions of any law made by the a High Court and the conditions of Legislature of a State. service of the State Election 2. State Election Commissioner can be Commissioner shall not be varied to his removed by governor. disadvantage after his appointment.

Which of the following statement is/are Source: The Hindu correct? a) 1 only Question 10 Consider the following statement b) 2 only 1. Ganga rive isLargest river basin in India c) Both 1 and 2 2. River Hooghly is long distributary of the Ganges River d) Neither 1 nor 2 Which of the following statement is/are Answer: a correct?

Explanation: a) 1 only  The superintendence, direction and b) 2 only control of the preparation of electoral rolls for, and the conduct of, all c) Both 1 and 2 elections to the Panchayats shall be vested in a State Election Commission d) Neither 1 nor 2 consisting of a State Election Commissioner to be appointed by the Answer: c Governor. Explanation:  Subject to the provisions of any law River Ganga made by the Legislature of a State, the conditions of service and tenure of Largest river basin in India and it Originate in office of the State Election Gangotri glacier atUttarakhand. Commissioner shall be such as the River Hooghly Governor may by rule determine.  Long distributary of the Ganges River.  Provided that the State Election  Splits from Ganga - Canal in Commissioner shall not be removed Murshidabad District, Farakka Barrage. from his office except in like manner

Source: The Hindu  On May 21, the poet Sarojini Naidu led 2,500 marchers on the Dharasana

Salt Works, some 150 miles north of FEB 16 Bombay. Several hundred British-led Indian policemen met them and Question 1 viciously beat the peaceful Consider the following statement regarding demonstrators. civil disobedience movement:  In January 1931, Gandhi was released from prison. He later met with Lord 1. The poet Sarojni Naidu participated in Irwin, the viceroy of India, and agreed dharasana salt satyagraha to call off the satyagraha in exchange 2. Lord Irwin, the viceroy of India, and agreed for an equal negotiating role at a to call off the satyagraha in exchange for an London conference on India’s future. In equal negotiating role at a London conference August, Gandhi traveled to the on India’s future. conference as the sole representative of the nationalist Indian National Which of the following statement is/are Congress. correct? a) 1 only Source: Spectrum b) 2 only c) Both 1 and 2 Question 2 d) Neither 1 nor 2 Consider the following statement Answer: c 1. Leader of the house appoints the pro-tem Explanation: speaker to preside over the sittings of the newly elected house  The observance of the Independence Day in 1930 was followed by the 2. Pro term speaker cannot administer the floor launching of the Civil Disobedience test. Movement under the leadership of Gandhi. It began with the famous Which of the following statement is/are Dandi March of Gandhi. On 12 March correct? 1930, Gandhi left the Sabarmati a) 1 only Ashram at Ahmadabad on foot with 78 other members of the Ashram for b) 2 only Dandi, a village on the western c) Both 1 and 2 sea-coast of India, at a distance of about 385 km from Ahmadabad. d) Neither 1 nor 2

Answer: d

Explanation: 2. Wholesale Price Index (WPI) released by the Ministry of Commerce and Industry  Pro-tem is a Latin phrase which means “for the time being”. The Pro 3. Monetary policy committee’s (MPC) tem Speaker is a temporary speaker mandated limit of CPI at 6% (4% plus 2%) appointed for a limited period of time.  The need of pro-tem speaker: The speaker of the Lok Sabha/legislative Which of the following statement is/are assembly vacates the office correct? immediately before the first meeting a) 1 only of the newly elected house. b) 1 and 2 only  President/governor appoints the pro-tem speaker to preside over the c) 1 and 3 only sittings of the newly elected house. d) 1, 2 and 3 Usually, the senior most member of the house is made the pro-tem speaker. Answer: d Duties of pro tem speaker: Explanation:  The Pro-tem Speaker presides over the first sitting of the Lok Sabha, Whole Price Index administers the oath of office to the  WPI measures the changes in the prices newly elected MPs. of goods sold and traded in bulk by  To conduct the vote for the speaker wholesale businesses to other and deputy speaker. businesses. In other words, WPI tracks prices at the factory gate before  On the election of new Speaker, the the retail level. office of the pro tem speaker ceases to  The numbers are released by exist. the Ministry of Commerce and Industry  He also administers the floor test.  Even as the WPI is used as a key measure of inflation in some Source: Laxmikanth economies, the RBI no longer uses it for policy purposes, including setting repo rates. Question 3

Consider the following statement Consumer price index

1. Consumer Price Index (CPI) released by  It is an index measuring retail inflation Central Statistics Office (CSO) under in the economy by collecting the Ministry of Statistics and Programme change in prices of most common implementation goods and services used by consumers.  It is released by Central Statistics Office (CSO) under Ministry of

Statistics and Programme  Desert locusts are usually restricted to implementation the semi-arid and arid deserts of Africa,  In April 2014, the RBI had adopted the the Near East and SouthWest Asia that CPI as its key measure of inflation receive less than 200 mm of rain  The December inflation is also way annually. above the monetary policy  They lay eggs in damp soil in the committee’s (MPC) mandated limit of bare ground, which is rarely found in CPI at 6% (4% plus 2%) areas with dense vegetation.  Desert locusts are “biphasic” animals, meaning they can take on Source: RBI website two entirely different form  In their “solitary” form, they are drab Question 4 brown in colour and relatively harmless to crops. Consider the following statement  Under certain conditions (such as regarding desert locusts optimum moisture and vegetation), the 1. They lay eggs in damp soil in the bare insects can switch into a “gregarious ground. form” and start forming swarms – turning electric yellow and displaying 2. They are “biphasic” animals swarming behavior Which of the following statement is/are correct? Source: current affairs a) 1 only b) 2 only Question 5 c) Both 1 and 2 Consider the following statement d) Neither 1 nor 2 regarding plasma

Answer: c 1. Plasma is the liquid portion of blood “yellowish” in color Explanation: 2. Major part of blood is plasma  They belong to the family of 3. It helps maintain blood pressure and grasshoppers and have life span of 90 volume. days.  Four species of locusts are found in 4. It helps to maintain a proper pH balance in India: Desert locust (Schistocerca the body gregaria), Migratory locust (Locusta Which of the following statement is/are migratoria), Bombay Locust correct? (Nomadacris succincta) and Tree locust (Anacridium sp.) a) 1 and 2 only

b) 1and 3 only a) 1 and 2 only c) 1, 3, 4 only b) 2 and 3 only d) 1, 2, 3 and 4 c) 1 and 3 only d) 1, 2 and 3 Answer: d Explanation: Answer: a  Plasma is the liquid portion of blood Explanation: “yellowish” in color.  Himalayan mountain range in Asia  About 55% of blood is plasma, and separates the plains of the Indian the remaining 45% are red blood cells subcontinent from the Tibetan (RBC), white blood cells (WBC) and Plateau. They act as a climatic divide platelets that are suspended in the between the Indian sub-continent and plasma. Central Asia.  Plasma serves four important  They prevent frigid, dry winds from functions in body such as maintain blowing south into the subcontinent, a proper pH balance in the body; which keeps South Asia much maintain blood pressure and warmer than corresponding temperate volume etc. regions in the other continents. Source: NCERT  The trapping of the monsoon winds, forcing them to shed their moisture within the subcontinent. Question 6  Differential heating of land and sea and the shift of ITCZ is responsible How Himalayan Mountains effects on for creating different air pressure Indian climate? zones causing a reversal in the 1 It also blocks the cold winter storms of East direction of monsoon winds. Hence Asia from entering India, thus protecting us third statement is incorrect. from severe cold. Source: NCERT 2. They effectively intercept the summer monsoons coming from the Bay of Bengal and Arabian Sea and cause precipitation in the form of rain or snow. Current affairs 3. They plays role in formation of low pressure situation, which led to reverse monsoon Question 7 Which of the following statement is/are Which of the following statement is correct correct? regarding Defense Acquisition Council?

1. It is highest body for capital acquisitions Which of the following pair is matched for the three services and the Indian Coast correctly? Guard. a) 1 and 3 only 2. Chairperson of the Defense Acquisition b) 1 and 4 only Council is Chief of defense staff c) 4 only

d) 2 and 4 only Which of the following statement is/are correct? Answer: c a) 1 only Explanation: b) 2 only  S-400 missile defence system-----Russia c) Both 1 and 2  Arjun Tank------Indigenous  Apache helicopters---- USA d) Neither 1 nor 2  Rafael jets----- France Answer: a

Explanation: Source: The Hindu Defence Acquisition Council

 Highest decision-making body in the Defense Ministry for deciding on new Question 9 policies and capital acquisitions for the three services and the Indian Coast Guard. Consider the following statement  Minister of Defense is chairperson of the regarding Theru Koothu: Defense Acquisition Council. 1. It is rural art form from Tamil Nadu Source: The Hindu 2. It involves exaggerated make-up, high-pitched singing and delivery. 3. In this art actors strut about the stage with Question 8 violent thumping of the feet, and execute fast Match the following pairs pirouettes. 1. S-400 missile defence system---- Unites Which of the following statement is/are States of America correct? 2. Arjun Tank------Israel a) 1 only 3. Apache helicopters---- Russia b) 1 and 2 only 4. Rafael jets----- France c) 1 and 3 only d) 1, 2 and 3

Explanation:

Answer: d  It is an extinct genus of basal animal that lived during the late Explanation: Ediacaran period in what is now Theru Koothu Australia, Russia and Ukraine.

 A theatre art form - rural folk threatre  Basal animals are animals which of Tamil Nadu. have radial symmetry in their body  ‘Theru’ - street ‘Koothu’ - play. plans. They have very simple bodies and tend to  Traditional and popular form of folk be diploblastic (derived from only theatre from the interiors of Tamil two embryonic cell layers). Nadu - confined to Tondaimandalam of Tamil Nadu.  Thought to represent the earliest flowering of complex multicellular life  Combination of three main modes of on our planet, these creatures arose in a creative expression - verse recital, world devoid of predators, and had no song and dance. need for hard protective carapaces or  Serves as a means of entertainment skeletons. for rural folks, and displays a  Their soft, squishy bodies resembled ritualized trajectory of traditional tubes, fronds or even thin, quilted human values. pillows, they bore scant similarity to the anatomy of animals today.  Involves exaggerated make-up, high-pitched singing and delivery. Actors strut about the stage with Source: The Indian express violent thumping of the feet, and execute fast pirouettes. FEB 17

Source: The Hindu Question1

Question 10 WhatwasthereasonforMahatmaGandhitoo Dickinsonia is recently in news is rganizeaChamparanSatyagraha? a) Extinct genus of a Mammal b) Extinct genus of a basal animal 1. ThepeasantsinnorthernIndiaoppressedbyEur opeanindigoplantersthroughTinkathiya c) Extinct genus of an avian fossil system. d) Extinct genus of a Reptile fossil 2. RajKumarShuklaandSantRautpersuadedGa ndhi toinitiateamovement. Answer: b

Whichofthestatementsgivenaboveis/arecorrect d) Theparliamenthasthepowertoaltertheterritor ? yofstateboundaries. a) 1only Answer: a b) 2only c) Both 1and2 Explanation: d) Neither 1 nor 2  Indiaisafederalsystembutwithmoretiltto Answer: c wardsaunitarysystemofgovernment.Artic Explanation: le1ofthe Indian Constitution states,  Champaran district was the part of ‘India, that is Bharat, shall be a union of permanent settlement area which states’. The consisted of the wordfederationisnotmentionedintheconst largezamindariestatesunderrichand itution. influentiallandlords.Mostofthevilla geswereleasedoutbythezamindarst FeaturesoftheFederalSystemof India othikadarsofwhomthemostinfluenti alwereEuropeanIndigoPlanters. 1. Dualgovernmentpolity  The story of Champaran begins in 2. Divisionofpowersbetweenvariouslevel the early nineteenth century when s European planters 3. Rigidityofconstitution 4. Independencejudiciary hadinvolvedthecultivatorsinagreeme 5. Bicameralism ntsthatforcedthemtocultivateindigoo n3/20th(notall)oftheirholdings(know  Divisionofpowersbetweenthecentrean nasthetinkathiasystem). dstates–therearethreelistsgivenintheS  RajKumarShuklaandSantRau eventhScheduleoftheConstitutionwhi tpersuadedGandhitogotoCha chgivesthesubjectseachlevel mparan,andthus,theChampar hasjurisdictionin:

anSatyagrahabegan.  UnionList Source:Spectrum  StateList  Concurrent List Question2

Thereisafederalismof GovernmentinIndiabecausethe

a) Divisionofpowersbetweenvariouslevels.

b) CouncilofMinistersisresponsibletotheLokSa bha.

c) LokSabha ismorepowerfulthantheRajyaSabha.

a39:39:22formulas. Source: Laxmikanth Source:EconomicSurvey Question3

Question4 ConsiderthefollowingstatementregardingN ationalInfrastructurePipeline Considerthefollowingstatement

1. Ithasoutlinedplanstoinvestmorethan102lakh 1. Regularextractionofgroundwaterforirrigatio croreoninfrastructureprojectsby2024-25. ndepositsarsenicinsoil. 2. The entry of arsenic into the food chain, in addition to drinking water 2. Centrewillspendallthecapitalexpenditure. increases possibilities of

biomagnification. 3. NIPincludeseconomicandsocialinfrastructur Whichofthestatementsgivenabove eprojects. is/arecorrect?

a) 1only Whichofthestatementsgivenaboveis/arecorre b) 2only ct? c) Both 1and2 d) Neither 1 nor 2 a) 1only b) 2and3only Answer: c c) 1and3only Explanation: d) 1, 2 and 3  Arseniccontaminationingroundwaterw illpenetratedthefoodchain. Answer: c  Regular extraction of ground Explanation: water for irrigation deposits arsenic in soil and consequently NationalInfrastructurePipeline itsuptake by the crops. Also, paddy farms flooded with  NIP will enable a forward outlook on infrastructure contaminated water eventually projects which will create jobs, causesaccumulationofarsenicinthef improveease of living, and oodcrops. provide equitable access to  Theentryofarsenicintothefoodchai infrastructure for all, thereby n,inadditiontodrinkingwaterincrea making growthmoreinclusive. sespossibilitiesofbiomagnification.  NIPincludeseconomicandsocialinfrastr  Bio magnification is concentration uctureprojects.  Ithasoutlinedplanstoinvestmoreth of a toxin (such as pesticides) at an successively higher levels ₹102lakhcroreoninfrastructurepr inafoodchain. ojectsby2024-25,withtheCentre, Statesandtheprivatesector tosharethecapitalexpenditurein

 Drinkingofarsenic-richwaterresul Answer:b tsinskincancer,cancersofthebladd er,kidneyandlung,diseasesofthebl oodvesselsandreproductivedisord Explanation: ers  Itwillbeusedformakingtollp aymentsdirectlyfromthecus Source:PIB tomerslinkedprepaidorsavi ngs/currentaccount. Question5  RadioFrequencyIdentificationtechnolo Considerthefollowingstatementswithrefere gy(RFID)passivetag.  Affixed on the windscreen of the ncetoFASTag: vehicle - enables the customer to drive through toll plazas -need not stop for any toll payments - 1. RadioFrequencyIdentificationtechnology(R making toll payments directly FID)usesinFASTag: while the vehicle is inmotion.

 Vehiclespecific-onceaffixedtoavehicl 2. AcustomerisallottedoneFASTagandcanuseit e,cannotbetransferredtoanothervehi onanynumberofvehicles. cle.  CanbepurchasedfromanyoftheNationa lElectronicTollCollection(NETC)Me 3. FASTag mberBanks canbeblacklistedonlybythelawenforcementa  Linkedtoprepaidaccount-nee nagencyduetoanyruleviolationcomplaintisre dstoberechargedortopped-up gisteredagainstthevehicle/customer. aspertheusageofthecustomer.  Inadequatebalance-FASTagge tsblacklistedatthetollplaza-cust Whichofthestatementsgivenaboveis/areinc omerwon'tbeabletoavailtheNE orrect? TCservicesandwouldberequire dto paythetollfarethroughcash.  Mobileapplication“MyFASTag”–ne wfeaturethatenablestocheckbalances (a) 1only tatusbyenteringthevehiclenumber.

(b) 2and3only Source: TheHindu

(c) 1and2only

(d) 1,2and3

Question6 aretheyoungestand theageoftherocksincreasesasonemov Considerthefollowingstatementregardingsea eawayfromthecrest(ridge). floorspreading

Source: NCERTCurrent affairs

1. Seafloorspreadinghelpsexplaincontinentaldr iftinthetheoryof platetectonics. Question7

2. The evidence of sea floor spreading is InternationalIntellectualPropertyIndexpu rocks closer to the mid-oceanic ridges have blishedby normal polarity andaretheyoungestandtheageoftherocksincre asesasonemovesawayfromthecrest(ridge). a) UnitedNationsEducational,ScientificandCu lturalOrganization Whichofthestatementsgivenaboveis/arecorrect ? b) USChamberofCommerce’sGlobalInnovati a) 1only onPolicyCentre(GIPC) b) 2only c) Both 1and2 d) Neither1nor2 c) WorldTradeOrganization

Answer: c Explanation: d) WorldBank

Answer: b  HarryHessproposedtheideaofSeeFloorSpr eading. Explanation:  Whenoceanicplatesdiverge,tensionalstres scausesfracturesto occurinthelithosphere. InternationalIntellectualPropertyIndexwasrel  Basalticmagmarisesfromthefracturesandc easedbyUSChamberofCommerce’sGlobalInn oolsontheoceanfloortoformnewseafloor. ovationPolicyCentre(GIPC).  Thenewlyformedseafloor(oceaniccrust) thengraduallymovesawayfromtheridge, anditsplaceistakenbyanevennewerseafl oorandthecyclerepeats. India’sranked40thamong53  With time, older rocks are spread farther countriesin2020,whilein2019Indiawasranke away from the spreading zone while younger rocks will dat 36th positionoutof50countries. befoundnearertothespreadingzone. Source: PIB  Seafloorspreadinghelpsexplaincontine ntaldriftinthetheoryofplatetectonics.  Rocksclosertothemid-oceanicridgesh avenormalpolarityand

Question 8 Question9 Considerthefollowingstatementsregardingth ePetroleum&ExplosivesSafetyOrganization( PESO): ConsiderthefollowingstatementregardingMa rineTurtlesofIndia: 1. ItComesundertheMinistryofCommerce.

2. PESO’s major work is to administer the 1. Flatback responsibilities delegated under the 2. Hawksbillturtles Explosives Act 1884 3. Greenturtles 4. Leatherbackturtles andPetroleumAct1934andtheRulesmad ethereunderwiththemotto"SafetyFirst". Whichofthefollowingturtlesis/areinhabitinIn Whichofthestatementsgivenaboveis/arecorrect dia? ? a) 1and2only b) 1, 2 and 3 only a) 1only b) 2only c) 1, 3 and 4 only c) Both 1and2 d) Neither1nor2 d) 2, 3 and 4

Answer: c Answer: d Explanation: Explanation:

Petroleum&ExplosivesSafetyOrganization(PE SO) MarineTurtlesofIndia:  FormerlyknownastheDepartmentofEx plosivesestablishedin1898.  Thenodalagencyforregulatingsafety ofhazardoussubstancessuchasexplos • Five species of sea turtles are known to ives,compressedgasandpetroleum. inhabit Indian coastal waters and islands.  Entrustedwiththeadministrationof The Olive :ExplosivesAct,1884,PetroleumAct Ridley,Green,Hawksbill,Loggerheada ,1934;InflammableSubstancesAct, ndtheLeatherbackturtles. 1952andtheRulesframedunderthes eActs. • ExcepttheLoggerhead,theremainingfourspec  HeadedbytheChiefControllerofExplosi iesnestalongtheIndiancoast. ves-HQatNagpur.  ComesunderDepartmentforPromotion ofIndustryandInternalTrade,Ministryo Source:TheHindu fCommerce.

Source:TheHindu

Question10 • AnnuallycoordinatedbytheBombay NaturalHistorySociety(BNHS)and ConsiderthefollowingstatementAsianwaterb WetlandsInternational. irdcensus • Apartoftheglobalproject‘International

WaterbirdCensusprogrammeandiscar 1. ItisInternationalprogrammethatfocuseson riedoutevery yearinJanuary. monitoringthestatusofwaterbirdsandwetlands . 2. AsianwaterbirdCensusprogrammeiscarriedo Source:TheMint utbiannually.

Whichofthefollowingstatementis/arecorrect?

a) 1only b) 2only c) Both1and2 d) Neither1nor2

Answer: a Explanation:

AsianWaterbirdCensus-2020

• Coversatleasttwodozensites,includin gCoringaWildlifeSanctuary,Kolleru LakeandKrishnaSanctuary

Asianwaterbirdcensus:

• Internationalprogrammethatfocus esonmonitoringthestatusofwaterbi rdsandwetlands. • Aim-increasepublicawarenessonissue srelatedtowetlandandwaterbirdconser vation•

FEB 18  Attempted to fuse the tenets of Hinduism and Islam to serve as a Question 1 guide for all humanity. With reference to the religious history of  He rebelled against a society that medieval India, the bhakti mystics was preferred ritual to devotion and known to pursue which of the following sincerity. Among the institutions that practices? he challenged was caste.  They believed that god is one. 1. Monotheism in religious practices Source: NCERT 2. Fusion of religion to guide humanity in right direction

3. Their devotion is to the cause of economic Question 2 justice and a more egalitarian world. With reference to the Fundamental Rights, Which of the following statement is/are consider the following statements correct? 1. Fundamental rights, available against the a) 1 only State’s action only not private actions b) 1 and 2 only 2. Only the Parliament can make law for giving effect to the Fundamental Rights c) 1 and 3 only Which of the statements given above is/are d) 1, 2 and 3 correct?

Answer: d a) 1 only

Explanation: b) 2 only

 The Bhakti movement started in the 7th c) Both 1 and 2 Century-8th Century in Kerala and Tamil Nadu. Later it spread to d) Neither 1 nor 2 Karnataka, Maharashtra, and it reached Answer: c North India in the 15th Century. The Bhakti movement reached its peak in Explanation: the 15th Century and 17th Century. • To maintain the uniformity only  Bhakti traditions are not religious in the parliament can make law. sense of emphasizing worship and • Most of the Fundamental Rights are devotion. available against the arbitrary action of  Their concern, indeed devotion, is to the State, with a few exceptions like the cause of economic justice and a those against the State’s action and more egalitarian world. against the action of private individuals.

• When the rights which are available one percentage of their net demand and against the State’s action only are time liabilities (NDTL). violated by the private individuals, Source: Ramesh singh there are no constitutional remedies but only ordinary legal remedies

Source: Laxmikanth Question 4

Question 3 With reference to the percentage of mangrove cover to the total area of State Consider the following statement regarding and union territories, Consider the Marginal Standing facility (MSF) following state and union territories: 1. Banks to borrow from the Reserve Bank of 1. Andaman and Nicobar islands India in an emergency situation only 2. Gujarat 2. Under MSF, banks can borrow funds up to one percentage of their net demand and time 3. Andhra Pradesh liabilities 4. West Bengal Which of the statements given above is/are correct? which one of the following is the correctdescending order? a) 1 only a) 2-3-1-4 b) 2 only b) 2-3-4- c) Both 1 and 2 c) 4-2-1-3 d) Neither 1 nor 2 d) 3-2-1-4 Answer: c Answer: c Explanation: Explanation:  Marginal standing facility (MSF) is a window for banks to borrow from the  Mangroves are a diverse group of Reserve Bank of India in an emergency salt-tolerant plant communities of situation when inter-bank liquidity dries up tropical and subtropical regions of the completely. world which can survive the limiting  Banks borrow from the central bank by factors imposed by lack of oxygen, pledging government securities at a rate high salinity and diurnal tidal higher than the repo rate under liquidity inundation. adjustment facility or LAF in short.  The mangrove cover in India is 4,975  The MSF rate is pegged 100 basis points or sq km, which is 0.15% of the a percentage point above the repo rate. country’s total geographical area. Under MSF, banks can borrow funds up to

 Among the states and UTs, West participants that can remain Bengal has the highest percentage of anonymous. area under total Mangrove cover • The block chain technology generally followed by Gujarat and Andaman has key characteristics of Nicobar Islands. decentralization, persistency, anonymity and auditability. With these Source: forest survey report traits, block chain can greatly save the cost and improve the efficiency.

Source: World Economic Forum website

Question 5 With the present state of development,

Block chain Technology can effectively do which of the following? Question 6

1. Enhance transparency, traceability and trust With reference to theHimalayan Rivers in e-governance systems consider the following statements

2. Data structure can remain anonymous 1. They have antecedent drainage system

3. Block chain can greatly save the cost and 2. They usually develops shallow valleys improve the efficiency. 3. They are still in Youth full stage Which of the statements given above is/are correct? Which of the statements given above is/are correct? a) 1 only a) 1 only b) 2 and 3 only b) 2 and 3 only c) 1 and 3 only c) 1 and 3 only d) 1, 2 and 3 d) 1, 2 and 3 Answer: d Answer: c Explanation: Explanation: • A block chain is a specific type of data structure which can be used to transact  These rivets originate from the lofty across nodes or participants. The Himalayan ranges and are named as the ownership rights are recorded in Himalayan Rivers. cryptographically stored and linked  These rivers have large basins and blocks which contain records of catchment areas. The total basin area of ownership of assets among the the Indus, the Ganga and the

Brahmaputra is 11.78, 8.61 and 5.8 Explanation: lakh square kilometers respectively.  The Himalayan rivers flow through  As per Article 155 and Article 156 of the Constitution, a Governor of a state deep V – shaped valleys called gorges. is appointed by the President, and he or These gorges have been carved out by down cutting carried on side by side she holds office “during the pleasure of with the uplift of the Himalayas. the President” for a term of five years.  These are examples of antecedent  As President is bound to act on the drainage. aid and advice of the Council of Ministers under Article 74 of the  These rivers flow across the young fold Constitution, in effect it is the central mountains and are still in government that appoints and a youthful stage. removes the Governors. “Pleasure of Source: NCERT the President” merely refers to this will and wish of the central government. Current affairs  A decision to remove a Governor can be challenged in a court of law. In such cases, first the petitioner will have to make a prima facie case of Question 7 arbitrariness or bad faith on part of the Consider the following statement regarding central government. governor  If a prima facie case is established, the court can require the central 1. Governor holds office during the pleasure of government to produce the materials on the President. Pleasure of the Presidentmerely the basis of which the decision was refers to this will and wish of the central made in order to verify the presence of government. compelling reasons.

2. A decision to remove a Governor can be Source: PrsIndia.org challenged in a court of law. Question 8 Which of the statements given above is/are correct? Consider the following statement regarding Pradhan Mantri Jeevan Jyoti Bima Yojana a) 1 only 1. The PMJJBY is available to people in the b) 2 only age group of 18 to 50 years. c) Both 1 and 2 2. The eligible people should have a bank d) Neither 1 nor 2 account who gives their consent to join or enable auto-debit. Answer: c

Which of the statements given above is/are Which of the statements given above is/are correct? correct? a) 1 only a) 1 only b) 2 only b) 2 only c) Both 1 and 2 c) Both 1 and 2 d) Neither 1 nor 2 d) Neither 1 nor 2

Answer: c Answer: a

Explanation: Explanation;

 The Pradhan Mantri Jeevan Jyoti Bima  The Finance Commission (FC) is Yojana was launched by the Central a constitutional body, that determines Government of India that provides life the method and formula coverage to the poor and low-income for distributing the tax proceeds section of the society. The scheme between the Centre and states, and offers a maximum assured amount of among the states as per the Rs.2 lakhs constitutional arrangement and  The PMJJBY is available to people in present requirements. the age group of 18 to 50 years having  Under Article 280 of the Constitution, a bank account who give their consent the President of India is required to to join / enable auto-debit. Aadhar constitute a Finance Commission at would be the primary KYC for the bank an interval of five years or earlier. account.  The 15th Finance Commission was  The premium is Rs. 330 per annum constituted by the President of India which is to be auto-debited in one in November 2017, under the installment from the subscriber’s bank chairmanship of NK Singh. Its account. recommendations will cover a period of five years from the year 2021-22 to Source: The Indian Express 2025-26. Question 9  It has recommended maintaining the vertical devolution at 41% Consider the following statement regarding

15th Finance Commission’s

1. Demographic performance added in revenue sharing formula

2. It has recommended maintaining the vertical devolution at 71%

Geospatial data:

 Data types that have a spatial component - data are connected to a place in Earth.  Data about objects, events, or phenomena that have a location on the surface of earth. Location - static in the short-term (location of a road) or dynamic (moving vehicle or pedestrian).  Technological progress made for capturing of geospatial data through many techniques / technologies -

satellite-based remote sensing etc.  Presently India relies heavily on foreign resources for mapping Source: Economic survey technologies and services. Question 10 Source: The Hindu

Which of the following statements is/are correct with reference to Geospatial data?

1. It is the data exclusively about objects that have a location on the surface of the earth. FEB 19

2 It uses satellite-based remote sensing. Question 1

3. Presently India has self-reliance in geo Which of the following were the main spatial data. proposals of Cripps Mission?

Select the correct answer from the code 1. The making of the constitution was to be given below. solely in Indian hands a) 1 only 2. Provinces were given free hand to form separate constitution b) 1 and 2 only 3. Indians were allowed a large share in the c) 2 only administration in the interim period d) 2 and 3 only Select the correct answer using the code Answer: c given below:

Explanation: a) 1 and 2 only b) 1 and 3 only

c) 2 and 3 only b) 1 and 2 only d) 1, 2 and 3 c) 2 and 3 only

Answer: d d) 1, 2 and 3

Explanation: Answer: c

The Cripps proposals Explanation:

 The making of the constitution was to The procedure of renaming of the state can be be solely in Indian hands now. initiated by either the Parliament or the State  A concrete plan was provided for the Legislator and the procedure is as follows: Constituent Assembly.  The renaming of a state requires  Option was to be made available to any Parliamentary approval under province to have a separate Article 3 and 4 of the Constitution. Constitution.  A bill for renaming a state may be  Free India could withdraw from the introduced in the Parliament on the Commonwealth. recommendation of the President.  Indians were allowed a large share in  Before the introduction of the bill, the President shall send the bill the administration in the interim period to the respective state assembly for expressing their views within a Source: Spectrum stipulated time. The views of the state assembly are binding, neither on the President nor on the Parliament. Question 2  On the expiry of the period, the bill will be sent to the Parliament for With respect to the change in name of any deliberation. The bill in order to take State in India, consider the following the force of a law must be passed by a statements: simple majority.  The bill is sent for approval to the 1. The renaming of a state requires President. After the approval of the said Parliamentary approval with special majority bill, the bill becomes a law and the name of the state stands modified. 2. Prior recommendation of the President is must and should Source: Laxmikanrh

3. Before recommending the Bill, the President has to refer the same to the State Legislature Question 3 concerned for expressing its views within a specified period In the context of India, Which of the following can be the implications of Which of the following statement is/are increasing Statutory Liquidity Ratio (SLR) correct? by the Reserve Bank of India? a) 1 only 1. Inflation will decrease

2. Commercial bank lending capacity will Select the correct answer using the code increase given below.

Select the correct answer using the code a) 1 only given below. b) 1 and 2 only a) 1 only c) 2 and 3 only b) 2 only d) 1, 2 and 3 c) Both 1 and 2 Answer: c d) Neither 1 nor 2 Explanation: Answer: a  The Ganges River dolphin inhabits Explanation: the Ganges-Brahmaputra-Meghna and Karnaphuli-Sangu river systems 16. Statutory Liquidity Ratio or SLR is a of Nepal, India, and Bangladesh. minimum percentage of deposits that a  It is National Aquatic animal of India. commercial bank has to maintain in the  It has been categorized as endangered form of liquid cash, gold or other on the Red List of Threatened Species securities. by the IUCN. 17. It is basically the reserve requirement that  It is also protected under the Schedule banks are expected to keep before offering I of the Wildlife Protection Act (1972) credit to customers. 18. Implications:  The presence of the Dolphin in a river system signals its good health  It increases to control inflation by and biodiversity. decreasing money supply  Vikramshila Gangetic Dolphin  It will decrease lending capacity of Sanctuary (VGDS) in Bihar is India's banks because they have less money only sanctuary for the Gangetic Source: Mrunal dolphin.

Source: Shankar Environment book

Question 4

Consider the following statement: Question 5

1. The Ganges River dolphin endemic to India Consider the following statement: only 1. Organochlorines are uses as a pesticides. 2. It has been categorized as endangered on the 2. Organochlorines are capable of causing Red List of IUCN adverse effects in the central and peripheral 3. It act as indicator species nervous system.

Select the correct answer using the code Select the correct answer using the code given below. given below. a) 1 only a) 1 only b) 2 only b) 2 only c) Both 1 and 2 c) Both 1 and 2 d) Neither 1 nor 2 d) Neither 1 nor 2

Answer: c Answer: d

Explanation: Explanation:

 Organochlorines are a group of  The western coastal plains are an chlorinated compounds that belong to example of submerged coastal plain. the class of persistent organic pollutants  Because of this submergence it is a (POPs) narrow belt and provides natural  POPs are chemicals of global concern conditions for the development of ports due to their potential for long-range and harbors. transport, persistence in environment,  Eastern coastal plain is broader and ability to bio-magnify and bio is an example of an emergent coast. accumulate in ecosystems. Because of its emergent nature, it has  Most commonly encountered POPs are less number of ports and harbors. organochlorine pesticides, such as  The continental shelf extends up to 500 DDT, industrial chemicals etc. km into the sea, which makes it  Organochlorines are capable of causing difficult for the development of good adverse effects in the central and ports and harbors peripheral nervous system. Source: NCERT Source: Current Affairs

Current affairs Question 6 Question 7 Consider the following statements with Consider the following statement regarding reference to Indian Coastal Plains: Glaucoma 1. The western coastal plains are an example of 1. It is disease related to eye emergent nature coastal plain 2. There is no cure for glaucoma 2. Eastern coastal plain are an example of submerged nature coastal plain Select the correct answer using the code given below.

a) 1 only a) 1 only b) 2 only b) 2 only c) Both 1 and 2 c) Both 1 and 2 d) Neither 1 nor 2 d) Neither 1 nor 2

Answer: c Answer: c

Explanation: Explanation:

 Glaucoma is a condition that damages  As per the World Health Organisation your eye's optic nerve. It gets worse (WHO), telemedicine is the delivery of over time. It's often linked to a buildup health care services, where distance is a of pressure inside your eye. critical factor, by all health care  There is no cure for glaucoma, but professionals using Information early treatment can often stop the Technology (IT) for diagnosis, damage and protect the vision treatment and prevention of disease and  The fluid inside your eye, called injuries, research and evaluation, etc. aqueous humor, usually flows out of all in the interests of advancing the your eye through a mesh-like channel. health of individuals and their If this channel gets blocked, or the eye communities. is producing too much fluid, the liquid  It is a doctor to doctor telemedicine builds up. Sometimes, experts don’t system, being implemented under know what causes this blockage. But it the Ayushman Bharat Health and can be inherited, meaning it’s passed Wellness Centre (AB-HWCs) from parents to children. programme.  AB-HWCs are envisaged to be the Source: The Hindu platform for delivery of an expanded range of primary health care services closer to the communities. Question 8

Consider the following statement regarding Source: The Hindu eSanjeevani:

1. It is a doctor to doctor telemedicine system

2. It is implemented under the Ayushman Bharat Health and Wellness Centre Question 9 (AB-HWCs) programme. Consider the following statements regarding Select the correct answer using the code Securities Appellate Tribunal (SAT) given below. Presiding officer

1. Presiding officer of the SAT appointed in grounds of proven misbehavior or consultation with the CJI or his nominee. incapacity.

2. A sitting or retired Judge of the Supreme Source: The Hindu Court or a sitting or retired Chief Justice of a High Court only appointed as presiding officer. Question 10

3. Presiding officer removal only through an Consider the following statements with order by the central government on grounds of reference to Securities and Exchange proven misbehavior or incapacity. Board of India.

Select the correct answer using the code 1. It is a statutory body given below. 2. Securities Appellate Tribunal (SAT) has a) 1 only been constituted to protect the interest of entities that feel aggrieved by SEBI’s b) 1 and 2 only decision c) 2 and 3 only Select the correct answer using the code d) 1, 2 and 3 given below.

Answer: d a) 1 only

Explanation: b) 2 only

 SAT is a statutory body established c) Both 1 and 2 under the provisions of Section 15K d) Neither 1 nor 2 of the SEBI Act, 1992. Answer: c  SAT consists of a Presiding Officer and two other members. The Explanation: Presiding officer of SAT shall be appointed by the Central  SEBI is a statutory body established Government in consultation with on April 12, 1992 in accordance with the Chief Justice of India or his the provisions of the Securities and nominee. Exchange Board of India Act, 1992.  The basic functions of the Securities  Qualification: A sitting or retired and Exchange Board of India is to Judge of the Supreme Court or a protect the interests of investors in sitting or retired Chief Justice of a securities and to promote and regulate High Court. - A sitting or retired the securities market. Judge of a High Court with not less  SEBI Board consists of a Chairman than 7 years of service as a high and several other whole time and part court judge. time members.  SEBI also appoints various  Removal : only through an order committees, whenever required to by the central government on

look into the pressing issues of that Provisions of the Government of India Act time. 1858  A Securities Appellate Tribunal (SAT) has been constituted to protect  Indian territories of Britain were to be the interest of entities that feel governed in the name of the British aggrieved by SEBI’s decision. Queen. Source: The Hindu  The Court of Directors and the Board of Control were scrapped.

FEB 20  The powers of the Company’s Court of Directors were vested with the Question 1 Secretary of State for India.  The representative of the British Consider the following statement regarding government in India was the Government of India Act 1858 Governor-General and Viceroy (both 1. This act made India a direct British colony. the same person to avoid conflict).  The Viceroy and the governors of the 2. This act also ended the doctrine of lapse. various presidencies were appointed by the Crown. 3. There was provision for Indians also to be  The Viceroy was to be assisted with an admitted to the service. Executive Council.

 This act made India a direct British Select the correct answer using the code colony. given below:  This act abolished the dual government of the Pitt’s India Act. a) 1 and 2 only  This act also ended the doctrine of b) 1 and 3 only lapse. c) 2 and 3 only  The Indian Civil Services was to be instituted for the administration of d) 1, 2 and 3 the country. There was provision for Indians also to be admitted to the Answer: d service. Explanation:  It was decided that the remaining Indian princes and chiefs (more than  The Government of India Act 1858 was 560 in number) would have their an Act of the British parliament that independent status provided they accept transferred the government and British suzerainty. territories of the East India Company to the British Crown. Source: NCERT

Question 2

Consider the following statement regarding Question 3 Lok Sabha speaker: Consider the following statement regarding 1. Speaker can be removed only by a resolution Exchange Traded Funds passed by the Lok Sabha by an absolute majority 1. Exchange Traded Funds are mutual funds

2. The Speaker appoints the Chairman of all 2. They offer anytime liquidity through the the parliamentary committees of the Lok Sabha exchanges. Select the correct answer using the code given 3. The speaker will resign by submitting resignation letter President below. a) 1 only Select the correct answer using the code given below: b) 2 only a) 1 and 2 only c) Both 1 and 2 b) 1 and 3 only d) Neither 1 nor 2 c) 2 and 3 only Answer: c d) 1, 2 and 3 Explanation:

Answer: a  Exchange Traded Funds (ETFs) are mutual funds listed and traded on Explanation: stock exchanges like shares.  The Speaker of the Lok Sabha is  In an ETF, one can buy and sell units at provided with a security of tenure. He prevailing market price on a real time can be removed only by a resolution basis during market hours. passed by the Lok Sabha by an  ETFs are cost efficient. Given that they absolute majority. don’t make any stock (or security  The Speaker appoints the Chairman choices), they don’t use services of star of all the parliamentary committees fund managers. of the Lok Sabha and supervises their  They allow investors to avoid the risk functioning. He himself of the of poor security selection by the fund Chairman of the Business Advisory manager, while offering a diversified Committee, the Rules Committee and investment portfolio. the General Purpose Committee.  The stocks in the indices are carefully  A speaker tender his resignation to selected by index providers and are a Deputy Speaker rebalanced periodically. Source: Laxmikanth  They offer anytime liquidity through the exchanges.

Source: Mrunal economy exclusively for the preservation of migrant animals.

Sources: Shankar Environment book Question 4

Consider the following statement with reference to convention on the conservation Question 5 of migratory species of wild animals: Consider the following statement: 1. It aims to protect only terrestrial and avian migratory species throughout their ranges 1. Food fortification is removal of one or more micronutrients 2. It is the only worldwide convention that was founded exclusively for the preservation of 2. The deficiency of micronutrients is also migrant animals. known as “hidden hunger”

Select the correct answer using the code given Select the correct answer using the code given below. below. a) 1 only a) 1 only b) 2 only b) 2 only c) Both 1 and 2 c) Both 1 and 2 d) Neither 1 nor 2 d) Neither 1 nor 2

Answer: b Answer: b

Explanation: Explanation:

 It aims to protect terrestrial, aquatic  Food fortification is the deliberate and avian migratory species addition of one or more throughout their ranges. micronutrients to food so as to  CMS was signed in Bonn, Germany, in correct or prevent a deficiency and 1979 as an intergovernmental treaty provide a health benefit. under the aegis of the United Nations  According to Food Safety and Environment Programme (UNEP) Standards (Fortification of Foods)  The legal instruments under CMS may Regulations, 2018 fortification of staple range from legally binding Agreements foods is not mandatory. to less formal Memoranda of  Food fortification is a “complementary Understanding and can be adapted to fit strategy” and not a replacement of a the requirements of each region. balanced & diversified diet to address  CMS is the only worldwide malnutrition. convention that was founded  Hidden hunger occurs when the quality of food people eat does not

meet their nutrient requirements, so  Examples of Residual Mountains in the food is deficient in India are the Nallamala hills, micronutrients such as the vitamins Mahendragiri hills, the Aravalli hills, and minerals that they need for their the Javadi hills, the Veliconda hills, growth and development. and the Palkonda range.

Source: WHO website Source: NCERT

Current affairs

Question 6 Question 7

Consider the following statement regarding Consider the following statement residual mountains 1. Tree Cities of the World programme is an 1. Residual mountains are developed through international program recognized by United erosion Nations Food and Agriculture Organization.

2. Aravalli hills are example of residual 2. Hyderabad is the only city from India to get mountains recognition as a Tree City

Select the correct answer using the code Select the correct answer using the code given below. given below. a) 1 only a) 1 only b) 2 only b) 2 only c) Both 1 and 2 c) Both 1 and 2 d) Neither 1 nor 2 d) Neither 1 nor 2

Answer: c Answer: c

Explanation: Explanation:

 Due to the agents of erosion, the  The Arbor Day Foundation and the softer rocks of the upland are eroded UN’s Food and Agriculture much more rapidly than the areas of Organization have recognized harder rock. Hyderabad as ‘2020 Tree City of the  After thousands of years, soft rocks are World’. worn down into the sand and the hard  Tree Cities of the World programme is rocks are left standing up in the area an international program. It recognizes that has been reduced in height. These cities and towns for properly and are called residual mountains. sustainably maintaining their urban  The residual mountains are also called forests and trees. relict mountains.

 Hyderabad is selected as a Tree City because of its initiatives like Haritha Haram programme and Urban Forest Parks plan.  Hyderabad is the only city from India to get recognition as a Tree City. With this recognition, Hyderabad joins 120 other cities from 23 countries including the US, the UK, Canada, and Australia.

Source: The Hindu

Question 8 Question 9 Consider the following countries: Making Peace with Nature Report 1. Cambodia published by

2. China a) World Economic Forum

3. Laos b) Inter Panel Climate Change

4. Myanmar c) United Nation Environment Programme

Which of the above countries border d) Global Environment Facility Vietnam? Answer: c a) 1 and 2 only Explanation: b) 1, 2 and 3 only  The first UNEP synthesis report is c) 2, 3 and 4 only titled: “Making Peace With Nature: A d) 1, 2, 3 and 4 scientific blueprint to tackle the climate, biodiversity and pollution Answer: b emergencies” and is based on evidence from global environmental assessments Explanation: Source: The Hindu  Vietnam is bordered by China to the north, the South China Sea to the east and south, the Gulf of Thailand (Gulf of Siam) to the southwest, and Cambodia and Laos to the west. Question 10

Consider the following statement:

1. The Parliament can abolish a legislative Source: The Hindu council or create by a simple majority.

2. The Legislative Council of a state shall not FEB 21 have more than one-half of the total strength of the State Assembly Question 1

Select the correct answer using the code Consider the following statement with given below. reference to Kakatiya Dynasty a) 1 only 1. They are known for construction of a b) 2 only network of tanks for irrigation and drinking water. c) Both 1 and 2 d) Neither 1 nor 2 2. Thousand Pillar Temple or Rudreshwara Swamy Temple builds during Kakatiya period. Answer: a Select the correct answer using the code Explanation: given below.  The Parliament can abolish a legislative council (where it already a) 1 only exists) or create it (where it does not exist) by a simple majority, that is, a b) 2 only majority of the members of each c) Both 1 and 2 House present and voting, if the legislative assembly of the concerned d) Neither 1 nor 2 state, by a special majority, passes a resolution to that effect. Answer: c  Under Article 171 of the Constitution, the Legislative Explanation: Council of a state shall not have more than one-third of the total  Kakatiyas is an Andhra strength of the State Assembly, and dynasty that flourished in the not less than 40 members. 12th century CE. The Kakatiya dynasty ruled from Warangal  Like the Rajya Sabha, the legislative (Telangana) from CE 1083-1323. council is a continuing chamber, that  They were known for is, it is a permanent body and is not the construction of a network of subject to dissolution. The tenure of tanks for irrigation and drinking a Member of the Legislative Council water and thereby gave a big boost (MLC) is six years, with one-third of to the overall development of the the members retiring every two years. region.

 There are hundreds of Hindu like Aadhar Bill, Insolvency, and temples built under the patronage of Bankruptcy Bill are also related to it. Kakatiya kings like Ganapati Deva,  Money Bill is defined in Article 110 Rudrama Devi and Prataparudra of of the Indian Constitution. Money Kakatiya dynasty. E.g. bills are concerned with financial matters like taxation, public expenditure, etc.  Thousand Pillar Temple or  To introduce this bill, Rudreshwara Swamy recommendation of President is Temple, Telangana. It is a required. star-shaped, triple shrine  Rajya Sabha does not have the (Trikutalayam) dedicated to Vishnu, power to amend or reject the Shiva and Surya. Money Bill.  Ramappa Temple, Warangal,  Whether a bill is a money bill or not Telangana. is decided by the Speaker of Lok Sabha

 Money Bill can be introduced only Source: NCERT in Lok Sabha

Question 2 Source: Laxmikanth Consider the following statement with reference to money bill Question 3 1. To introduce money bill recommendation Which among the following could be the of President is required reasons for the low tax to GDP ratio? 2. Money Bill can be introduced only in Lok 1. Complex laws Sabha 2. High share of Informal jobs 3. Rajya Sabha can reject the Money Bill 3. Tax exemption of agriculture Select the correct answer using the code given below: Select the correct answer using the code given below: a) 1 and 2 only a) 1 and 2 only b) 1 and 3 only b) 1 and 3 only c) 2 and 3 only c) 2 and 3 only d) 1, 2 and 3 d) 1, 2 and 3 Answer: a Answer: d Explanation: Explanation:  Money bills are concerned with financial matters like taxation, public  Tax to gross domestic product (GDP) expenditure, etc. The bill is ratio is the ratio of taxes collected by significant for Indian Polity and a government and the GDP of the governance as many important issues nation.

 Taxes constitute an important for the convention were started in the component of revenue and the late 1980s under the auspices of aforesaid ratio is a key barometer that the United Nations Environment indicates the ability of the Programme (UNEP). government to invest in various development initiatives. India has had  It came into force in 1992. a comparatively low tax-to-GDP  The Basel Convention secretariat is ratio. situated in Geneva, Switzerland.  Lower per capita income, tax  It applies Prior Consent litigations, tax exemptions for Approval procedure to regulate the agricultural income and lower transboundary movement of the compliances are reasons for low tax to hazardous and other wastes. GDP ratio.  Non-parties cannot transport Source: NIOS hazardous waste to and from each other unless specially agreed. Basel Question 4 Convention states such transportation, illegal. Consider the following statement with reference to Basel Convention: Source:Shankar environment book

1. It aims to protect the environment by bringing measures to control and regulate Question 5 hazardous and other waste disposals. With reference to space technology, 2. It regulates the Trans boundary movement consider the following statements: of the hazardous and other wastes. 1. Geosynchronous Orbit (GEO) takes a Select the correct answer using the code given satellite around the Earth at a rate of once per below. day, keeping it roughly in the same area over the ground. a) 1 only 2. A Geostationary Orbit (GSO) is a b) 2 only geosynchronous orbit with an inclination of c) Both 1 and 2 zero, meaning, it lies on the equator. which of the following statements d) Neither 1 nor 2 is/areincorrect?

Answer: c a) 1 only

Explanation: b) 2 only

 The Basel Convention aims to protect c) Both 1 and 2 the environment by bringing measures to control and regulate hazardous and d) Neither 1 nor 2 other waste disposals. The negotiations Answer: d

Explanation: by various exogenous agencies and deposited.  A Geosynchonous Orbit (GEO) takes a  These deposits through compaction satellite around the Earth at a rate of turn into rocks. This process is called once per day, keeping it roughly in the lithification. same area over the ground.  In several sedimentary rocks, the layers  A Geostationary Orbit (GSO) is a of deposits maintain their geosynchronous orbit with an characteristics even after lithification. inclination of zero, meaning, it lies on the equator.  Sandstone, shale are some of the  All geostationary satellites are examples for Sedimentary Rocks. geosynchronous. Not all  Mudstone, shale, geosynchronous satellites are and limestone are examples of geostationary. sedimentary rock likely to contain fossils. As the layers of sediment build Source: ISRO website up on top of one another, they create a physical timeline. The oldest layers,

along with the organisms that were Question 6 fossilized as they formed, are deepest.

Consider the following statement with Source: GC Leong reference to sedimentary rock

1. Sedimentary Rocks developed through Current affairs lithification process.

2. Sandstone, shale are some of the examples for Sedimentary Rocks. Question 7 Consider the following statement with 3. Sedimentary rocks that contain fossils. reference to Code on Social Security, 2020 Select the correct answer using the code 1. It applicable specifically toorganized given below: sectors employees. a) 1 and 2 only 2. It subsumed- the Employee’s Compensation Act, 1923; The Employee’s b) 1 and 3 only State Insurance Act, 1948; The Maternity c) 2 and 3 only Benefit Act, 1961 and The Unorganized Workers’ Social Security Act, 2008 d) 1, 2 and 3

Answer: d Select the correct answer using the code Explanation: given below.

 Rocks of the earth’s surface area a) 1 only exposed to denudation agents and are broken up into various sizes of b) 2 only fragments. These fragments are carried

c) Both 1 and 2 Select the correct answer using the code given below. d) Neither 1 nor 2 a) 1 only Answer: b b) 2 only Explanation: c) Both 1 and 2 Code on Social Security, 2020 d) Neither 1 nor 2  An Act to amend and consolidate the laws relating to social security with Answer: c the goal to extend social security to Explanation: all employees and workers either in the organised or unorganised or any  It is a statutory multi-disciplinary other sectors. body, established by the Government  Subsumed- the Employee’s of India under the Ministry of Compensation Act, 1923; The Environment and Forests, to combat Employee’s State Insurance Act, organized wildlife crime in the country. 1948; The Maternity Benefit Act,  The Bureau has its headquarters in 1961; The Unorganised Workers’ New Delhi. Social Security Act, 2008 etc.  Platform work: a work arrangement  It also assists and advises the customs authorities in inspection of the outside of a traditional consignments of flora & fauna as per employer-employee relationship in the provisions of the Wild Life which organisations or individuals use Protection Act, Convention on an online platform to access other International Trade in Endangered organisations or individuals. Species of Wild Fauna and Flora (CITES) and Export Import (EXIM) Source: The Hindu Policy governing such an item.

 It combats the organized wildlife crime in the country.  Section 38(Z) of the WPA act of 1972. Question 8 Source: https://www.worldwildlife.org/ and Consider the following statement with The Hindu reference to Wildlife Crime Control Bureau (WCCB) Question 9 1. It is a statutory multi-disciplinary body. Consider the following statement: 2. It combats the organized wildlife crime in 1. Project Elephant is Centrally Sponsored the country. Scheme.

2. The Ministry of Environment, Forest and Which of the pairs given above are correctly Climate Change provides the financial and matched? technical support to major elephant range states in the country through the project a) 1 only Elephant. b) 1 and 2 only Select the correct answer using the code given below. c) 3 only a) 1 only d) 1 and 3 only b) 2 only Answer: c c) Both 1 and 2 Explanation: d) Neither 1 nor 2  Baitam elephant reserve located in Odisha Answer: c  Intanki elephant reserve located at Explanation: Nagaland  elephant reserve located at  Project Elephant: It is a centrally Kerala. sponsored scheme and was launched in February 1992 for the protection of Source: The Hindu elephants, their habitats and corridors.

 The Ministry of Environment, Forest and Climate Change provides the financial and technical support to major elephant range states in the FEB 22 country through the project. Source: The Hindu Question 1 Consider the following statement with reference to august offer during freedom

movement 1. Dominion status as the objective for India.

Question 10 2. The recognition of Indiansright to frame their own constitution solely by Indians Consider the following pairs. 3. The Congress accepted the August Offer Elephant Reserve State Select the correct answer using the code 1. Anamudi Tamil Nadu given below:

2. Baitami Gujarat a) 1 only

3. Intanki Nagaland b) 1 and 3 only

c) 2 and 3 only d) Bicameralism d) 1, 2 and 3 Answer: d Answer: a Explanation: Explanation: In a federation system, there are two seats of  On 8 August 1940, early in the Battle power that are autonomous in their own of Britain, the Viceroy of India, Lord spheres. Linlithgow, made the so-called August Offer. Features of the Federal System of India  A fresh proposal promising  Dual government polity the expansion of the Executive  Division of powers between various Council to include more Indians, levels  Dominion status as the objective for  Rigidity of constitution India.  Independence judiciary  The establishment of an Advisory War Council, to give the Indians a majority  Dual citizenship of 8 out of 12 for the first time, but the  Bicameralism whites remained in charge of defence, finance and home. Unitary features of the constitution  The recognition of Indians’ right to frame their own constitution after  The flexibility of the constitution war mainly by Indians  Supremacy of parliament  Integrated judiciary  The Congress rejected the August  Single citizenship Offer.  All India services  The Congress Working  Emergency powers Committee meeting at Wardha on 21st August 1940 rejected this offer, and Source: Polity Laxmikanth asserted its demand for complete freedom from the imperial power Question 3

Source: Spectrum (Modern India) With reference to Balance of Payments, which of the following is not a component of current account?

Question 2 a) Transfers

Which of the following Features of the b) External commercial borrowing Federal System of India? c) Grants in aid a) The flexibility of the constitution d) Dividend b)Supremacy of parliament over constitution Answer: b c) Integrated judiciary Explanation:

Balance of Payments: d) Neither 1 nor 2

Transfers, Grants in aid, Dividend are Answer: c components of current account. Explanation: External commercial borrowing is a component of capital account.  It is a specialized body set up under the National Green Tribunal Act (2010) for effective and expeditious disposal of cases relating to environmental protection and conservation of forests and other natural resources.

 The Tribunal comprises of the Chairperson, the Judicial Members and Expert Members. They shall hold office for term of five years and are not eligible for reappointment.  The Chairperson is appointed by the Central Government in consultation with Chief Justice of India (CJI). Source: NCERT  A Selection Committee shall be formed by central government to appoint the Judicial Members and Expert Members. Question 4 Consider the following statement with  The Tribunal has jurisdiction over all reference National Green Tribunal (NGT) civil cases involving substantial question relating to environment 1. The Chairperson is appointed by the (including enforcement of any legal Central Government in consultation with right relating to environment). Chief Justice of India (CJI).  Being a statutory adjudicatory body 2. The Tribunal is not bound by the like Courts, apart from original procedure laid down under the Code of Civil jurisdiction side on filing of an Procedure 1908, but shall be guided by application, NGT also has appellate principles of 'natural justice'. jurisdiction to hear appeal as a Court (Tribunal). Select the correct answer using the code  The Tribunal is not bound by the given below. procedure laid down under the Code of Civil Procedure 1908, but a) 1 only shall be guided by principles of 'natural justice'. b) 2 only Source: https://greentribunal.gov.in/ c) Both 1 and 2

Question 5  India is not a member of the Nuclear Suppliers Group. The main reason for Consider the following statement with this is said to be that India is not a party reference to Nuclear Suppliers Group to the 1968 Treaty on the Non-Proliferation of Nuclear Weapons. 1. It is a Group of nuclear supplier countries It was in May 2016 that India formally that seek to prevent nuclear proliferation applied for the NSG membership but was denied the membership as a joint 2. India is a member of the Nuclear Suppliers decision by the other countries. Group Source: Select the incorrect answer using the code https://www.nuclearsuppliersgroup.org/en/ given below. a) 1 only Question 6 b) 2 only Consider the following statement regarding c) Both 1 and 2 black soils: d) Neither 1 nor 2 1. These soils are also known as the ‘Regur Soil’ Answer: b 2. The black soil retains the moisture for a very Explanation: long time

Nuclear Supply Group 3. They contains phosphorous, nitrogen and organic matter  Brought in 1974– in response to the Indian nuclear test (smiling Buddha). Select the correct answer using the code given  It is a multilateral export control below: regime.  It is a Group of nuclear supplier a) 1 and 2 only countries that seek to prevent nuclear proliferation by controlling the export b) 1 and 3 only of materials, equipment and technology that can be used to manufacture nuclear c) 2 and 3 only weapons.  The NSG first met in November 1975 d) 1, 2 and 3 in London, and is thus popularly referred to as the “London Club”. Answer: a  It is not a formal organization, and its guidelines are not binding. Decisions, Explanation: including on membership, are made by consensus.  Black soil covers most of the Deccan  Membership: 48 supplier states. Plateau which includes parts of Maharashtra, Madhya Pradesh, Gujarat,

Andhra Pradesh and some parts of b) 2 only Tamil Nadu.  In the upper reaches of the Godavari c) Both 1 and 2 and the Krishna, and the north western part of the Deccan Plateau, the black d) Neither 1 nor 2 soil is very deep.  These soils are also known as the Answer: d ‘Regur Soil’ or the ‘Black Cotton Soil’. The black soils are generally Explanation: clayey, deep and impermeable. They swell and become sticky when wet and  Prabuddha Bharata or Awakened India is shrink when dried. So, during the dry an english-language monthly journal of the season, these soils develop wide Ramakrishna Mission. cracks. Thus, there occurs a kind of  Prabuddha Bharata was founded in 1896 ‘self ploughing’. by P. Aiyasami, B. R. Rajam Iyer, G. G.  Because of this character of slow Narasimhacharya, and B. V. Kamesvara absorption and loss of moisture, the Iyer, in Madras (now Chennai), at the black soil retains the moisture for a behest of Swami Vivekananda. very long time, which helps the crops,  It is the "longest-running" monthly especially; the rain fed ones, to sustain English magazine of the country. even during the dry season.  It carries articles on social sciences and  Chemically, the black soils are rich humanities comprising historical, in lime, iron, magnesia and alumina. psychological, cultural and social sciences They also contain potash. But they themes. lack in phosphorous, nitrogen and  Luminaries like Netaji Subhas Chandra organic matter. The color of the soil Bose, Bal Gangadhar Tilak, Sister ranges from deep black to grey. Nivedita, Sri Aurobindo, Sarvepalli Radhakrishnan etc. wrote in the journal. Source: NCERT Source: The tribune

Current affairs Question 8 Question 7 Which of the following is/are G7 countries? Consider the following statement 1. Canada 1. Prabuddha Bharata is an english-language monthly 2. Russia

2. It is journal published by Ramakrishna 3. United Kingdom Mission. 4. Japan Select the incorrect answer using the code given below. 5. China a) 1 only

Select the correct answer using the code d) Neither 1 nor 2 given below. Answer: c a) 1, 2 and 3 only Explanation: b) 1, 3 and 4 only ASMI c) 1, 4 and 5 only  It is India’s first indigenous 9mm d) 1, 2, and 5 only Machine Pistol that has been jointly developed by DRDO and Indian Army. Answer: b  3D Printing process has been used in designing and prototyping of various Explanation: parts.

 G7 is a bloc of industrialized democracies Source:the Hindu Canada, France, Germany, Italy, Japan, the United Kingdom, and the United States.  Russia belonged to the forum from 1998 Question 10 through 2014, when the bloc was known as the Group of Eight (G8), but was Consider the following statement suspended following its annexation of 1. Exercise Kavach bilateral Air exercise Crimea between Indian Air Force and French Air and Space Force Source: current affairs 2. Desert Knight-21 is a joint Military exercise involving assets of Indian Army, Indian Navy, Indian Air Force and Indian Question 9 Coast Guard

Consider the following statement regarding Select the incorrect answer using the code ASMI given below.

1. It is India’s first indigenous 9mm Machine a) 1 only Pistol b) 2 only 2. It jointly developed by DRDO and Indian Army. c) Both 1 and 2

Select the correct answer using the code d) Neither 1 nor 2 given below. Answer: c a) 1 only Explanation: b) 2 only Exercise Kavach: It is a joint Military exercise c) Both 1 and 2 involving assets of Indian Army, Indian Navy, Indian Air Force and Indian Coast Guard.

Desert Knight-21: It is a bilateral Air  Mathura Style was indigenously exercise between Indian Air Force and French developed while Gandhara Style had Air and Space Force. strong Greek influence and it was based on Greco-Roman Norms (known as Source: current affairs Graeco-Buddhist School of Art).  In Mathura School material used was Spotted Red Sandstone while in FEB 23 Gandhara School, Blue-grey Mica schist / Grey Sandstone was used.

Source: Nitin Singhania Question 1

Consider the following statement regarding the Gandhara School of Art: Question 2 1. It is entirely indigenous and does not have Consider the following statement: any external influence 1. Swaran Singh Committee in 1976 2. It is patronage by Kushana Dynasty recommended Fundamental Duties. 3. Material used was red sandstone 2. 44th Amendment Act of 1976 added

Fundamental Duties to the Indian Constitution. Select the correct answer using the code given below: Select the correct answer using the code a) 1 only given below: b) 1 and 3 only a) 1 only c) 2 only b) 2 only d) 1, 2 and 3 c) Both 1 and 2 Answer: c d) Neither 1 nor 2 Explanation:  During the Kushana Empire’s rule, the Answer: a Gandhara art prospered in India. Above Explanation: all, Kanishka, the greatest of the Kushanas was a famous backer of art  42nd Amendment Act of 1976 added 10 and architecture. Fundamental Duties to the Indian  The Gandhara School of art flourished Constitution. 86th Amendment Act 2002 in his reign. Gandhara School was later added 11th Fundamental Duty to profoundly influenced by Greek the list. methodologies.  Swaran Singh Committee in 1976  It is patronage by Kushana Dynasty recommended Fundamental Duties, the  It has external influence Greek and necessity of which was felt during the possibly Macedonian influence. internal emergency of 1975-77.

 The Fundamental Duties are dealt with • Strengthening of the banking system of Article 51A under Part-IV A of the Indian the country. Constitution. Source: NCERT Source: Laxmikanth

Question 4 Question 3 Consider the following ststement Which of the following are the main objectives of the monetary policy 1. coral bleachingwill impact marine ecosystem, as coral reefs are some of the most 1. Regulation of monetary growth and bio diverse and productive ecosystems. maintenance of price stability 2. coral Reefs act as natural barriers to 2. Encourage flow of credit into priority and shorelines, protecting them from the effects of neglected sectors moving water. 3. Strengthening of the banking system of the 3. Without coral reefs, ocean will not be able to country absorb as much CO2, leaving more CO2 in atmosphere 4. Ensuring adequate expansion of credit Select the correct answer using the code given below: Select the correct answer using the code a) 1 only given below: a) 1 and 2 only b) 1 and 3 only b) 3 and 4 only c) 2 only c) 2 and 4 only d) 1, 2 and 3 d) 1, 2,3 and 4 Answer: d Answer: d Explanation: Explanation: Coral reefs are one of the most biologically The government through the reserve bank of diverse marine ecosystems on the Earth. India employs the monetary policy as an Coral bleaching occurs when coral polyps expel instrument of achieving the objectives of algae that live inside their tissues. general economic policy. The main objectives of the monetary policy are as follows: Consequences of Coral Bleaching: • Regulation of monetary growth and  It will impact marine ecosystem, as maintenance of price stability. coral reefs are some of the most bio • Ensuring adequate expansion of credit. diverse and productive ecosystems. • Assist economic growth.  Reefs act as natural barriers to • Encourage flow of credit into priority shorelines, protecting them from the and neglected sectors. effects of moving water. As coral reefs die, coastlines become more susceptible

to damage and flooding from storms, living beings. Cleaving of the DNA includes hurricanes, and cyclones. editing of genes (cut paste of the DNA)  Without coral reefs, ocean will not be Source: NCERT able to absorb as much CO2, leaving

more CO2 in atmosphere. Question 6  Loss of the coral reefs will have a devastating impact on tropical Consider the following statement: countries’ economies, food supplies, and safety of their coastal communities. 1. The rotation of the earth about its axis affects the direction of the wind and this force is called Source: The Hindu the Coriolis force.

2. It is inversly proportional to the angle of Question 5 latitude. Consider the following statement 3. The force acts parallel to the pressure 1. Genome editing will lead to change in gradient force. genome sequence of the organism.

2. Genetic editing allows genetic material to be Select the correct answer using the code added, removed, or altered at particular given below: locations in the genome. a) 1 only Select the correct answer using the code b) 1 and 3 only given below: c) 2 only a) 1 only d) 1, 2 and 3 b) 2 only Answer: a c) Both 1 and 2 Explanation: d) Neither 1 nor 2  The rotation of the earth about its axis Answer: c affects the direction of the wind and this force is called the Coriolis force. Explanation:  It is directly proportional to the angle of Genome editing is a technology that give latitude. scientists the ability to change an organism's  It deflects the wind to the left direction in DNA. the southern hemisphere and the right direction in the northern hemisphere. • This allows genetic material to be added, removed, or altered at particular locations in the  The deflection is more when the wind genome. velocity is high.  It is maximum at the poles and is absent at • It is a three-stage complex mechanism of the equator. unwinding, cleaving and rewinding of DNA to bring desirable changes in the genome of any  The force acts perpendicular to the pressure gradient force.

 The pressure gradient force is  National Pension System (NPS) is a perpendicular to an isobar. government-sponsored pension  The higher the pressure gradient force, the scheme. It was launched in January more is the speed of the wind and the 2004 for government employees. larger is the deflection in the direction of However, in 2009, it was opened to wind happens. all sections.  As a result of these two forces functioning perpendicular to each other, in the  Any Indian citizen between 18 and 60 low-pressure areas the wind blows around years can join NPS. The only it. condition is that the person must  The Coriolis force is zero at the equator comply with know your customer and the wind blows perpendicular to the (KYC) norms. isobars.  NRI can join NPS. However, the account will be closed if there is a Source: NCERT change in the citizenship status of the NRI.

Source: The Hindu Current affairs

Question 7 Question 8 Consider the following statement with Consider the following statements with refernce to National Pension Scheme (NPS); reference to Bio-Compressed Natural Gas (Bio-CNG); 1. It applicable only for government employees 1. It s calorific value is higher than biogas.

2. Non residents of India not allowed to 2. It is more environment-friendly fuel than join in NPS biogas Select the correct answer using the code Select the correct answer using the code given below: given below: a) 1 only a) 1 only b) 2 only b) 2 only c) Both 1 and 2 c) Both 1 and 2 d) Neither 1 nor 2 d) Neither 1 nor 2

Answer: d Answer: c Explanation: Explanation:

 Bio-CNG 9 Contains about 92-98% established in other places because of of methane and 2-8% CO2. pet releases, and have become  Calorific value is 167% - higher than an invasive species in many areas biogas. where they outcompete native species.  Ideal fuel for automobiles and power  The red-eared slider is included in generation - high methane content and the list of the world's 100 most invasive calorific value, low quantity of species published by the International moisture, hydrogen sulphide and Union for the Conservation of Nature. impurities. Source: The Hindu  More environment-friendly fuel than biogas - Low emission level.

Source: The Hindu Question 10

Consider the following statement regarding Mission Bhagiratha: Question 9 1. Flagship programme of Andhra Pradesh Consider the statement with reference to Red-eared slider turtle: 2. Ensures safe and sustainable piped drinking water supply from surface water sources to all 1. It is native to the south Asia households. 2. In other places they become an invasive Select the correct answer using the code species in many areas where they outcompete given below: native species. a) 1 only Select the correct answer using the code given below: b) 2 only a) 1 only c) Both 1 and 2 b) 2 only d) Neither 1 nor 2 c) Both 1 and 2 Answer: b d) Neither 1 nor 2 Explanation:

Mission Bhagiratha:

Answer: b  Flagship programme of Telangana government. Explanation:  Aimed at providing safe drinking water  Red-eared sliders are native to to every household. the Southern United States and northern Mexico, but have become

 Ensures safe and sustainable piped Source: NCERT drinking water supply from surface

water sources to all households. Question 2 Source: The Hindu Consider the following statement; FEB 24 1. Parliamentary privileges are certain rights Question 1 and immunities enjoyed by members of Consider the following statement Parliament, individually and collectively.

1. The construction of Qutb Minar was 2. The parliament of India enacted a law to originally planned by Aibak but it was codify the privileges. completed by Iltutmish. Select the correct answer using the code 2. Double dome architecture introduced during given below: Lodhi dynasty. a) 1 only Select the correct answer using the code b) 2 only given below: c) Both 1 and 2 a) 1 only d) Neither 1 nor 2 b) 2 only Answer: a c) Both 1 and 2 Explanation: d) Neither 1 nor 2  Parliamentary privileges are certain Answer: c rights and immunities enjoyed by Explanation: members of Parliament, individually and collectively, so that they can  The construction of Qutb Minar was “effectively discharge their functions”. originally planned by Aibak but it was  Parliamentary privileges are completed by Iltutmish. The planning of mentioned in Article 105 of the Indian Qutb Minar was purely Islamic as it was Constitution and those of State originally intended to serve as a place for legislatures in Article 194. the muazzin to call Muslims to prayer  When any of these rights and though, afterwards, it became famous as a immunities is disregarded, the offence is tower of victory. called a breach of privilege and is  The double dome is one of the classic punishable under law of Parliament. features of the Indo-Islamic architecture  According to the Constitution, the which flourished during the 16th and 17th powers, privileges and immunities of centuries. Sikander Lodi introduced the Parliament and MP's are to be defined double dome. by Parliament. No law has so far been

enacted in this respect. In the absence of  Deposit insurance facility of Deposit any such law, it continues to be Insurance and Credit Guarantee governed by British Parliamentary Corporation is not available to conventions. depositors of NBFCs. Source:PSRIndia.org Source: Mrunal Economy

Question 3 Question 4

With reference to non-banking financial With reference to Lichens, which of the companies, which of the following following statements is/are correct? statements is/are correct? 1. Lichens are symbiotic associations of 1. NBFC can accept demand deposits. fungi and algae

2. NBFCs do not form part of the payment 2. Lichens grows rapidly compare to algae and and settlement system and cannot issue fungi cheques drawn on it. 3. Deposit insurance facility of Deposit 3. They can live in extreme condition. Insurance and Credit Guarantee Corporation is available to depositors of NBFCs. Select the correct answer using the code given below: Select the correct answer using the code given below: a) 1 only a) 1 only b) 1 and 2 only b) 1 and 2 only c) 2 only c) 2 only d) 1and 3 d) 1, 2 and 3 Answer: d

Answer: c Explanation:

Explanation:  Lichen is not a single organism but a symbiosis among different organisms  A Non-Banking Financial Company like fungus and a cyanobacterium (NBFC) is a company registered under or algae. the Companies Act, 1956.  Mostly, lichens grow slowly and they  NBFC cannot accept demand can live exterme condition. deposits. Source: NCERT  NBFCs do not form part of the payment and settlement system and cannot issue cheques drawn on it. Question 5

Consider the following statement: o Agni: Ballistic missiles with different ranges, i.e. Agni 1. NAG is a anti tank missile. (1,2,3,4,5)

2. Trishul is a short range low level surface to o Trishul: Short range low air missile. level surface to air missile. o Nag: 3rd generation anti-tank 3. Agni is ballistic missiles with different missile. ranges. o Akash: Medium range surface to 4. Akash is a medium range surface to air air missile. missile.

5. Prithvi is a Short range surface to surface ballistic missile. Source: DRDO website

Select the correct answer using the code Question 6 given below: With reference to peninsular drainage, a) 1 and 2 only which of the following statements is/are correct? b) 1, 3 and 5 only 1. Peninsula rivers are much older than the c) 3, 4 and 5 only Himalayan Rivers d) 1, 2, 3, 4, 5 2. They are non-perennial rivers and depends on rain water Answer: d 3. The peninsular rivers have reached mature Explanation: stage IGMDP (Integrated Guided Missile Development Program): Select the correct answer using the code given below:  It was conceived by Dr. A.P.J. Abdul Kalam to enable India attain a) 1 only self-sufficiency in the field of missile b) 1 and 2 only technology. It was approved by the Government of India in 1983 and c) 2 only completed in March 2012.  The 5 missiles d) 1, 2 and 3 (P-A-T-N-A) developed under this program are: Answer: d

Explanation: o Prithvi: Short range surface to surface ballistic missile.  Peninsula Rivers are much older than the Himalayan rivers {Discordant}.

 The peninsular drainage is mainly c) Both 1 and 2 Concordant except for few rivers in the d) Neither 1 nor 2 upper peninsular region.  They are non-perennial rivers with a Answer: b maximum discharge in the rainy season. Explanation:  The peninsular rivers have reached mature stage {Fluvial Landforms} and have  Law Commission of India is almost reached their base level. [Vertical an executive body established by an down cutting is negligible]. order of the Government of India.  The rivers are characterized by broad and  Its major function is to work for legal shallow valleys. reform.  The river banks have gentle slopes except  Its membership primarily comprises for a limited tract where faulting forms legal experts, who are entrusted a steep sides. mandate by the Government.  The main water divide in peninsular rivers  The commission is established for a is formed by the , which fixed tenure and works as an advisory run from north to south close to the body to the Ministry of Law and western coast. Justice.  The velocity of water in the rivers and Source: https://lawcommissionofindia.nic.in/ the load carrying capacity of the streams is and The Hindu low due to low gradient.  Most of the major rivers of the peninsula such as the Mahanadi, the Godavari, the Krishna and the Cauvery flow eastwards Question 8 and drain into the Bay of Bengal. These With reference to Chief Information rivers make deltas at their mouths. Commissioner (CIC), which of the following statements is/are correct? Source: NCERT 1. The CIC will hold office for a term of three

years only. Current affairs 2. Salaries, Allowances, and other Terms and

Conditions of service will be determined by the Question 7 parliament.

Consider the following statement; Select the correct answer using the code 1. Law Commission of India is statutory given below: body. a) 1 only 2. Law Commission works as an advisory b) 2 only body to the Ministry of Law and Justice. c) Both 1 and 2 Select the correct answer using the code given below: d) Neither 1 nor 2 a) 1 only Answer: a b) 2 only Explanation:

 The CIC and ICs (at the central and Source: The Hindu state level) will hold office for a term of three years. Question 10  Tenure - notified by the central government. Consider the following statement  Salaries, Allowances, and other Terms regarding Joint Comprehensive Plan of Action and Conditions of service will be determined by the central government. 1. It is agreement between United Nation and  The Chief Information Commission Iran. (CIC) is the authorized body in India to 2. The objective of this agreement is act upon complaints received from nuclear non-proliferation in Iran. individuals who have been unable to Select the correct answer using the code submit requests of information to a given below: Central or State Public Information a) 1 only Officer due to either the officer not having been appointed, or the b) 2 only respective officer refused to entertain c) Both 1 and 2 the application under the Right to d) Neither 1 nor 2 Information Act (RTI Act) Answer: b Source: The Hindu Explanation:

Joint Comprehensive Plan of Action Question 9 (JCPOA): Global innovation Index published by • Also known as Iran Nuclear Deal. a) World economic forum and World Bank • Signed by the P5 countries (the US, UK, France, China and Russia), b) United Nations Educational, Scientific Germany, the European Union and and Cultural Organization and Columbia Iran. University • Objective: Nuclear non-proliferation in Iran - cutting its stockpile of c) United Nation Environment programme enriched Uranium. and World Bank d) World Intellectual Property Organization Source: The Hindu (WIPO), Cornell University and INSEAD Business School

Answer: d FEB 25 Explanation: Question 1 • Global Innovation Index (GII) list, 2020 released jointly by the World Consider the following statements: Intellectual Property Organization (WIPO), Cornell University and INSEAD Business School.

1. Chauth was one fourth of the land 5. Contempt of constitutional institution revenue paid to the Marathas in order to avoid the Maratha raids. 2. Sardeshmukhi was an additional levy of Select the correct answer using the code given ten percent on those lands which the below: Marathas claimed hereditary rights. a) 1, 2 and 3 only Which of the statements given above b) 2, 3 and 5 only is/are correct? c) 1, 2 and 4 only d) 1, 2 and 5 only a) 1 only b) 2 only Answer: c c) Both 1 and 2 Explanation: d) Neither 1 nor 2 Article 19(1)(a): Freedom of speech and Answer: c expression, provides every citizen with the right to express one’s views, opinions, beliefs, Explanation: and convictions freely by word of mouth, writing, printing, picturing or in any other  Chauth and sardeshmukhi were the manner. taxes collected not in the Maratha kingdom but in the neighboring Article 19(2) confers the right on the State to territories of the Mughal empire or impose reasonable restrictions on the exercise Deccan sultanates of the freedom of speech and expression on the grounds of,  Chauth was one fourth of the land revenue paid to the Marathas in order to  Sovereignty and integrity of India, avoid the Maratha raids.  Security of the state,  Sardeshmukhi was an additional levy of ten percent on those lands which the  Friendly relations with foreign states, Marathas claimed hereditary rights.  Public order, decency or morality, Source: NCERT  Contempt of court, defamation, and incitement to an offence. Question 2 Source: Laxmikanth State can impose reasonable restrictions on the exercise of the freedom of speech and Question 3 expression on the following grounds. Which of the following is not included in 1. Sovereignty and integrity of India RBI quantitative measures of credit control 2. Security of the state 1. Open Market Operations 3. Protest against legislative laws 2. Cash Reserve Ratio 4. Public order, decency or morality 3. Consumer Credit Regulation 4. Bank Rate Policy

Select the correct answer using the code  Guidelines: RBI issues oral, written given below: statements, appeals, guidelines, warnings etc. to the banks. a) 1 and 3 only  Rationing of credit: The RBI controls b) 2 and 4 only the Credit granted / allocated by c) 3 only commercial banks. d) 1, 2 and 4  Moral Suasion: psychological means Answer: c and informal means of selective credit control. Explanation:  Direct Action: This step is taken by the RBI against banks that don’t fulfill The quantitative measures of credit control are: conditions and requirements. RBI may  Bank Rate Policy: The bank rate is the refuse to rediscount their papers or may Official interest rate at which RBI give excess credits or charge a penal rate rediscounts the approved bills held by of interest over and above the Bank rate, commercial banks. For controlling the for credit demanded beyond a limit. credit, inflation and money supply,  Open Market Operations: OMO The Open market Operations refer to direct Source: Mrunal economy sales and purchase of securities and bills

in the open market by Reserve bank of India. The aim is to control volume of Question 4 credit.  Cash Reserve Ratio: Cash reserve ratio In the context of which one of the following refers to that portion of total deposits in are the terms Incineration and commercial Bank which it has to keep Phytoremediation mentioned? with RBI as cash reserves. a) Extraction of rare earth metals  Statutory Liquidity Ratio: It refers to that portion of deposits with the banks b) Waste management which it has to keep with itself as liquid c) Hydrogen fuel based automobiles assets (Gold, approved govt. securities d) Extraction of precious metals etc.). Answer: b Qualitative measures: Explanation:  Margin requirements: This refers to Waste management process: difference between the securities offered and and amount borrowed by the banks.  Incineration  Consumer Credit Regulation: This This is a process where wastes and other refers to issuing rules regarding down unwanted substances are burnt. During payments and maximum maturities of combustion, the organic waste turns into ash, installment credit for purchase of goods. flue gas, and heat. The inorganic constituents of the waste remain in the form of an ash. It is also termed as thermal treatment.

 Phytoremediation immune system can recognize the antigen later in life. Many vaccines In this scenario, plants are directly used to clean up or contain contaminants in the soil. contain a single antigen that the body This method of bioremediation will help will recognize. mitigate the environmental problem without  However, the antigen of some bacteria the need to excavate the contaminant material does not elicit a strong response from and dispose of it elsewhere. the immune system, so a vaccination against this weak antigen would not protect the person later in life Source: current affairs Source: NCBI Question 5 Consider the following statement with Question 6 reference to conjugate vaccine: Which one of the following flowing rivers of 1. Conjugate vaccine is combination of weak India not following through has rift valley? and strong antigen 1. Narmada 2. This enhances the stability and the 2. Damodar effectiveness of the vaccine. 3. Luni Which of the statements given above 4. Gandak is/are correct? Select the correct answer using the code a) 1 only given below: b) 2 only a) 1 and 2 only c) Both 1 and 2 b) 2 and 3 only d) Neither 1 nor 2 c) 3 and 4 only d) 1, 2 and 4 Answer: c

Explanation: Answer: c Explanation:  A conjugate vaccine is a substance that is composed of a polysaccharide A rift valley is a linear shaped lowland antigen fused (conjugated) to a carrier between several highlands or mountain ranges molecule. This enhances the stability created by the action of a geologic rift or fault. and the effectiveness of the vaccine. A rift valley is formed on a divergent plate  Vaccines are used to prevent diseases boundary, a crustal extension or spreading by invoking an immune response to an apart of the surface, which is subsequently antigen (the foreign part of a bacteria or further deepened by the forces of erosion. virus that the immune system recognizes) Rift valley flowing rivers in India  This is usually accomplished with an  Narmada river, attenuated or dead version of a bacteria  Damodar river, or virus in the vaccine, so that the

 Mahi river and  Lyme disease  Source: The Hindu Source: NCERT

Question 8 Current affairs Saffron revolution recently in media, is Question 7 related to Which among the following diseases is/are a a) Protest against China in Hon Kong zoonotic disease(s)? b) Protest against elections results in United States of America 1. Salmonellosis c) protests against Myanmar's military 2. Lyme disease. d) protests against Taiwan military 3. Brucellosis. 4. Rabies Answer: c

Select the correct answer using the code Explanation: given below:  The phrase "Saffron Revolution" a) 1 and 2 only connects the protests against b) 2 and 3 only Myanmar's military dictatorship to the c) 3 and 4 only saffron-colored robes which are widely d) 1, 2, 3 and 4 associated with Buddhist monks, who were at the forefront of the Answer: d demonstrations. The robes of Burmese monks are similar to the color of whole Explanation: saffron.  The goal of the revolution is to A zoonotic diseases is an establish democracy, free election etc. infectious disease caused by a pathogen (an infectious agent, such as a bacterium, virus, Source: The Hindu parasite or prion) that has jumped from an animal (usually a vertebrate) to a human. Question 9 Consider the following statement with  Zoonotic influenza reference to Pradhan Mantri JI-VAN;  Salmonellosis  West Nile virus 1. This scheme is promoting Second  Plague Generation Biofuels Technology  Emerging coronaviruses 2. The ethanol produced by the scheme (e.g., severe acute respiratory beneficiaries will be mandatorily syndrome, Middle East respiratory supplied to Oil Marketing Companies syndrome) (OMCs) to further enhance the  Rabies blending percentage under Ethanol  Brucellosis Blending Programme (EBP).

Which of the statements given above  Centre for High Technology (CHT), a is/are correct? technical body under the aegis of MoP&NG, will be the a) 1 only implementation Agency for the b) 2 only scheme. c) Both 1 and 2 d) Neither 1 nor 2 Source: The Hindu Answer: c

Explanation: Question 10 Pradhan Mantri JI-VAN (Jaiv Indhan- Vatavaran Anukool fasal awashesh Nivaran) Consider the following statements with Yojana. reference to National Commission of Women:  The scheme will be supported financially by Viability Gap Funding  It review the constitutional and legal (VGF) to Second Generation (2G) safeguards for women. Integrated Bioethanol Projects using lignocellulosic biomass and other  Recommendation made by the National renewable feedstock. Commission of Women are binding on the government  The objective of the scheme is to create an ecosystem for setting up Which of the statements given above commercial projects and boost is/are correct? to Research and Development in 2G Ethanol sector. a) 1 only  Under the scheme funds have been b) 2 only allocated for supporting 12 c) Both 1 and 2 Commercial projects, 10 d) Neither 1 nor 2 Demonstration Projects: Phase-I (2018-19 to 2022-23) 6 Answer: a Commercial and 5 demonstration projects will be Explanation: supported. National Commission for Women: Phase-II (2018-19 to 2022-23) Remaining 6 It was set up as a statutory body in January Commercial and 5 1992 under the National Commission for demonstration projects will Women Act, 1990. be supported. Its mission is to strive towards enabling  The ethanol produced by the scheme women to achieve equality and equal beneficiaries will be mandatorily participation in all spheres of life by securing supplied to Oil Marketing Companies her due rights and entitlements through (OMCs) to further enhance the suitable policy formulation, legislative blending percentage under Ethanol measures, etc. Blending Programme (EBP).

Its functions are to:  Each nadu was further divided into (Urs) villages which form part of the last unit of the administration.  Review the constitutional and legal safeguards for women.  Uttaramerur inscriptions speak about the  Recommend remedial legislative administration of the Cholas. measures.

 Facilitate redressal of grievances.  Advise the Government on all Source: TN NCERT policy matters affecting women. It only has the powers to suggest amendments and submit reports which are not obligatory on a state or Union Question 2 Governments. In the context of India, which one of the Source: The Hindu following statement is/are correct with reference to cabinet? FEB 26 1. The word cabinet mentioned in the Constitution. Question 1 2. It consist only senior minsters. With reference to the history of India, the Select the correct answer using the code given terms Kottam, Nadu and Urs denote? below: a) Urban land division a) 1 only b) Religious festivals c) Different types of coins b) 2 only d) Administrative division c) Both 1 and 2 Answer: d d) Neither 1 nor 2 Explanation:

Administrative Divisions during chola kingdom: Answer: a  The success of the Chola administration Explanation: depended more on the proper functioning of the administrative division us. Union cabinet

 Generally mandalams were named after  The Union Council of Ministers exercises the original names or the titles of the Chola executive authority in the Republic of kings. India.  Each mandalam was divided into number  It consists of senior ministers, called of Kottams or Valanadus. Each kottam was 'cabinet ministers', junior ministers, and sub divided into nadu. called 'ministers of state' and, rarely, deputy ministers.

 A smaller executive body called the debt. Internal debt is categorized into Union Cabinet is the supreme marketable and non-marketable securities. decision-making body in India.  The Union government broadly classifies its liabilities into two broad categories.  The word cabinet was not mentioned in The debt contracted against the the original constitution. Consolidated Fund of India is defined as  Article 352 public debt and includes all other funds only defines the cabinet saying that it is received outside Consolidated Fund of 'the council consisting of the prime India under Article 266 (2) of the minister and other ministers Constitution, where the government of cabinet rank appointed under Article merely acts as a banker or custodian. The second type of liabilities is called public 75' and does not describe its powers and account. functions  Internal debt constitutes more than 93% of the overall public debt Source: Dk Basu

Source: RBI Annual report Question 3

Consider the following statement: Question 4

1. Public debt is the total liabilities of the With reference to India’s Agasthyamala central government contracted against the Biosphere Reserve, which of the following Public Accounts of India. statements is/are correct? 2. Indian internal debt constitutes high 1. Agastyamalai is also home to the percentage in public debt. Kanikaran, one of the oldest surviving Which of the statements given above ancient tribes in the world. is/are correct? 2. It spread over two southern states Kerala and Tamil Nadu. a) 1 only 3. It is also home to rare endemic animals b) 2 only include tiger, Asian Elephant, and Nilgiri Tahr. c) Both 1 and 2 d) Neither 1 nor 2 Select the correct answer using the code given below: Answer: b a) 1 only Explanation: b) 2 and 3 only  Public debt is the total liabilities of the c) 1 and 3 only central government contracted against d) 1, 2 and 3 the Consolidated Fund of India. It is further classified into internal & external Answer: d

Explanation:

Agasthyamala Biosphere Reserve: 2. It enables the simultaneous multi-wavelength observations of various  ABR in situated at the southern-most astronomical objects with a single satellite. end of the Western Ghats and spread over two southern states Kerala and Which of the statements given above Tamil Nadu. is/are correct?  It was established in 2001.  It is named after Mala peak that a) 1 only rises up to almost 1868 metres above sea level, in , Kerala. b) 2 only  In March 2016, it was included in the World Network of Biosphere Reserves c) Both 1 and 2 of UNESCO.  ABR covers an area of 3,500 sq km at an d) Neither 1 nor 2 altitude ranging from 100 metres to Answer: c 1,868 metres above the Mean Sea Level.  It covers Peppara and Shendurney Explanation: wildlife sanctuaries and parts of the sanctuary in Kerala and the  AstroSat is the first dedicated Indian Kalakad Mundanthurai Tiger Reserve of astronomy mission aimed at studying Tamil Nadu. celestial sources in X-ray, optical and  Its flora mostly consists of tropical forests and is home to 2,254 species of UV spectral bands simultaneously. The higher plants including about 400 that payloads cover the energy bands of are endemic. Ultraviolet (Near and Far), limited  About 400 Red Listed Plants, 125 optical and X-ray regime (0.3 keV to species of orchids and rare, endemic and 100keV). threatened plants have been recorded  One of the unique features of AstroSat from the reserve. mission is that it enables the  It is also home to rare endimic animals include tiger, Asian Elephant, and simultaneous multi-wavelength Nilgiri Tahr. observations of various astronomical  It is home to Kanikaran tribe, one of the objects with a single satellite. oldest surviving ancient tribes in the world. Source: ISRO website

Source: UNESCO site Question 6 Question 5 Which of the following factors responsible Consider the following statement with for avalanches reference to Astrosat 1. Heavy Snowfall 1. AstroSat is the first dedicated Indian 2. Earthquakes astronomy mission aimed at studying 3. Higher Temperatures celestial sources in X-ray, optical and UV 4. Wind direction spectral bands simultaneously.

Which of the statements given above Layering of Snow is/are correct? • The gradual snowfall creates layer by a) 1 and 2 only layer accumulation of snow that b) 2 and 3 only hypersensitive the snowpack. If c) 3 and 4 only something catastrophic events happen then these layers of snow falls down d) 1,2,3, and 4 that leads to avalanche. Answer: d Steeper Slopes Explanation: • An avalanche is also caused by the  Avalanche, a mass of material moving influence of gravity. If gradual rapidly down a slope. snowfalls accumulated on the slopes of the mountain then it prone to rush  An avalanche is typically triggered downs the slopes at greater speeds. when material on a slope breaks loose from its surroundings; this material then quickly collects and carries Higher Temperatures additional material down the slope. • Temperature is one of the important  There are various kinds of avalanches, factors for the avalanche because of including rock avalanches (which high temperature the surface layer of consist of large segments of shattered the snowpack gets melted. The rock), ice avalanches (which typically accumulated snow will become highly occur in the vicinity of a glacier), susceptible to sliding down. and debris avalanches (which contain a variety of unconsolidated materials, Earthquakes such as loose stones and soil). It is one of the important factors that Factors responsible for the Avalanche: • triggered the layer of accumulated snowpack because earthquakes Heavy Snowfall generate seismic waves that cause the ground to vibrate. • When a high rate of snowfall occurred leading to the snow accumulation on the mountain slopes triggered the weaker layer of snow in the snowpack Source: NCERT of unstable areas of the mountain causes Avalanche.

Wind Direction Current affairs

• The direction of the wind determines Question 7 the patterns of the snowfall as well snow accumulation on the mountain In the context of Indian Constitution, slopes. If the strong wind blows, then Consider the following statement with the upward direction of the winds reference to Union territories of India might trigger the steep slope which causes an avalanche.

 Union territories has federal • The president of India is the executive relationship with the union government head of the union territory.  The president of India is the executive • Union territory is appointed by an head of the union territory. administrator who is appointed by the president.

Source: The Hindu Which of the statements given above is/are correct? a) 1 only Question 8 b) 2 only Human capital index published by c) Both 1 and 2  United Nations Development Programme  Oxford economics d) Neither 1 nor 2  World Bank Answer: b  International Monetary Fund

Explanation: Answer: c

• A union territory is a type of  World Bank released the report administrative division in the Republic titled “The Human Capital Index 2020 of India. Unlike the states of India, Update: Human Capital in the Time of which have their own governments, COVID-19”. union territories are federal units  The Human Capital Index (HCI) 2020 governed directly by the union is a collaboration between the Human government. Development Practice Group and the • The States Reorganization Development Economics Group of the Commission, 1956 recommended World Bank. creation of a different category for  India has been ranked at the 116th territories which did not fit the model position among 174 countries in the of a state. Human Capital Index 2020. • A union territory is a small Source: The Hindu administrative unit that is ruled by the union. The union territories are administered and controlled directly by the Central Government of India. Question 9 • Union territories has unitary Which of the following is/are sources of relationship with the central particulate matter? government which means all the legislative and executive powers rest 1. Industrial process based on combustion with the Union. activities 2. Fuel combustion

3. Dust paved from roads Select the correct answer using the code given below: Select the correct answer using the code given below: a) 1 only b) 2 and 3 only a) 1 only c) 1 and 3 only b) 2 and 3 only d) 1, 2 and 3 c) 1 and 3 only d) 1, 2 and 3 Answer: b

Answer: d Explanation:

Explanation: National Commission for Scheduled Castes:

Sources of PM  In order to provide the Scheduled Castes of Indian society safeguards against  Sources of fine particles include all exploitation and to promote their social, types of combustion activities (motor economic, educational and cultural vehicles, power plants, wood burning, development, the Commission was set up etc.) and certain industrial processes. by the Government of India.  Sources of coarse particles include crushing or grinding operations, and  The National Commission for dust from paved or unpaved roads. Scheduled Tribes (NCST) was established by amending Article 338  Other particles may be formed in the air from the chemical change of gases. and inserting a new Article 338A in They are indirectly formed when gases the Constitution through the from burning fuels react with sunlight Constitution (89th Amendment) Act, and water vapor. These can result from 2003. fuel combustion in motor vehicles, at  By this amendment, the erstwhile power plants, and in other industrial National Commission for Scheduled processes. Castes and Scheduled Tribes was replaced by two separate Commissions Source: The Hindu namely- the National Commission for Scheduled Castes (NCSC), and the National Commission for Scheduled Tribes (NCST) from February, 2004. Question 10  It consists of a chairperson, a Consider the following statement with vice-chairperson and three other reference to National Commission for members. They are appointed by the Scheduled Castes: President by warrant under his hand and seal. 1. It is statutory body  Lack of infrastructure, lack of powers to 2. They are appointed by the President by take affirmative action (Commission’s warrant under his hand and seal. recommendations are not binding) have 3. It submits its annual report to the been some of the issues faced by the president NCSC. It has been criticized for inefficient functioning, vague selection

and appointment process, exorbitant With reference to the provisions contained in budget etc. part III of the constitution of India, which of  The National Commission for Scheduled the following statements is/are correct? Castes submits its annual report to the President of India 1. They are enforceable by courts 2. They cannot be restricted in an area which Source: The Hindu has been placed under martial law or military rule.

FEB 27 3. The rights laid down in this part are to influence the making of laws by the state Question 1 Select the correct answer using the code With reference to the cultural history of given below India, which one of the following is the correct description of the term Syadvada? a) 1 only b) 2 only a) Merchant guild during Gupta empire c) 1 and 3 only b) Buddhism philosophy that accepts d) 2 and 3 only authority of Vedas c) Jainism philosophy ‘method of examining Answer: c different probabilities d) The earliest text explains about dharma Explanation: sutras Features of Fundamental Rights Answer: c  Fundamental rights are different from Explanation: ordinary legal rights in the manner in which they are enforced. If a legal right is Major Jainism Philosophies violated, the aggrieved person cannot directly approach the SC bypassing the  Anekantavada: Emphasises that the lower courts. He or she should first ultimate truth and reality is complex, approach the lower courts. and has multiple-aspects i.e theory of  Some of the fundamental rights are plurality.It refers to the simultaneous available to all citizens while the rest are acceptance of multiple, diverse, even for all persons (citizens and foreigners). contradictory viewpoints.  Fundamental rights are not absolute rights.  Syadvada: All judgments are They have reasonable restrictions, which conditional, holding good only in means they are subject to the conditions of certain conditions, circumstances, or state security, public morality and decency senses. Syadavada literally means the and friendly relations with foreign ‘method of examining different probabilities’. countries.  They are justiciable, implying they are Source: NIOS enforceable by courts. People can approach the SC directly in case of Question 2 violation of fundamental rights.

 Fundamental rights can be amended by the Reserve Bank of India (RBI) at regular Parliament by a constitutional amendment intervals and issued at a discount to but only if the amendment does not alter face value. the basic structure of the Constitution.  Treasury Bills are basically instruments  Fundamental rights can be suspended for short term (maturities less than one during a national emergency. But, the year) borrowing by the Central rights guaranteed under Articles 20 and 21 Government. cannot be suspended.  Treasury Bills were first issued in India in 1917. At present, the active T-Bills  The application of fundamental rights are 91-days T-Bills, 182-day T-Bills can be restricted in an area which has and 364-days T-Bills. been placed under martial law or military rule.  The State governments do not issue any treasury bills. Source: Laxmikanth  Interest on the treasury bills is determined by market forces. Question 3 Source: RBI

In the context of the Indian economy, consider the following statement regarding treasury bills:

1. Treasury Bills are basically instruments Question 4 for short term borrowings with maturities Consider the following statement with less than one year 2. It is debt instrument borrowed by the both reference to Forest-PLUS 2.0 initiative Central Government and state government. 1. It is collaboration between US Agency for 3. Interest on the treasury bills is determined International Development (USAID) and by Reserve Bank of India India's Ministry of Environment, Forest Select the correct answer using the code and Climate Change given below: 2. It help India to achieve targets under REDD+ a) 1 only b) 2 only Which of the statements given above c) 1 and 3 only is/are correct? d) 2 and 3 only a) 1 only Answer: a b) 2 only Explanation: c) Both 1 and 2  Treasury Bills are short term (up to one year) borrowing instruments of the d) Neither 1 nor 2 Government of India or by a central Answer: c authority of any country which enable investors to park their short term Explanation: surplus funds while reducing their market risk. They are auctioned by the

 Forest-PLUS 2.0 is five year programme Source: current affairs that focuses on developing tools and techniques to bolster ecosystem management and harnessing ecosystem Question 6 services in forest landscape management.  It is initiated by US Agency for Consider the following statements with International Development (USAID) and reference to Jet streams: India's Ministry of Environment, Forest 1. Jet Streams develop where air masses of and Climate Change. differing temperatures meet  The Forest-PLUS focused on capacity 2. The Jet Stream is a geostrophic building to help India participate in wind blowing horizontally through the Reducing Emissions from Deforestation lower layers of the troposphere and forest Degradation (REDD+). Which of the statements given above Source: MoEF&CC website is/are correct?

a) 1 only Question 5 b) 2 only c) Both 1 and 2 Sycamore in media is related to d) Neither 1 nor 2

a) Block chain technology Answer: a b) Edge computing c) Super computer Explanation: d) Quantum computing  The Jet Stream is a geostrophic Answer: d wind blowing horizontally through the upper layers of the troposphere, Explanation: generally from west to east, at an altitude of 20,000 - 50,000 feet.  Quantum computing is the area of  Jet Streams develop where air masses study focused on developing computer of differing temperatures meet. So, technology based on the principles of usually surface temperatures quantum theory, determine where the Jet Stream will  Quantum Computers encode form. information as quantum bits, or qubits,  Greater the difference in temperature, which can exist in superposition faster is the wind velocity inside the  Google’s quantum computer, named jet stream. Sycamore, claimed ‘supremacy’  Jet Streams extend from 20 degrees because it reportedly did the task in 200 latitude to the poles in both seconds that would have apparently hemispheres. taken a supercomputer 10,000 years to The genesis of the Jet-streams is provided by complete. three kinds of gradients:

 Thermal gradient between pole and however, it also aims to encourage equator local companies to set up or expand existing manufacturing units.  Pressure gradient between pole and equator  The PLI scheme will be implemented by the concerned  Pressure gradient between surface and ministries/departments. subsurface air over the poles.  Savings from one PLI scheme of an Source: NCERT approved sector can be utilized to fund another sector.

Source: The Hindu Current affairs

Question 7 Question 8 Consider the following statement regarding Consider the following statement with production linked incentive scheme: reference to anti defection law:

1. A scheme that aims to give 1. Legislators may be disqualified on companies incentives on grounds of defection by the Presiding Officer of a legislature. incremental sales from products nd manufactured in domestic units. 2. It introduced through 52 constitutional amendment. 2. The scheme applicable only domestic Which of the statements given above companies only. is/are correct? Which of the statements given above a) 1 only is/are correct? b) 2 only a) 1 only c) Both 1 and 2 b) 2 only d) Neither 1 nor 2 c) Both 1 and 2 Answer: c d) Neither 1 nor 2 Explanation: Answer: a

Explanation: Anti-Defection Law The 10th Schedule of the Indian Constitution

(which talks about the anti-defection law) is Production Linked Scheme: designed to prevent political defections prompted by the lure of office or material benefits or other like considerations. The  A scheme that aims to give Anti-defection law was passed by Parliament in companies incentives on 1985 and reinforced in 2002. incremental sales from products manufactured in domestic units.  The 10th Schedule of the Indian  The scheme invites foreign Constitution popularly referred to as the companies to set units in India, ‘Anti-Defection Law’ was inserted by

the 52nd Amendment (1985) to the Kumbh Mela: Constitution.  The Kumbh Mela (the festival of the  ‘Defection’ has been defined as, “To sacred pitcher) is anchored in Hindu abandon a position or association, often mythology. to join an opposing group”.  It is the largest public gathering and  The anti-defection law was enacted to collective act of faith, anywhere in the ensure that a party member does not world. violate the mandate of the party and in  Crowds gather at the sacred confluence of case he does so, he will lose his the Ganga, the Yamuna, and the mystical membership of the House. The law Sarasvati. Primarily, this congregation applies to both Parliament and state includes Ascetics, Saints, Sadhus, Sadhvis, assemblies. Kalpvasis, and Pilgrims from all walks of life.  The Anti-Defection Law aims to  The Mela was included in the list of prevent MPs from switching political “Intangible Cultural Heritage of parties for any personal motive. Humanity” by UNESCO in 2017.  Legislators may be disqualified on grounds of defection by the Presiding Kumbh Mela is celebrated four times over a Officer of a legislature, based on a course of 12 years. petition by any other member of the house. The geographical location of Kumbh Mela spans over four locations in India and the Mela Source: The Hindu site keeps rotating between one of the four pilgrimage places on four sacred rivers as Question 9 listed below:

With reference to cultural history of India 1. Haridwar on the Ganges in Uttarakhand. consider the following statement 2. Ujjain on the Shipra in Madhya Pradesh. 3. Nashik on the Godavari in Maharashtra. 1. The Kumbh Mela was included in the list 4. Prayagraj at the confluence of the Ganges, of “Intangible Cultural Heritage of the Yamuna, and the mythical Sarasvati in Humanity” by UNESCO Uttar Pradesh. 2. Kumbh Mela is celebrated four times over a course of 12 years. Source: The Hindu

Which of the statements given above Question 10 is/are correct? a) 1 only Consider the following statement with reference to Indian National Centre for b) 2 only Ocean Information c) Both 1 and 2 d) Neither 1 nor 2 1. An autonomous under Ministry of forest and environment Answer: c 2. INCOIS with National Remote Sensing Centre (NRSC) is going to carryout Explanation:

bathymetric study for better understanding 2. The movement restricted to Bengal only. of ocean floor

Which of the statements given above Select the correct answer using the code given is/are correct? below. a) 1 only a) 1 only b) 2 only b) 2 only c) Both 1 and 2 c) Both 1 and 2 d) Neither 1 nor 2 d) Neither 1 nor 2

Answer: b Answer: a

Explanation: Explanation:

Indian National Centre for Ocean Swadeshi movement: Information Services  This was a comprehensive movement that  Autonomous Body under the Earth lasted 6 years. Sciences ministry.  Although this was regarded as a cultural  Unit of the Earth System Science movement celebrating the rich harvest of Organization (ESSO) -ESSO-INCOIS. history- the rich traditions like folk music,  Mandated to provide the best possible paintings, the culture of Bengal was highlighted. But very soon, it got integrated ocean information and advisory with the political upheavals that followed services through sustained ocean after the partition of Bengal. Krishna observations and through systematic Kumar Mitra, in his  INCOIS with National Remote Sensing newspaper, ‘Sanjivani’ highlighted the Centre (NRSC) is going to carryout prospects of national education and bathymetric study – for the aerial ‘Economic Swadeshi’. mapping of Andaman and Nicobar  Very soon, this movement started gaining Islands and Lakshwadeep to get a better momentum with the help of the entire picture of the ocean floor. Bengali middle-class intelligentsia.  After the partition of Bengal, the Swadeshi Source: The Hindu movement got a big boost, because of the integration of the boycott movement with FEB 28 the Swadeshi movement.

Question 1  Most of the Bengali intelligentsia that was initially not in favor of the boycott With reference to Swadeshi movement, movement got integrated with the consider the following statements: Swadeshi movement.  Very soon, the Swadeshi movement 1. Most of the Bengali intelligentsia that was spread to different parts of India, like initially not in favor of the boycott Bihar, U.P., C.P., Bombay and Madras. movement got integrated with the

Swadeshi movement.

Source: Spectrum  a minister cannot be elected as the member of the Estimates Committee. Question 2 Source: Laxmikanth

With reference to Estimates Committee, consider the following statements: Question 3 1. Members are taken from both lower and upper house Consider the following statement with 2. The principle of election used is reference to Aadhaar Enabled Payments Proportional Representation by the means System (AEPS): of a single transferable vote 1. AePS is developed by the National 3. Minster can be a member of the committee Payments Corporation of India (NPCI) Select the correct answer using the code 2. It allows people to carry out financial given below: transactions on a Micro-ATM 3. Funds can be transferred from one bank a) 1 only account to another simply through their b) 2 only Aadhaar numbers. c) 1 and 3 only d) 2 and 3 only Select the correct answer using the code given below: Answer: b a) 1 only Explanation: b) 1 and 3 only c) 2 and 3 only  Estimates Committee has 30 members. d) 1, 2 and 3 only All the members are taken from Lok Sabha (Lower House). That means that Answer: d there is no representation from Rajya Sabha (Upper House Explanation:  Lok Sabha elects members of Estimates Aadhaar enabled Payment System (AePS) Committee from amongst its members. The principle of election used is  AePS is developed by the National Proportional Representation by the Payments Corporation of India means of a single transferable vote. All (NPCI) that allows people to carry out parties are duly represented in the financial transactions on a estimates committee. Micro-ATM by furnishing just their Aadhaar number and verifying it with  Every member elected by the Lok the help of their fingerprint/iris scan. Sabha from amongst its members are a part of the committee for a year. After a  With the help of this payment system, year, a new election takes place and funds can be transferred from one bank account to another simply members are changed or re-elected through their Aadhaar numbers.

 This system adds another layer of security to financial transactions

as bank details would no longer be required to be furnished while Question 5 carrying out these transactions. Consider the following with reference Source: Current Affairs Raman Spectroscopy: 1. A non-destructive chemical analysis technique which provides detailed Question 4 information about chemical structure, Consider the following statement with phase and polymorphy, crystallinity and reference to Nature-based Solutions molecular interactions. initiative: 2. It is based upon the interaction of light 1. It is initiative of United Nation with the chemical bonds within a Environment Programme. material. 2. Nature-based Solutions included both natural and societal challenges. Select the correct answer using the code given below. Select the correct answer using the code given below. a) 1 only b) 2 only a) 1 only c) Both 1 and 2 b) 2 only d) Neither 1 nor 2 c) Both 1 and 2 d) Neither 1 nor 2 Answer: c

Answer: b Explanation:

Explanation: • Raman Spectroscopy is a non-destructive chemical analysis  Nature-based Solutions is initiated by technique which provides detailed IUCN information about chemical structure,  NbS are actions to protect, sustainably phase and polymorphy, crystallinity manage and restore natural or modified and molecular interactions. It is based ecosystems that address societal upon the interaction of light with the challenges effectively and adaptively, chemical bonds within a material. simultaneously providing human • It is a light scattering technique, wellbeing and biodiversity benefit. whereby a molecule scatters incident  They are underpinned by benefits that light from a high intensity laser light flow from healthy ecosystems and target major challenges like climate source. change, disaster risk reduction, food and water security, health and are Source: current affairs critical to economic development. Source: IUCN website Question 6

Which of the following Indian states are 2. Members are not eligible for immediate share border with Nepal: re-election after serving two consecutive terms. 1. Sikkim 2. Bihar Select the correct answer using the code 3. Himachal Pradesh given below. 4. Uttarakhand a) 1 only Which of the statements given above is/are correct? b) 2 only c) Both 1 and 2 a) 1 and 2 only d) Neither 1 nor 2 b) 2 and 3 only c) 1, 2 and 4 only Answer: c d) 1,2,3, and 4 Answer: c Explanation: Explanation:  The Human Rights Council is an inter-governmental body within the  Indian states that share borders with United Nations system responsible for Nepal are Uttarakhand, Uttar Pradesh, strengthening the promotion and Sikkim, West Bengal, and Bihar. protection of human rights around the world.  It is made up of 47 United Nations Member States which are elected by the UN General Assembly (UNGA)

 Members of the Council serve for a period of three years and are not eligible for immediate re-election after serving two consecutive terms. Source: The Hindu

Question 8 Source: Internet Consider the following statement Supply Current affairs Chain Resilience Initiative

1. It is trilateral initiative between Indian, Japan, and Australia Question 7 Consider the following statement with 2. The main objective is to reduce dependency reference to United Nation Human Right on China Council: Select the correct answer using the code 1. It replaced the former United Nations given below. Commission on Human Rights.

a) 1 only a) 1 only b) 2 only b) 2 only c) Both 1 and 2 c) Both 1 and 2 d) Neither 1 nor 2 d) Neither 1 nor 2

Answer: c Answer: c

Explanation: Explanation:

Supply Chain Resilience Initiative Inflation Targeting:  In the context of international trade, supply  It is a central banking policy that chain resilience is an approach that helps a revolves around adjusting monetary country to ensure that it has diversified its policy to achieve a specified annual supply risk across a clutch of supplying rate of inflation. nations.  The principle of inflation targeting  India, Japan and Australia have begun is based on the belief that long-term discussions on launching a trilateral Supply economic growth is best achieved by Chain Resilience Initiative (SCRI) to maintaining price stability, and price reduce dependency on China, necessitated stability is achieved by controlling by Beijing’s aggressive political and inflation. military behavior.  Strict inflation targeting is adopted when the central bank is only concerned about keeping inflation as Source: The Hindu close to a given inflation target as possible, and nothing else.  Flexible inflation targeting is adopted when the central bank is to some extent Question 9 also concerned about other things, for instance, the stability of interest rates, Consider the following statements with exchange rates, output and reference to inflation targeting in India: employment.

1. Flexible inflation targeting is adopted when the central bank is to some extent Source: The Mint also concerned about other things, for instance, the stability of interest rates, exchange rates, output and employment Question 10 2. Strict inflation targeting is adopted when the central bank is only concerned about At present, which of the following countries keeping inflation as close to a given from the South Asia are members of the UN inflation target as possible, and nothing Human Rights Council: else. 1. India 2. Bangladesh Select the correct answer using the code 3. Nepal given below. 4. Sri Lanka

Which of the statements given above is/are correct? a) 1 and 2 only b) 2 and 4 only c) 1, 2 and 3 only d) 1,2,3, and 4

Answer: c Explanation:

Source: The Hindu and UNHRC